xxiii olimpiadas españolas de física, enunciados y soluciones de las pruebas propuestas - pag...

214
XXIII OLIMPIADA ESPAÑOLA DE FÍSICA ENUNCIADOS Y SOLUCIONES DE LAS PRUEBAS PROPUESTAS Bilbao, 20 al 23 de abril de 2012 REAL SOCIEDAD ESPAÑOLA DE FÍSICA

Upload: solomon-krane

Post on 01-Jan-2016

158 views

Category:

Documents


4 download

DESCRIPTION

Colección de enunciados resueltos de las olimpiadas españolas de física 2012.

TRANSCRIPT

Page 1: XXIII olimpiadas españolas de física, enunciados y soluciones de las pruebas propuestas - pag 214.pdf

XXIII OLIMPIADA ESPAÑOLA DE FÍSICA

ENUNCIADOS Y SOLUCIONES

DE LAS

PRUEBAS PROPUESTAS

Bilbao, 20 al 23 de abril de 2012

REAL SOCIEDAD ESPAÑOLA DE FÍSICA

Page 2: XXIII olimpiadas españolas de física, enunciados y soluciones de las pruebas propuestas - pag 214.pdf

XXIII OLIMPIADA ESPAÑOLA DE FÍSICA

P1. Un deporte de invierno: los saltos de esquí.

En esta espectacular modalidad deportiva, los espectadores se mantienen en vilo desde que el atleta inicia el descenso por la pista hacia el trampolín hasta que, tras saltar al "vacío", realiza un estético vuelo, aterriza suavemente, frena y se detiene. Este atleta, mediante un largo y duro entrenamiento, ha adquirido la suficiente destreza para lograr que la física esté de su parte permitiéndole mejorar sus marcas.

El perfil de la pista de saltos, mostrado en la figura 1, presenta cuatro partes: la primera es la zona de aceleración, en la que los saltadores que han partido de alguna de las salidas escalonadas, sin velocidad inicial, descienden por una pronunciada pendiente hasta el trampolín, en el que la pendiente se reduce hasta un ángulo α. En el borde S del trampolín, el saltador inicia el vuelo sobre la zona de salto, que tiene forma aproximadamente parabólica y una longitud L característica de la instalación: se habla así de pistas de 50, 80 ó 90 m.

La tercera zona, P - K, es la de aterrizaje, de longitud M y pendiente constante de ángulo β. A partir del límite K de esta región, el denominado punto crítico, se inicia la cuarta zona, de frenado, en la que no debe producirse el aterrizaje pues disminuye rápidamente la pendiente y el impacto sería peligroso. Según sean las condiciones meteorológicas, el estado de la pista y la calidad de los competidores, los jueces deciden el punto de partida adecuado (una de las salidas escalonadas) para conseguir que el aterrizaje se realice siempre en la zona adecuada, y no en la de frenado. En concreto, los jueces fijan la altura h0 del punto de salida respecto al borde S del trampolín.

Los principales parámetros de un gran trampolín, (L ≈ 90 m), se encuentran indicados en el cuadro de datos de la figura 1.

a) Suponiendo que en el descenso por la zona de aceleración (desde la salida hasta el punto S), el rozamiento con el suelo y el aire disipan una energía Q que es igual al 20 % de la energía mecánica inicial, determine y calcule el módulo de la velocidad de despegue del saltador, 0v , sabiendo que ha iniciado el descenso, sin velocidad inicial, desde una salida con la altura 0h dada en el cuadro de valores.

DATOS

20

m/s819

m046º040º007m004m022m079m043

,g

,h,

,,d

,M,N,H

=

=======

βα

Sv

nv

0v α

Fig. 1

Salidas escalonadas

Zona de aceleración

Zona de frenado

H

N

d

α L

P

K

M

Trampolín

Zona de aterrizaje

Zona de saltoS

0h

0v

β

K

Page 3: XXIII olimpiadas españolas de física, enunciados y soluciones de las pruebas propuestas - pag 214.pdf

XXIII OLIMPIADA ESPAÑOLA DE FÍSICA

A pesar de que el trampolín tiene una pendiente final descendente α, la velocidad real de salida del saltador, Sv , es prácticamente horizontal, pues se da un impulso perpendicular a la pista justo antes de iniciar el vuelo. Para ello, el saltador desciende con las piernas flexionadas y las extiende en el extremo final del trampolín, haciendo un gesto similar al de un salto, sin carrerilla, perpendicular al suelo. Así, además de descender en una postura más aerodinámica, consigue imprimirse una velocidad nv perpendicular al trampolín y alargar el salto. (Observa el diagrama vectorial indicado en la figura 1).

b) Calcule cuál debe ser el valor de la velocidad nv y compruebe si podrá conseguirla un atleta capaz de realizar un salto vertical de altura m600,he = , ¡sin carrerilla!

Suponga que la velocidad de salida es exactamente horizontal y no tenga en cuenta los efectos de la fuerza de fricción con el aire ni los de sustentación, debidos a la aerodinámica del saltador. (En la práctica el segundo efecto domina sobre el primero, de forma que los saltos reales son algo más largos que los idealizados parabólicos).

c) Obtenga la ecuación de la trayectoria que hipotéticamente describirá el saltador, referida al sistema de coordenadas OXY indicado en la figura 2 que se encuentra en la “hoja de respuestas” adjunta. En dicha figura se representa también el perfil de la pista (sombreado), desde el punto de salida S del trampolín hasta el punto crítico K.

d) Sobre la figura 2, haga una representación gráfica de la trayectoria y determine aproximadamente el punto C de aterrizaje, con sus correspondientes coordenadas Cx e Cy .

e) Determine las componentes de la velocidad fv del saltador cuando toca la pista en la zona de aterrizaje. Calcule el ángulo ϕ que forma la velocidad con el eje OX.

f) Determine y calcule el ángulo, φ , que forma la velocidad final fv con la pista.

Page 4: XXIII olimpiadas españolas de física, enunciados y soluciones de las pruebas propuestas - pag 214.pdf

XXIII OLIMPIADA ESPAÑOLA DE FÍSICA

P1. Hoja de respuestas

Apartado Resultados analíticos Resultados numéricos Puntos

a =0v =0v 1,0 + 0,5

b =nv ¿Es el saltador capaz? SI o NO

1,5

c =y 1,5

d =Cx =Cy 2,5

e =fxv =fyv =ϕ 1,0 + 0,5

f =φ =φ 1,0 + 0,5

X (m)

Fig. 2

20

40

P

20 40 60 80 100

10

30

50

Y (m)

O

K

S Sv

β

Page 5: XXIII olimpiadas españolas de física, enunciados y soluciones de las pruebas propuestas - pag 214.pdf

XXIII OLIMPIADA ESPAÑOLA DE FÍSICA

P1. Solución

a) El saltador inicia el descenso sin velocidad inicial, desde una altura 0h respecto al borde de salida del trampolín, S. Se sabe que la energía disipada, Q, es el 20 % de la energía mecánica inicial. Si se toma como referencia para la energía potencial gravitatoria la horizontal por el punto S, se tiene

Qmvhgm += 200 2

1 ,

donde 02,0 hgmQ =

Por tanto

00 6,1 hgv =

m/s9,260 =v

b) Del diagrama de velocidades de la figura del enunciado se deduce que

αtgvvn 0= ⇒ m/s3,3=nv

¿Es el saltador capaz de imprimirse esta velocidad, teniendo en cuenta que en los entrenamientos en el gimnasio es capaz de saltar en vertical una altura m6,0=eh ? Teniendo en cuenta de nuevo la conservación de la energía

ee hgmmv =221

De donde

m/s4,32 == ee hgv ⇒ ne vv >

En conclusión, el saltador SI que es capaz de salir de la pista de saltos con velocidad horizontal.

c) Necesitamos deducir la ecuación de la trayectoria que seguirá el saltador, considerando que inicia el salto en el extremo del trampolín con una velocidad inicial horizontal Sv , de módulo

220 nS vvv += ⇒ m/s127,vS =

Emplearemos el sistema de coordenadas dado en la figura 2, con eje vertical OY pasando por el punto de salida y eje horizontal, OX, pasando por el punto K. En la figura 2 se aprecia que la ordenada del punto de salida S es aproximadamente

m57≈Sy

De una forma más precisa, la ordenada inicial puede deducirse a partir de los datos del enunciado

βsenMHyS += ⇒ m157,yS =

Suponiendo que el salto se realiza en el vacío, la trayectoria del saltador sería la de un “tiro parabólico” desde el punto S de coordenadas 0=Sx e m157,yS = , con una velocidad inicial horizontal, de módulo

m/s1,27=Sv . En consecuencia, las ecuaciones del movimiento son

⎭⎬⎫

−==

tgvvv

y

Sx (1) ⇒ ⎪⎭

⎪⎬⎫

−=

=

2

21 tgyy

tvx

S

S (2)

Eliminando el tiempo se obtiene la ecuación cartesiana de la trayectoria

222

xvgyy

SS −=

Page 6: XXIII olimpiadas españolas de física, enunciados y soluciones de las pruebas propuestas - pag 214.pdf

XXIII OLIMPIADA ESPAÑOLA DE FÍSICA

Teniendo en cuenta los valores numéricos, en unidades del SI, la ecuación resulta

2310696157 x,,y −×−= (3)

d) En la Tabla I se muestran las coordenadas de una serie de puntos deducidos por medio de (3). Con ellos se puede trazar, de forma aproximada, la trayectoria del salto sobre la figura 3, (análoga a la figura 2, que se encuentra en la “hoja de respuestas”).

Tabla 1

x (m) 20 40 60 80 92,4

y (m) 54,5 46,4 33,0 14,3 0

La intersección de la trayectoria con el área sombreada del perfil, permite estimar que el punto C de “toma de tierra” está próximo a P y dentro de la zona de aterrizaje. Concretamente las coordenadas de C son

m0,82=Cx ; m0,12=Cy (4)

e) De (1) y (2) se tiene

Sx vv = ; xvgvS

y −=

Luego en el punto C se tiene

C

Sfy

Sfx

xvgv

vv

−=

=

El ángulo ϕ que forma la velocidad, fv , al llegar a C, con la horizontal, viene dado por

xf

yf

vv

artg=ϕ

Tomando el valor absoluto de yfv , la expresión del ángulo ϕ es

2S

C

vxgartg=ϕ ⇒ º7,47=ϕ

f) El ángulo, φ , que forma la velocidad final con la pista en el punto de aterrizaje es, por fin

βϕφ −= ⇒ º7,7=φ

Page 7: XXIII olimpiadas españolas de física, enunciados y soluciones de las pruebas propuestas - pag 214.pdf

XXIII OLIMPIADA ESPAÑOLA DE FÍSICA

P1. Hoja de respuestas

Apartado Resultados analíticos Resultados numéricos Puntos

a 00 6,1 hgv = m/s9,260 =v 1,0 + 0,5

b m/s3,3=nv ¿Es el saltador capaz? SI

1,5

c 2310696157 x,,y −×−= 1,5

d m082,xC = ; m012,yC = 2,5

e Sfx vv = ; CS

fy xvgv −= º7,47=ϕ 1,0 + 0,5

f βϕφ −= º7,7=φ 1,0 + 0,5

X (m)

Fig. 3

20

40

P

20 40 60 80 100

10

30

50

Y (m)

O

K

S Sv

ϕ

φ

C

fv

β

Page 8: XXIII olimpiadas españolas de física, enunciados y soluciones de las pruebas propuestas - pag 214.pdf

XXIII OLIMPIADA ESPAÑOLA DE FÍSICA

P2. El electrómetro monofilar de Wulf.

Con objeto de demostrar que la radiación en la superficie de la Tierra provenía de sustancias radiactivas existentes en el suelo, el sacerdote jesuita alemán Theodor Wulf diseñó y construyó un electrómetro de hilo que lleva su nombre. Con este instrumento, en 1910, quiso demostrar que la radiación debía disminuir con la altura. El resultado de la experiencia, realizada en la torre Eiffel, resultó negativo. La variación era pequeña pero en sentido contrario: aumentaba con la altura. Esto obligó a admitir la existencia de una radiación de origen externo que competía con la emanada de la propia Tierra. En 1912 el físico austriaco Victor Francis Hess, hizo mediciones ascendiendo en globo hasta los 5000 m: "La mejor explicación para los resultados de mis observaciones se basa en el supuesto de que una radiación de gran poder penetrante entra en nuestra atmósfera desde arriba". Posteriores medidas, realizadas desde globos no tripulados, le llevaron a la conclusión de que la intensidad de la radiación procedente del exterior (rayos cósmicos) aumenta con la altitud, varía con la latitud y es algo más intensa de día que de noche. Victor Hess, que puede considerarse el “padre” de los rayos cósmicos, recibió el Premio el Nobel de Física en 1936.

Los electrómetros son aparatos para medir diferencias de potencial o cargas eléctricas. Aunque existen diversos tipos, vamos a centrarnos en el electrómetro de Wulf de la figura 1, que se expone en la Facultad de Ciencias de la Universidad de Zaragoza y cuyo esquema se representa en la figura 2.

En esencia, está constituido por una carcasa por cuyas caras laterales salen al exterior, perfectamente aislados, los soportes conductores de dos placas metálicas paralelas, A y B. Un fino hilo conductor HC tiene su extremo H conectado eléctricamente al borne 1 del aparato, y su extremo inferior está sujeto a la carcasa por medio de un bucle de cuarzo C, aislante, que permite regular la fuerza de tensión del hilo. El aparato dispone de un sistema lateral de iluminación, que permite observar el hilo con un microscopio dotado de un micrómetro. De esta forma es posible medir con gran precisión las pequeñas desviaciones laterales del hilo que se producen cuando entre los bornes 1 y 2 se aplica una diferencia de potencial VΔ .

Las placas A y B, separadas una distancia d, se conectan a dos baterías de fem V0 cada una, como se indica en la figura 2. Por tanto, entre dichas placas se establece un campo eléctrico que, para simplificar el problema, puede considerarse uniforme. Si el hilo HC no está cargado no sufrirá ninguna fuerza electrostática, como se representa en la figura 3. Pero si existe una diferencia de potencial 0>VΔ , el hilo adquirirá una carga q+ y tenderá a desplazarse lateralmente hacia la placa conductora negativa (figura 4) hasta que la fuerza electrostática esté compensada por las componentes horizontales de la fuerzas de tensión T

r en los extremos del hilo.

Dado que la carga q del hilo es extremadamente pequeña, también lo será el desplazamiento del hilo, lx << , lo que justifica la necesidad del microscopio. En consecuencia, el ángulo α que forma la tensión del

hilo en sus extremos con la vertical será también muy pequeño ( ααα gsen t≈≈ ) y, por la misma razón, se puede considerar que el módulo T de la tensión se mantiene constante e independiente de x. Como

Fig. 1 Fig. 2

V0 V0

H

B

C 2

1 A

Page 9: XXIII olimpiadas españolas de física, enunciados y soluciones de las pruebas propuestas - pag 214.pdf

XXIII OLIMPIADA ESPAÑOLA DE FÍSICA

Fig. 6

S

I

b

1

2

simplificación adicional, puede suponerse que, en equilibrio, el hilo es prácticamente recto en el espacio comprendido entre las placas, como se representa en la figura 5.

a) Determine el módulo del campo eléctrico E existente entre las placas A y B, en función de 0V y d.

Cuando entre los bornes 1 y 2 se establece la diferencia de potencial VΔ , el hilo adquiere una carga q que se reparte uniformemente por el hilo como una densidad lineal de carga Lq /=λ . La parte del hilo comprendido entre las placas, cuya carga es lλ , por efecto del campo eléctrico uniforme sufrirá un desplazamiento x, tal como se muestra en la figura 5.

b) Deduzca la expresión que relaciona el desplazamiento x del hilo, con de V0, L, l, d, T y q.

Para modificar el rango de medidas puede modificarse la tensión T del hilo, aunque no es sencillo determinar su valor. Tampoco es fácil encontrar la relación entre la carga del hilo y la diferencia de potencial VΔ que se aplica entre los bornes del electrómetro. Sin embargo, para pequeños valores de x, la desviación del hilo y el voltaje aplicado VΔ son proporcionales, es decir

VKx Δ= . Suponga que, mediante una operación previa de calibrado, se sabe que la constante de proporcionalidad es m/V10595 6−×= ,K .

En estas condiciones, como se muestra en la figura 6, se unen los bornes 1 y 2 del electrómetro, a través de un interruptor I, a las armaduras S y S´ de un condensador plano cargado. Las armaduras son circulares, de radio

m150,0=r , y están separadas una distancia m120,0=b . Cuando se cierra el interruptor I, se observa en el electrómetro una desviación del hilo mm139,01 =x .

Fig. 4

T

T

x

Fig. 3

d

T

T

l L

Fig. 5

T

T

x

α

α

x

Page 10: XXIII olimpiadas españolas de física, enunciados y soluciones de las pruebas propuestas - pag 214.pdf

XXIII OLIMPIADA ESPAÑOLA DE FÍSICA

c) Determine y calcule la diferencia de potencial 1VΔ existente entre las armaduras del condensador, así como su carga 1Q . (Permitividad dieléctrica del aire: 21212 mNC10868 −−−×= ,ε ).

A partir del instante en que se cierra el interruptor I se observa que la desviación del hilo 1x disminuye lentamente hasta anularse, lo que significa que la diferencia de potencial entre las placas del condensador disminuye con el mismo ritmo hasta hacerse cero. Si no existiese ningún tipo de corriente de fuga a través de los materiales aislantes del montaje, el fenómeno se debería exclusivamente a que algún tipo de radiación, suficientemente energética, ioniza las moléculas del gas (aire) existente entre las placas. Los iones positivos emigran a la placa negativa y los negativos a la positiva y, paulatinamente, se descarga el condensador.

d) Si el tiempo que transcurre desde que se acciona el interruptor hasta que 01 ≈x es s10667 3×= ,τ , y suponiendo que la descarga del condensador se deba solo a ionizaciones simples del aire entre sus armaduras (cada molécula ionizada da lugar a un solo electrón y a un ion positivo), determine y calcule el número N de ionizaciones que se realizan por segundo y por 3cm en el espacio comprendido entre las armaduras del condensador. (carga elemental C106021 19−×= ,e ).

Page 11: XXIII olimpiadas españolas de física, enunciados y soluciones de las pruebas propuestas - pag 214.pdf

XXIII OLIMPIADA ESPAÑOLA DE FÍSICA

P2. Solución

a) En la figura 7 se representan exclusivamente las conexiones de las placas del electrómetro con las dos baterías y la conexión a tierra, y es fácil ver que la diferencia de potencial entre ambas es 02V . Dado que dichas placas constituyen un condensador plano, el módulo del campo eléctrico en su interior es

dVE 02

=

b) Cuando el hilo tenga carga positiva, tenderá a desplazarse hacia la placa negativa, la B en nuestro caso. Aceptando la simplificación propuesta en el enunciado, consideremos como sistema mecánico en equilibrio la porción de hilo entre las placas (figura 8). Las fuerzas exteriores que actúan son las tensiones en sus extremos, de módulo T, y la fuerza electrostática F debida al campo eléctrico. Si λ es la densidad lineal de carga eléctrica del hilo, esta fuerza es

dVlElF 02λλ == (1)

En el equilibrio, la fuerza resultante horizontal debe ser nula, por lo que

0sen2 =− αTF (2)

Teniendo en cuenta el pequeño valor de α ,

αα tgsen ≈

De la figura 5 del enunciado es fácil deducir que

( ) 2/tg

lLx

−=α

De donde,

lL

xTdVl

−=

222 0λ ⇒ ( )TV

dllLx

20λ−

=

Y como Lq /=λ , resulta

( ) q

TV

LdllLx

20−

= (3)

c) Al conectar las armaduras del condensador a los bornes del electrómetro se observa una desviación del hilo mm13901 ,x = , luego, teniendo en cuenta el valor de la constante de calibración K, el voltaje es

KxV 1=Δ ⇒ V924,V =Δ

La capacidad del condensador plano viene dada por bAC /ε= , donde A es el área de las armaduras. Por consiguiente,

brC

2πε=

Fig. 7

V0

A B

V0

Fig. 8

T

T

l

α

F

α

Page 12: XXIII olimpiadas españolas de física, enunciados y soluciones de las pruebas propuestas - pag 214.pdf

XXIII OLIMPIADA ESPAÑOLA DE FÍSICA

Si la diferencia de potencial a la que se conecta es VΔ , el valor absoluto de la carga de cada una de sus armaduras será VCQ Δ= , por lo que, en función de los parámetros del problema

Kx

brQ

2πε= ⇒ C10301 10−×= ,Q

d) La carga de cada armadura se neutraliza al cabo del tiempo τ como consecuencia de las cargas de signo opuesto que le llegan procedentes de las ionizaciones que han tenido lugar en el aire existente entre las placas, cuyo volumen es br 2π . Por lo tanto, el número de ionizaciones que han tenido lugar por segundo y por unidad de volumen es

breQN 2πτ

=

Teniendo en cuenta la expresión de Q,

Kx

beN 2τ

ε= ⇒ )m/(sesionizacion10512 36 ⋅×= ,N

)cm/(sesionizacion512 3⋅= ,N

P2. Tabla de respuestas

Apartado Resultados analíticos Resultados numéricos Puntos

a dVE 02

= 1,5

b ( ) qT

VLd

llLx2

0−=

3

c KxV 1=Δ

Kx

brQ

2πε=

V924,V =Δ

C10301 10−×= ,Q 2 + 1

d Kx

beN 2τ

ε=

),N 3cm/(sesionizacion512 ⋅= 1,5 +1

Page 13: XXIII olimpiadas españolas de física, enunciados y soluciones de las pruebas propuestas - pag 214.pdf

XXIII OLIMPIADA ESPAÑOLA DE FÍSICA

P3. El giróscopo interferencial de fibra óptica.

Un giróscopo mecánico es un aparato consistente en un disco que gira rápidamente sobre un eje libre que, debido a la conservación del momento angular, tiende a mantenerse en una dirección constante. Fue inventado por el físico francés Jean Bernard Léon Foucault (1819-1868) y se utiliza habitualmente para mantener la orientación en el espacio, en particular para la estabilización del rumbo de barcos, aviones y satélites. Pero existen otros dispositivos no mecánicos, más precisos, versátiles y sencillos, que se usan con los mismos fines.

El también físico francés Georges Sagnac descubrió en 1911 que “una onda electromagnética que se mueve en un camino cerrado es influenciada por la velocidad angular del sistema”. Basados en el llamado “efecto Sagnac”, a partir de 1960 comenzó la utilización de los giróscopos ópticos, que derivaron en 1970 en los giróscopos interferenciales de fibra óptica. En este ejercicio se describen, de forma simplificada, estos giróscopos.

Como ejercicio preliminar, considere que un haz de luz láser, de longitud de onda λ (en el vacío), se propaga con velocidad v por una fibra óptica rectilínea, de índice de refracción efectivo1 n, que se mueve longitudinalmente con velocidad V.

a1) Determine el tiempo 1t que tarda la luz en recorrer una longitud L de fibra, cuando dicha luz se propaga en el mismo sentido que se mueve la fibra (figura 1.a), y el tiempo 2t cuando lo hace en sentido opuesto (figura 1.b).

a2) Determine la diferencia entre estos dos tiempos, 21 ttt −=Δ . Aproxime el resultado teniendo en cuenta que vV <<

Considere ahora que la fibra óptica se curva en una circunferencia de longitud RL π2= , y que gira con velocidad angular Ω en torno a su centro en sentido horario. Por la fibra se propagan dos haces de luz en sentidos opuestos.

b) Determine la diferencia, tΔ , entre los tiempos +t y −t que tarda la luz en recorrer el anillo circular de longitud L en sentidos horario y antihorario, respectivamente (Figuras 2.a y 2.b). Exprese el resultado en función del área A del anillo.

1 El índice de refracción efectivo n hace referencia a la velocidad de avance de la luz a lo largo del eje de la fibra,

n/cv = , donde c es la velocidad de la luz en el vacío.

V

L Fig. 1.a

v

V

L

Fig. 1.b

v

v

Ω

+t

Fig. 2.a

v

Ω

−t

Fig. 2.b

Page 14: XXIII olimpiadas españolas de física, enunciados y soluciones de las pruebas propuestas - pag 214.pdf

XXIII OLIMPIADA ESPAÑOLA DE FÍSICA

El funcionamiento de un giróscopo de fibra óptica se esquematiza en la figura 3. Un 50% de la luz emitida por el láser se refleja en el divisor de haz (+), recorre N espiras circulares de fibra en sentido horario y, tras reflejarse en el divisor de haz, incide en el detector. El otro 50% de la luz se transmite a través del divisor de haz (-), recorre las N espiras en sentido antihorario e incide en el detector después de transmitirse a través del divisor.

Si el dispositivo no gira ( 0=Ω ), al detector llegan dos haces coherentes que han recorrido el mismo camino óptico y que, por tanto, se superponen en fase e interfieren constructivamente. Sin embargo, cuando todo el conjunto de la figura 3 gira con velocidad angular Ω en torno al centro de las espiras, entre ambos haces habrá una diferencia de fase que cambiará el estado interferencial en el detector (efecto Sagnac).

Determine:

c1) La diferencia entre los tiempos que tardan los dos haces en llegar al detector, TΔ , cuando el dispositivo gira con una velocidad Ω en torno a su centro.

c2) La diferencia de fase, δ , entre los dos haces de luz.

En los giróscopos de fibra óptica, el sistema optoelectrónico de detección analiza el estado interferencial de los dos haces, determina esta diferencia de fase y permite deducir con gran precisión la velocidad angular de rotación y, en consecuencia, los cambios de rumbo.

d) Determine la mínima velocidad angular de rotación, minΩ , para que ambos haces interfieran destructiva-mente en el detector, de forma que se anule la intensidad total.

Suponga que el sistema de detección es capaz de apreciar variaciones de la diferencia de fase mrad1=δΔ .

e) Con los datos que se indican a continuación, calcule la precisión ΔΩ con que es posible obtener la velocidad angular de rotación.

Radio de las espiras de fibra óptica: m100,5 2−×=R

Número de espiras: 3200=N

Índice de refracción efectivo de la fibra: 48,1=n

Longitud de onda (en el vacío) del láser: μm3,1=λ

Velocidad de la luz en el vacío: m/s100,3 8×=c

Fig. 3

Detector de luz

Divisor de haz Láser

R

Bobina de N espiras de fibra óptica

O

+ -

+-

-

+

Ω

Page 15: XXIII olimpiadas españolas de física, enunciados y soluciones de las pruebas propuestas - pag 214.pdf

XXIII OLIMPIADA ESPAÑOLA DE FÍSICA

P3. Solución

a1) Los tiempos 1t y 2t que tarda la luz en recorrer el segmento de la fibra de longitud L, cuando dicha fibra se mueve con velocidad V, en el mismo sentido que la luz o en sentido contrario, verifican las expresiones siguientes, respectivamente,

⎭⎬⎫

−=+=

22

11

tVLtvtVLtv

VvLt

VvLt

+=

−=

2

1

a2) La diferencia, tΔ de los tiempos anteriores es

2221211

VvLV

VvVvLttt

−=⎟

⎠⎞

⎜⎝⎛

+−

−=−=Δ

Teniendo en cuenta que ncvV /=<< , las expresión anterior de tΔ puede aproximarse

22vLVt ≈Δ

La velocidad de propagación de la luz en la fibra es ncv /= , de forma que

2

22c

LVnt =Δ (1)

b) La situación es análoga a la del apartado anterior, con RL π2= y RV Ω= . Por lo tanto

ΩπΔ 2

224c

Rnt =

En función del área de la espira circular 2RA π=

ΩΔ 2

24c

Ant = (2)

c1) De acuerdo con el enunciado, al detector llegan dos haces. Uno ha recorrido las N espiras de la bobina en sentido horario y el otro en sentido antihorario. Por lo tanto, teniendo en cuenta el resultado anterior (2),

tNT ΔΔ = ⇒ ΩΔ 2

24c

AnNT = (3)

c2) La diferencia de fase, δ , entre los dos haces es

TΔωδ =

donde λπω /c2= es la frecuencia angular de la luz.

Teniendo en cuenta (3)

Ωλ

πδc

ANn28= (4)

Page 16: XXIII olimpiadas españolas de física, enunciados y soluciones de las pruebas propuestas - pag 214.pdf

XXIII OLIMPIADA ESPAÑOLA DE FÍSICA

d) Los haces que alcanzan el detector interferirán destructivamente cuando πδ m= , con m = 1, 3, 5…

La frecuencia minΩ corresponderá al primer mínimo de interferencia, es decir πδ = . Sustituyendo en (4),

ANn

c2min 8λΩ =

e) Si la mínima diferencia de fase detectable es δΔ , tomando incrementos en (4) se obtiene la precisión ΔΩ con que es posible obtener la velocidad angular de rotación

δΔπ

λΔΩANn

c28

=

Con mrad1=δΔ y los datos que figuran en el enunciado, se obtiene

/hº85/sº106,1rad/s108,2 24 =×=×= −−ΔΩ

P3.Tabla de respuestas

Apartado Resultados analíticos Resultados numéricos Puntos

a1

VvLt

VvLt

+=

−=

2

1

0,5

0,5

a2 2

22c

LVnt =Δ 1,5

b ΩΔ 2

24c

Ant =

1

c1 ΩΔ 2

24c

AnNT =

1

c2 Ωλ

πδc

ANn28=

2

d ANn

c2min 8λΩ = 2

e

/hº85/sº106,1

rad/s108,22

4

=×=

=×=−

−ΔΩ 1,5

Page 17: XXIII olimpiadas españolas de física, enunciados y soluciones de las pruebas propuestas - pag 214.pdf

XXIII OLIMPIADA ESPAÑOLA DE FÍSICA

Prueba experimental Constante de Planck y comportamiento de un LED

Objetivo.

Se va a construir un circuito eléctrico para alimentar LEDs de diferentes colores y obtener un valor aproximado de la constante de Planck. Además se determinará, para uno de estos LEDs, el valor de una constante característica llamada factor de idealidad. Materiales.

- Cinco LEDs, de colores diferentes. - Pila de 9 V. - Potenciómetro y resistencia de protección. - Dos polímetros con sondas y cuatro pinzas (cocodrilos). - Regleta de conexión. - Destornillador.

Modelo teórico2.

Un LED (Light-Emitting Diode) es un dispositivo optoelectrónico que emite luz cuando circula por él una corriente eléctrica I (figura 1). Para que circule esta corriente es necesario que la diferencia de potencial entre sus terminales, V, sea superior a un cierto valor umbral, 0V .

En esencia, un LED es un semiconductor en el que los electrones se encuentran en niveles de energía muy próximos que forman bandas. La de menor energía es la banda de valencia, que está normalmente llena de electrones. Existe otra banda, con energía superior y que contiene pocos electrones, llamada banda de conducción. Ambas bandas están separadas por una banda prohibida, de energía E (figura 2).

Para que un electrón pueda excitarse desde la banda de valencia hasta la de conducción debe absorber, como mínimo, una energía 0qVE = , donde q es la carga elemental (valor absoluto de la carga del electrón). Esta energía es aportada por la batería que alimenta el circuito con el LED.

Cuando el electrón se desexcita y regresa a la banda de valencia se emite un fotón de energía νhE = , donde h es la constante de Planck y ν la frecuencia de la radiación emitida. Por tanto, con este modelo simplificado, sería de esperar que se cumpliese la igualdad qV0 = hν . En la práctica, se encuentra esta relación lineal entre 0V y ν, pero con un término independiente, C, aproximadamente constante, que no puede justificarse con este modelo, es decir

νqhCV +≈0 (1)

Por otra parte, para V superior a 0V , la corriente I aumenta de modo aproximadamente exponencial (figura 3), en la forma

( )kTqVseII η≈ (2)

donde sI es la llamada corriente de saturación, k la constante de Boltzmann, T la temperatura absoluta y η se conoce como factor de idealidad del LED.

2 Se presenta un modelo muy simplificado, suficiente para los objetivos de esta prueba experimental. El valor de la

constante de Plank que se obtiene es correcto en orden de magnitud, pero puede diferir del valor real en más de un 10 %, dependiendo de los LEDs concretos empleados en las medidas.

Fig. 3

V

I

V0

Banda de conducción

Banda de valencia

E qV0 Fotón

Fig. 2

Fig. 1

LED V

I+

Page 18: XXIII olimpiadas españolas de física, enunciados y soluciones de las pruebas propuestas - pag 214.pdf

XXIII OLIMPIADA ESPAÑOLA DE FÍSICA

Montaje.

El esquema eléctrico del equipo experimental se presenta en la figura 4, donde se indican los puntos de conexión de los cables, la figura 5 es una fotografía del montaje real. En la figura 6 se muestra una fotografía ampliada del potenciómetro.

Instrucciones de montaje:

• El potenciómetro multivuelta presenta, de un lado, dos cables libres (rojo y negro) y, del otro, un conector para la pila de 9 V. No conectes todavía la pila al circuito.

• Conecta el cable negro del potenciómetro a un terminal de la regleta, donde se asegura el contacto apretando el tornillo.

• Conecta la resistencia de protección en el otro terminal de la regleta.

• Conecta las puntas de prueba del miliamperímetro mediante pinzas (“cocodrilos”) al cable rojo del potenciómetro y a la resistencia de protección.

• Con las otras dos pinzas, conecta las puntas de prueba del voltímetro a los terminales de la regleta.

• Los LEDs se conectarán en el otro extremo de la regleta. La patilla más larga del LED es el ánodo (+), por donde debe entrar la corriente.

• El miliamperímetro debe estar en la escala de 2 mA, y el voltímetro en la de 20 V.

• Inicialmente, para anular la tensión de alimentación del LED, gira el tornillo del potenciómetro en sentido antihorario hasta el final de su recorrido.

• Atención: para evitar que se agote la pila, mantén el circuito abierto cuando no estés midiendo.

Fig. 5

Voltímetro Miliamperímetro

Pila Regleta LED

Pinzas

Potenciómetro

Resistencia de protección

Pinzas

Conector

Potenciómetro

Fig. 6

Fig. 4

Potenciómetro Regleta

Pila

Resistencia de protección

Miliamperímetro Voltímetro V LED

AI

+

Page 19: XXIII olimpiadas españolas de física, enunciados y soluciones de las pruebas propuestas - pag 214.pdf

XXIII OLIMPIADA ESPAÑOLA DE FÍSICA

Medidas y preguntas.

1ª parte. Determinación de h.

1.a) Conecta en la regleta el LED infrarrojo (cápsula oscura). Conecta la pila y aumenta la tensión de alimentación del LED, girando el potenciómetro en sentido horario, hasta que circule una corriente de 0,010 mA. Supondremos que, en estas circunstancias, la tensión indicada por el voltímetro es aproximadamente la tensión umbral para este diodo, V0. Anota el valor de V0 .

Restableciendo cada vez el potenciómetro a su posición inicial, repite la medida de V0 para los otros cuatro LEDs: rojo, amarillo, azul y violeta. El aspecto exterior de estos LEDs es similar, pero los distinguirás al hacer pasar corriente.

No olvides desconectar la pila al finalizar esta serie de medidas.

Traslada tus medidas a la Tabla 1 de la hoja de respuestas, donde se indica la longitud de onda de emisión de cada LED. Para calcular la frecuencia ν, recuerda que c = λν, donde c es la velocidad de la luz en el vacío y λ la longitud de onda.

1.b) Representa gráficamente los valores de V0 (en ordenadas) frente a las frecuencias ν (en abscisas).

1.c) Obtén el valor de la pendiente de la recta que mejor se ajusta a los puntos de la gráfica.

1.d) Deduce el valor de h.

1.e) Haz una estimación de la incertidumbre de la pendiente.

1.f) Teniendo en cuenta lo anterior, haz una estimación de la incertidumbre del valor de h. 2ª parte. Determinación del factor de idealidad, η.

Gira el potenciómetro en sentido antihorario hasta el final de su recorrido y conecta el LED rojo en la regleta. Selecciona en el voltímetro la escala de 2 V. Conecta la pila y aumenta la tensión de alimentación hasta que el amperímetro indique, de nuevo, una intensidad de 0,010 mA.

2.a) Partiendo de la situación anterior, aumenta sucesivamente la tensión de alimentación a intervalos regulares de aproximadamente 0,02 V, hasta un valor máximo de 1,70 V. Anota en cada caso los valores de V y de I en la Tabla 2 de la hoja de respuestas. En esta tabla dispones de una columna vacía, para alguna magnitud derivada de las anteriores que necesites en el siguiente apartado.

2.b) A partir de la gráfica y del ajuste que estimes oportunos, determina el valor del coeficiente de idealidad, η, del LED rojo.

2.c) Haz una estimación de la incertidumbre de este coeficiente.

Datos:

Temperatura ambiente: T = (295 ± 3) K

Carga elemental: q = 1,60 × 10-19 C

Velocidad de la luz en el vacío: c = 3,00 × 108 m/s

Constante de Boltzmann: k = 1,38 × 10-23 J/K

Page 20: XXIII olimpiadas españolas de física, enunciados y soluciones de las pruebas propuestas - pag 214.pdf

XXIII OLIMPIADA ESPAÑOLA DE FÍSICA

Constante de Planck y comportamiento de un LED Solución 1ª parte. 1.a) Tabla 1 1.b) Gráfica.

1.c) Pendiente de la recta:

A partir de las coordenadas de los puntos auxiliares A y B

( )( ) 114 s103,00-8,00

V800,0925,2−×

−=

−−

=AB

ABxxyyp sV1025,4 15 ⋅×= −p

1.d) Valor de h:

Según la ecuación (1) del enunciado, la pendiente de la recta anterior es p = h/q. Por tanto

pqh = sJ1080,6 34 ⋅×= −h

LED λ (nm) ν (× 1014Hz) V0 (V) Infrarrojo 938 3,20 0,87

Rojo 632 4,75 1,55 Amarillo 593 5,06 1,66

Azul 464 6,47 2,40 Violeta 405 7,41 2,58

V0 (V)

ν (x 1014 Hz)

B

A

Page 21: XXIII olimpiadas españolas de física, enunciados y soluciones de las pruebas propuestas - pag 214.pdf

XXIII OLIMPIADA ESPAÑOLA DE FÍSICA

1.e) Incertidumbre de la pendiente:

En la siguiente figura se realiza una estimación gráfica de las rectas que, con pendientes máxima y mínima, se ajustan razonablemente a los puntos experimentales

( )( ) sV1093,3

s103,00-7,90V850,0775,2 15

114min ⋅×=×

−=

−−

= −−

′′

′′

AB

ABxxyyp

( )( ) sV1059,4

s103,00-7,90V750,0000,3 15

114max ⋅×=×

−=

−−

= −−

′′′′

′′′′

AB

ABxxyyp

sV1033,02

15minmax ⋅×=−

=Δ −ppp sV103,0 15 ⋅×=Δ −p

Nota: Un cálculo analítico aplicando el método de mínimos cuadrados conduce a resultados similares. Se obtiene, con un nivel de confianza del 68 %,

( ) sV103024 15 ⋅×±= −,,p

Para que el nivel de confianza aumente al 95 %, es necesario triplicar el margen de error.

1.f) Incertidumbre de la constante de Planck

Supuesto que el valor de q es exacto, pqh Δ=Δ sJ105,0 34 ⋅×=Δ −h

En total, el resultado del experimento, expresado con el número adecuado de cifras significativas, es

( ) sJ105,08,6 34 ⋅×±= −h

V0 (V)

ν (x 1014 Hz)

B’’

A’

B’

A’’

Page 22: XXIII olimpiadas españolas de física, enunciados y soluciones de las pruebas propuestas - pag 214.pdf

XXIII OLIMPIADA ESPAÑOLA DE FÍSICA

2ª parte. 2.a) Tabla 2

Midiendo con el LED rojo se obtienen los siguientes resultados:

V (V) I (mA) ln I 1,551 0,010 -11,5 1,580 0,020 -10,8 1,600 0,031 -10,4 1,620 0,050 -9,90 1,640 0,076 -9,48 1,660 0,118 -9,04 1,680 0,180 -8,62 1,700 0,270 -8,22

2.b) Factor de idealidad:

Tomando logaritmos en la ecuación (2) del enunciado

VTk

qII s η+≈ lnln

Es decir, se espera una relación aproximadamente lineal entre ln I y V, con pendiente

kTqp

η=

A continuación se presenta la correspondiente gráfica.

V (V)

ln I

Page 23: XXIII olimpiadas españolas de física, enunciados y soluciones de las pruebas propuestas - pag 214.pdf

XXIII OLIMPIADA ESPAÑOLA DE FÍSICA

La pendiente de la recta a la que se ajustan los puntos experimentales puede obtenerse a partir de las coordenadas de los dos puntos auxiliares indicados

( )1-V1,22

V540,1696,1)70,11(25,8

=−

−−−=p

Por tanto, el factor de idealidad del LED es

Tkp

q=η (3)

78,1=η

2.c) Incertidumbre del factor de idealidad

Las pendientes máxima y mínima pueden estimarse en las rectas de la gráfica anterior, a partir de las

coordenadas de sus puntos extremos

( )1-

min V47,21V1,54-1,710

)65,11(00,8=

−−−=p

( )1-

max V87,22V1,54-1,704

)75,11(00,8=

−−−=p

1-minmax V7,0

2=

−=Δ

ppp

V (V)

ln I

Page 24: XXIII olimpiadas españolas de física, enunciados y soluciones de las pruebas propuestas - pag 214.pdf

XXIII OLIMPIADA ESPAÑOLA DE FÍSICA

Supuestas q y k exactas, las únicas fuentes de error en la determinación del factor de idealidad son las incertidumbres de la pendiente, Δp, y de la temperatura, ΔT. Las incertidumbres respectivas transmitidas a Δη pueden obtenerse numéricamente o tomando incrementos (en valor absoluto) en la expresión (3)

056,02 =Δ

=Δpp

pp

Tkq

p ηη

018,02 =Δ

=ΔTT

TT

kpq

T ηη

La incertidumbre total de η podría estimarse como la suma de las dos anteriores pero, dado que las dos fuentes de error son independientes, es más correcto considerar

059,022 =Δ+Δ=Δ Tp ηηη

Nótese que el margen de error de la temperatura resulta casi irrelevante en el resultado final.

Redondeando a una única cifra significativa, pues se trata de estimaciones, se obtiene por fin

060,=ηΔ

El resultado final del experimento es

06,078,1 ±=η

Page 25: XXIII olimpiadas españolas de física, enunciados y soluciones de las pruebas propuestas - pag 214.pdf

Real Sociedad Española deFísica

XXII OLIMPIADA ESPAÑOLA DE FÍSICA

ENUNCIADOS Y SOLUCIONES

DE LAS

PRUEBAS PROPUESTAS

Murcia, 7 al 10 de abril de 2011

REAL SOCIEDAD ESPAÑOLA DE FÍSICA

Page 26: XXIII olimpiadas españolas de física, enunciados y soluciones de las pruebas propuestas - pag 214.pdf

XXII OLIMPIADA ESPAÑOLA DE FÍSICA

Página 3 de 22

Problema 1. Murcia: Sol, mar y salinas.

La Región de Murcia disfruta de un privilegiado clima mediterráneo. Goza de inviernos suaves y veranos calurosos, teniendo en promedio 300 días de sol al año. Gracias a ello gran parte de su territorio es un vergel: la huerta murciana. Sin embargo, sus recursos hídricos no son especialmente abundantes, las precipitaciones son escasas y concentradas en pocos días. Por ello, a lo largo de toda su historia, los murcianos han sabido aprovechar hasta la última gota de agua, como lo demuestran los numerosos ingenios hidráulicos que han construido: molinos, norias, acequias, azudes, etc., parte de los cuales todavía están en uso. Podemos mencionar los molinos existentes en el río Segura, de visita obligada en el centro de la ciudad de Murcia, y las norias de Abarán y Alcantarilla.

Por otra parte, también han sabido aprovechar su abundante energía solar. Ejemplos de ello son los parques fotovoltaicos, entre los que destaca el existente en Jumilla, el mayor de Europa cuando fue inaugurado en 2008, y múltiples salinas como las de San Pedro del Pinatar en la laguna del Mar Menor, protagonista de este problema.

Refiriéndonos a la radiación solar, se denomina constante solar a la energía que, por unidad de tiempo y unidad de superficie normal a la dirección de propagación, llega a las capas altas de la atmósfera terrestre. Esta constante, que es una densidad superficial de potencia, o intensidad de energía, tiene por valor

2kW/m3661,k = . Debido a la absorción y difusión en la atmósfera, a la superficie de la Tierra sólo llega, en días soleados, una fracción 50,=β de dicha intensidad solar.

Para simplificar el problema admitiremos que la trayectoria aparente del Sol está en un plano perpendicular a la superficie de la Tierra1.

En un día soleado, la energía que se recibe en la superficie de la Tierra depende de la altura angular del Sol, es decir del ángulo θ que se muestra en la figura 1. Naturalmente, este ángulo varía a lo largo del día.

a) Para un día soleado y para una “altura” angular del Sol, θ , determina la potencia P que deposita la radiación solar en un área S de la superficie terrestre.

b) Determina la potencia media, P , que recibe la superficie S a lo largo de un día, es decir para πθ ≤≤0 .

Véanse las Notas 1 y 2 al final del ejercicio.

En las salinas, la energía solar se utiliza para evaporar el agua de mar y extraer la sal disuelta. El proceso es complejo y se lleva a cabo mediante la parcelación de las aguas en distintos estanques: almacenadores, calentadores y cristalizadores en los que se precipita la sal. Son estos últimos estanques los que centrarán la atención de este ejercicio.

1 Realmente la latitud de Murcia es de unos 38º, y el ángulo que forma el plano ecuatorial de la Tierra con el de la eclíptica

es de unos 23º. Por tanto, en verano, el plano de la órbita aparente del Sol forma un ángulo de unos 15º con la vertical.

θ

Fig.1 Oeste

Este

S

Page 27: XXIII olimpiadas españolas de física, enunciados y soluciones de las pruebas propuestas - pag 214.pdf

XXII OLIMPIADA ESPAÑOLA DE FÍSICA Página 4 de 22

Supongamos que los estanques de cristalización de las salinas de San Pedro tienen una profundidad media m150,h = y con una concentración de sal del 4,5% en masa, 0450,cm = . En condiciones de presión y

temperatura medias, la densidad del agua es 33 kg/m10031 ⋅= ,ρ y su calor de vaporización es kJ/kg1042 3⋅= ,L .

c) Considerando que el tiempo medio de insolación en un día es horas1221 =/T , determina el número n de días soleados que se necesitan para evaporar el agua de los estanques de cristalización y calcula su valor.

Estudiemos ahora aspectos relativos a la emisión de energía por el Sol. Como se ha mencionado, la constante solar k es la densidad superficial de potencia que llega a las capas altas de la atmósfera terrestre. A partir de este dato y sabiendo que la distancia Tierra-Sol es m1049,1 11⋅=R ,

d) Determina la potencia total emitida por el Sol, SP , y calcula su valor.

La energía que emite el Sol conlleva una disminución de su masa de acuerdo con la conocidísima fórmula de Einstein 2cmE = , donde c es la velocidad de la luz, m/s109982 8⋅= ,c .

e) Determina la masa que pierde el Sol cada segundo, Sμ , y calcula su valor.

Por último, vamos a estudiar si esta pérdida de masa afecta de forma apreciable al radio de la órbita de la Tierra en torno al Sol.

f) Teniendo en cuenta la ley de Gravitación Universal y la conservación del momento angular de la Tierra respecto al Sol, determina la variación relativa del radio de la órbita terrestre, R/RΔ , en función de la variación relativa de la masa del Sol, SS M/MΔ .

g) Calcula la variación anual del radio de la órbita terrestre, sabiendo que la masa del Sol es kg1099,1 30⋅=SM .

Nota 1 .- El valor medio de una función ( )xf en un intervalo 12 xxx −=Δ se define como

( )dxxfx

fx

x∫=2

1

Geométricamente este valor medio coincide con la altura de un rectángulo de base xΔ y cuya área sea igual a la comprendida entre la curva ( )xf y el eje X, entre 21 y xx , como se muestra en la figura 2.

Nota 2.- ∫ −= ααα cossen d

∫ = ααα sencos d

( )xf

f

1x 2x Fig. 2

Page 28: XXIII olimpiadas españolas de física, enunciados y soluciones de las pruebas propuestas - pag 214.pdf

XXII OLIMPIADA ESPAÑOLA DE FÍSICA

Página 5 de 22

Solución: Problema 1. Murcia: Sol, mar y salinas.

a) De acuerdo con el enunciado, la intensidad que llega a la superficie de la Tierra procedente del Sol es una fracción 50,=β de la constante solar.

kk β=′ (1)

Para determinar la potencia instantánea que deposita la radiación sobre un área S cuando la altura del Sol es θ (figura 1 del enunciado), es preciso considerar la proyección de dicha superficie en dirección perpendicular a los rayos, como se muestra en la figura 3. Como θsenSS =′ , la potencia instantánea, P , en el área S , será

θβ senSkP = (2)

b) Esta potencia P es función de θ , que varía a lo largo del día. La potencia media diaria se calcula, de acuerdo con la Nota 1 del enunciado, evaluando el valor medio de (2) desde que el Sol sale hasta que se pone, es decir desde 0=θ hasta πθ = .

∫=π

θθπ

β0sen1 dSkP ⇒ SkP β

π2

= (3)

c) El volumen de agua de mar en los estanques cristalizadores de las salinas de San Pedro es ShV = , donde S es ahora el área de dichos estanques. Por lo tanto la masa de agua de mar que contienen es Shm ρ= . Como la concentración de sal es 0450,cm = , la masa de agua que hay que evaporar es

( )magua cShm −= 1ρ

Por lo tanto, la energía que se necesita para la evaporación es

( )magua cShLmLW −== 1ρ

Como la potencia que recibe la salina es P , durante un día soleado la energía absorbida es igual al producto de P por s10324horas12 4

21 ⋅== ,T / . En consecuencia, el número de días soleados necesarios para extraer la sal será

( ) ( )2121 2

11

/

m

/

mTSk

cShLTP

cShLnβ

πρρ −=

−= ⇒

( )212

1

/

mTk

chLnβ

πρ −=

Teniendo en cuenta los datos numéricos del enunciado, resulta

dias19=n

d) Dado que el Sol emite en todas las direcciones, si la constante solar k es la energía que llega a la Tierra por unidad de tiempo y unidad de superficie, a una esfera de radio R, igual a la distancia Sol-Tierra y centrada en el Sol le llegará toda la energía que el Sol emite por segundo, es decir, la potencia SP que nos piden.

kRPS24π= ⇒ MW1083 20⋅= ,PS

Fig.3

θ

S

Page 29: XXIII olimpiadas españolas de física, enunciados y soluciones de las pruebas propuestas - pag 214.pdf

XXII OLIMPIADA ESPAÑOLA DE FÍSICA Página 6 de 22

e) De acuerdo con el resultado anterior, en un intervalo de tiempo τ el Sol emite una cantidad de energía,

τπ kRWS24=

Y en virtud de la famosa ecuación de Einstein, esta emisión de energía supone que el Sol pierde en ese intervalo de tiempo una masa

2

2

24

ckR

cWM S

SτπΔ ==

Por tanto en un tiempo s1=τ el Sol pierde una masa

2

24c

kRS

πμ = ⇒ kg/s1024 9⋅= ,Sμ

f) La ley de gravitación proporciona la relación entre la masa del Sol y el radio orbital R de la Tierra. Si TM es la masa de la Tierra y ω su velocidad angular orbital, se tiene

RMR

MMG TTS 2

2 ω= ⇒ 32

RG

M Sω

= (4)

Nótese que una variación de SM afecta a R y a ω, pero estas dos variables no son independientes. La fuerza de interacción gravitatoria es central, luego debe conservarse el momento angular de la Tierra respecto al Sol

20 RML T ω= (5)

Eliminando ω entre (4) y (5), queda

RRMG

LMT

S11

2

20 γ== (6)

Donde 220 TMG/L=γ es una constante. Tomando incrementos en (6)

2RRM S

ΔγΔ −= (7)

El signo negativo de (7) significa que una perdida de masa del Sol implica un aumento de la distancia Sol-Tierra. ¡Nos alejamos del Sol poco a poco!

Dividiendo ambos miembros de (7) por la masa (actual) del Sol y teniendo en cuenta (6), se obtiene

S

SMM

RR ΔΔ

−=

g) En un tiempo s10153año1 7⋅== ,T la pérdida de masa del Sol es

kg1031 17⋅−=−= ,TM SS μΔ

Y el aumento de la distancia Tierra-Sol resulta

S

SMMRR ΔΔ −= ⇒ cm01,R =Δ

Page 30: XXIII olimpiadas españolas de física, enunciados y soluciones de las pruebas propuestas - pag 214.pdf

XXII OLIMPIADA ESPAÑOLA DE FÍSICA

Página 7 de 22

Problema 2. Tablón oscilante.

Se coloca un tablón delgado y homogéneo, de masa M y longitud L , sobre un par de rodillos que giran con la misma velocidad angular constante, pero en sentidos opuestos. En la figura 1 se muestra este sistema cuando el tablón está colocado simétricamente respecto a los rodillos. La distancia entre los ejes de los rodillos es b y el coeficiente de rozamiento entre el tablón y dichos rodillos es μ .

a) Cuando el tablón se aparta una distancia x de la posición simétrica, como se representa en la figura 2, dibuja un diagrama en el que se muestren las fuerzas que actúan sobre el tablón.

b) Demuestra que el tablón permanece en equilibrio si se coloca exactamente en la posición simétrica respecto a los cilindros ( 0=x ).

Cuando el tablón se libera en una posición como la representada en la figura 2, es decir separado una distancia x de la posición de equilibrio, realiza un movimiento oscilatorio armónico. Se supone que la velocidad angular de los rodillos es lo suficientemente elevada para que en ningún momento el tablón deje de deslizar sobre ellos.

c) Determina el periodo T de las oscilaciones del tablón.

Estando el tablón en la posición de equilibrio, se le aplica un impulso horizontal de magnitud I , de forma que empieza a oscilar en torno a dicha posición de equilibrio.

d) Determina el máximo impulso que se puede aplicar al tablón, maxI , para que permanezca siempre apoyado sobre los dos rodillos.

Fig. 1

b

L Y

X

Fig. 2

x

Y

X

Page 31: XXIII olimpiadas españolas de física, enunciados y soluciones de las pruebas propuestas - pag 214.pdf

XXII OLIMPIADA ESPAÑOLA DE FÍSICA Página 8 de 22

Solución Problema 2. Tablón oscilante

a) Las fuerzas que actúan sobre el tablón son las representadas en la figura 3. Como en todo momento existe deslizamiento entre el tablón y los rodillos, el módulo de cada fuerza de rozamiento alcanza su valor máximo:

11 NFr μ= y 22 NFr μ=

b) En la posición simétrica ( 0=x ) las reacciones normales son iguales: 221 /MgNN == , por lo que las fuerzas de rozamiento (máximas puesto que existe deslizamiento) tienen el mismo módulo y, al ser de sentidos opuestos, la fuerza resultante horizontal es nula. En consecuencia el tablón permanece en equilibrio.

c) Si el tablón se aparta una distancia x de la posición de equilibrio, como se indica en la figura 4, las reacciones normales dejan de ser iguales, y por consiguiente las fuerzas de rozamiento. La fuerza neta horizontal que actúa sobre el tablero es

( )2121 NNFFF rrx −=−= μ (1)

Para determinar esta fuerza es necesario conocer el valor de las fuerzas normales. En primer lugar,

MgNN =+ 21 (2)

Por otra parte, como el tablón solo se desplaza horizontalmente, el momento de las fuerzas exteriores tiene que ser cero, respecto a cualquier punto. En particular, respecto al centro de masas del tablón, se puede escribir

⎟⎠⎞

⎜⎝⎛ −=⎟

⎠⎞

⎜⎝⎛ + xbNxbN

22 21 (3)

De (2) y (3) se deduce que

⎟⎠⎞

⎜⎝⎛ −=

bxMgN

21

1 ⎟⎠⎞

⎜⎝⎛ +=

bxMgN

21

2

Y, por tanto, la fuerza horizontal (1) sobre el tablón es

xb

gMFxμ2

−=

y la aceleración con que se mueve viene dada por

xb

ga μ2−= (4)

x

Y

X

N1 N2

Mg

Fr1 Fr2

Fig. 3

L

Fig. 4

x

Y

Xb

N1 N2

Mg

Fr1 Fr2

Page 32: XXIII olimpiadas españolas de física, enunciados y soluciones de las pruebas propuestas - pag 214.pdf

XXII OLIMPIADA ESPAÑOLA DE FÍSICA

Página 9 de 22

A la vista de la expresión (4) se deduce inmediatamente que el movimiento del tablón es oscilatorio armónico, independiente de la masa del tablón y con una pulsación y un periodo dados por

b

gμω 2=

gbTμ

π2

2=

d) Dado que el tablón realiza un movimiento oscilatorio armónico, su elongación y su velocidad son

( )ϕω += tAtx sen)( (5a)

( )ϕωω += tAtv cos)( (5b)

siendo A la amplitud y ϕ la fase en 0=t , cuyos valores dependen de las condiciones iniciales del movimiento.

De acuerdo con el enunciado, el tablón está inicialmente en reposo (posición de equilibrio) y se pone en movimiento mediante un impulso horizontal I . Como el impulso es igual a la variación del momento lineal del sistema, se tiene

0vMI =

donde 0v es la velocidad con que comienza a desplazarse el tablón.

Por lo tanto, las condiciones iniciales son

( )( ) 00

00

vtv

tx

==

==

Aplicando estas condiciones en (5a) y (5b), se deduce que

ϕsen0 A= y ϕω cos0 Av = ⇒ 0=ϕ y Av ω=0 (6)

Si la longitud del tablón es L , para que al oscilar permanezca siempre apoyado sobre los dos rodillos la máxima amplitud de sus oscilaciones, tiene que cumplir la relación (figura 5)

LbA =+max2 ⇒ 2max

bLA −= (7)

En consecuencia, la velocidad inicial máxima es

20

bLv −= ω

Finalmente, el impulso máximo que se podrá aplicar es

2max

bLMI −= ω

y teniendo en cuenta el valor de ω queda

( )b

gbLMI μ221

max −=

Fig. 5

b

maxA maxA L

Page 33: XXIII olimpiadas españolas de física, enunciados y soluciones de las pruebas propuestas - pag 214.pdf

XXII OLIMPIADA ESPAÑOLA DE FÍSICA Página 10 de 22

Problema 3. El experimento de Millikan.

En el año 1909 Robert Millikan y Harvey Fletcher diseñaron y realizaron el primer experimento para medir la carga del electrón. Hasta entonces los electrones sólo habían podido ser observados en forma de rayos catódicos, pero con ellos sólo se podía determinar la relación entre su carga y su masa. Con este experimento, Millikan logró medir el valor de la carga y, por tanto, también el de la masa. Para ello supuso (por aquel entonces no estaba verificado) que la carga del electrón era la fundamental y, en consecuencia, la carga de cualquier cuerpo sería un múltiplo de dicha cantidad.

El equipo básico para realizar el experimento de Millikan está representado esquemáticamente en la figura 1. En una cámara cerrada se pulverizan pequeñas gotas de un aceite especial, de densidad ρ . Algunas de estas gotas se electrizan levemente en el momento de ser pulverizadas. Lógicamente, cada gota empieza a caer verticalmente por la acción de la gravedad, pero a su vez el aire ejerce sobre ellas una fuerza de resistencia que se opone a su movimiento y cuyo módulo es proporcional a la velocidad de caída. Esta fuerza, para un pequeño cuerpo esférico, satisface la ley de Stokes

vrFr ηπ6=

Donde η es la viscosidad del fluido (aire) en el que caen las gotas de aceite, r es el radio de las gotas y v su velocidad de caída.

Como las gotas se cargan ligeramente al capturar iones presentes en el aire, o simplemente por fricción con la boquilla del pulverizador, con este experimento se comprobó que las cargas son un múltiplo de la carga elemental e, y pudo determinarse su valor.

Dado que la densidad del aire es mucho menor que la del aceite, en este problema no se tendrá en cuenta el empuje hidrostático sobre las gotas (principio de Arquímedes).

a) Escribe la ecuación del movimiento de una gota de aceite, de masa m , que cae en el aire bajo la acción de la gravedad, g.

b) Transcurrido un corto intervalo de tiempo a partir del instante en el que la gota comienza su caída en el aire, su velocidad tiende a un valor constante, que se denomina velocidad límite, Lv . Determina dicha velocidad en función de la aceleración de la gravedad g , de la densidad ρ del aceite, de la viscosidad del aire η y del radio r de la gota.

c) Dibuja cualitativamente la grafica de la velocidad de caída de la gota en función del tiempo, suponiendo que parte del reposo.

La velocidad límite se puede medir por observación directa de la caída de la gota con un microscopio que dispone de una escala graduada. Sin embargo, no es fácil medir el radio y masa de las gotas.

V

Spray de aceite

Microscopio

Gotitas de aceite aceite

Campo eléctrico uniforme

Fig. 1

S

E

Page 34: XXIII olimpiadas españolas de física, enunciados y soluciones de las pruebas propuestas - pag 214.pdf

XXII OLIMPIADA ESPAÑOLA DE FÍSICA

Página 11 de 22

En una experiencia de laboratorio en el que se utiliza un montaje como el descrito antes, se observa que, en ausencia de campo eléctrico ( 0=E ), una determinada una gota cae con una velocidad límite

m/s1020,1 4−⋅=Lv . Sabiendo que la viscosidad del aire es sPa1080,1 5 ⋅⋅= −η , la densidad del aceite es 32 kg/m10998 ⋅= ,ρ y que la aceleración de la gravedad es 2m/s819,g =

d) Determina la expresión de la masa m de la gota y calcula su valor.

Cerrando el interruptor S, se establece un campo eléctrico uniforme E como se muestra en la figura 1, cuyo valor se puede regular cambiando el potencial V. Ajustando este campo eléctrico se puede parar la gota, es decir, mantenerla en reposo.

e) Si el campo que mantiene a la gota en reposo es V/m10359 4⋅= ,E , calcula el valor de esta carga q .

En la misma experiencia y siguiendo el mismo procedimiento, se determinaron las cargas de otras gotas. Los valores que se obtuvieron son C10544 19−⋅=′ ,q y C10053 19−⋅=′′ ,q . A partir de los valores de qqq ′′′ y,

f) Calcula el valor e de la carga fundamental del electrón.

Nota: En el experimento original, Robert Millikan y Harvey Fletcher obtuvieron un valor ligeramente menor al conocido

actualmente para la carga del electrón.

Page 35: XXIII olimpiadas españolas de física, enunciados y soluciones de las pruebas propuestas - pag 214.pdf

XXII OLIMPIADA ESPAÑOLA DE FÍSICA Página 12 de 22

Solución Problema 3. El experimento de Millikan

a) Las fuerzas que actúan sobre la gota son las representadas en la figura 2. En virtud de la 2ª ley de Newton la ecuación del movimiento es:

rFgmam −= ⇒ vrgmam πη6−= (1)

b) De acuerdo con (1), conforme aumenta el módulo de la velocidad de caída de la gota, la aceleración disminuye. A partir del valor de la velocidad que anula la aceleración, el movimiento será uniforme y dicha velocidad será la llamada velocidad límite Lv

0=a ⇒ r

gmvL πη6= (2)

Por otra parte, la masa de la gota en función de la densidad del aceite es

ρπ 3

34 rm = (3)

Eliminando r entre (2) y (3), se obtiene como velocidad límite

2

92 rgvL η

ρ=

c) Tal como sugiere el enunciado, consideraremos que en el instante inicial la velocidad vertical de la gota es nula, por lo que la gráfica v(t) pasa por el origen de coordenadas. Además, al eliminar en (1) el término dependiente de la velocidad en el instante inicial, se deduce que la pendiente en el origen es

g=ϕtg

Además, con el transcurso del tiempo la velocidad tiende al valor límite, por lo que Lvv = es una asíntota. Con todo ello, el aspecto de la gráfica es el representado en la figura 3.

d) Eliminando r entre las expresiones (2) y (3) y despejando la masa, se obtiene

3

332162g

vm L

ρ

ηπ= ⇒ kg10364 15−⋅= ,m

e) Cuando la gota se encuentra en reposo por la acción del campo eléctrico E , se verificará que qEmg = . Por lo tanto

4

15

1035981910364

⋅⋅⋅

=−

,,,q ⇒ C10574 19−⋅= ,q

f) Sabemos que las cargas qq,q ′′′ y deben ser múltiplos de una cantidad e menor, por lo que dividiendo la carga de las tres gotas por la menor de ellas podemos hallar proporciones simples entre ellas

Fig. 3

t

v vL

ϕ

Fr

mg

Fig. 2

Page 36: XXIII olimpiadas españolas de física, enunciados y soluciones de las pruebas propuestas - pag 214.pdf

XXII OLIMPIADA ESPAÑOLA DE FÍSICA

Página 13 de 22

23501 =≈

′′,

qq ⇒ qq ′′≈

23

23491 ≈≈

′′′ ,

qq ⇒ qq ′′≈′

23

1=′′′′

qq

Las cargas no pueden ser fracciones de la carga elemental, por lo que concluimos que las cargas de cada gota deben ser:

⎪⎪⎭

⎪⎪⎬

=′′

=′

=

eq

eq

eq

2

3

3

⇒ C10521 19−⋅≈ ,e

Page 37: XXIII olimpiadas españolas de física, enunciados y soluciones de las pruebas propuestas - pag 214.pdf

XXII OLIMPIADA ESPAÑOLA DE FÍSICA Página 14 de 22

CBr

I

R

Fig. 2

C

I

R R

N/2 esp. N/2 esp.

Prueba experimental. Bobinas de Helmholtz

Objetivos.

Se van a construir un par de bobinas en montaje de Helmholtz y se va a estudiar experimentalmente el campo magnético que crean en su zona central, CB , en función de la corriente que circula por ellas, I. Además de la dependencia )(IBC , se obtendrá la componente horizontal del campo magnético terrestre. Materiales.

- Hilo esmaltado de cobre (6,5 m) con conectores ("bananas") en sus extremos. - Dos cilindros de metacrilato transparente con cinta adhesiva de doble cara en su superficie exterior. - Cinta adhesiva y masilla ("blu-tack"). - Cartulina. - Regla. - Dos imanes cilíndricos iguales, e hilo de algodón para colgarlos. - Cilindro y barra de PVC. - Cables de conexión eléctrica. - Potenciómetro. - Pila de petaca. - Polímetro. - Cronómetro. Modelo teórico.

El campo magnético en el centro O de una bobina de N espiras circulares de radio R, delgadas y apretadas, por las que circula una corriente I es

R

INB

20

= (1)

donde 270 AN104 −−⋅= πμ es la permeabilidad del vacío. La dirección y el

sentido de este campo se indican en la figura 1, donde por simplicidad se ha dibujado una única espira. Este campo no es uniforme, sino que decrece rápidamente a lo largo del eje de simetría de la espira (OY en la figura 1).

En muchas ocasiones interesa disponer de un campo magnético uniforme en una zona del espacio. Uno de los montajes más empleados para conseguirlo son las llamadas bobinas de Helmholtz: se hace circular la misma corriente y en el mismo sentido por dos bobinas iguales y coaxiales situadas en planos paralelos, separados una distancia igual al radio de las espiras (figura 2; de nuevo se ha representado una única espira en cada bobina). Puede demostrarse que con esta configuración geométrica el campo magnético en torno al centro geométrico C del sistema es muy uniforme, con variaciones inferiores al 1% dentro de una región con dimensiones del orden de R/2.

Lógicamente, el campo magnético en C seguirá siendo directamente proporcional al número total N de espiras de las dos bobinas (N/2 en cada una) y a la corriente I que circula por ellas. Pero la distancia de C a los conductores es mayor que el radio de las espiras, por lo que es de esperar que CB sea inferior al que se tiene en el centro de una única bobina de N espiras, es decir

OC BKB = (2)

donde K es una constante menor que la unidad. El primer objetivo de esta prueba experimental es determinar el valor de esta constante.

Y OBr

I

R

Fig. 1

X

Z

O

N espiras

Page 38: XXIII olimpiadas españolas de física, enunciados y soluciones de las pruebas propuestas - pag 214.pdf

XXII OLIMPIADA ESPAÑOLA DE FÍSICA

Página 15 de 22

Para medir el campo magnético se va a emplear una brújula formada por los dos imanes cilíndricos, unidos longitudinalmente y colgados mediante un hilo. En equilibrio, el eje del cilindro se orienta en la dirección del campo magnético, y el periodo T de pequeñas oscilaciones torsionales de la brújula (en el sentido de retorcer el hilo) en torno a dicha orientación de equilibrio depende del módulo del campo, B, en la forma2

BT

α=21 (3)

donde α es una constante que depende de la "potencia" del imán (de su momento magnético) y de la masa y dimensiones del cilindro (de su momento de inercia). Para nuestra brújula

( ) 214 sT101004 −−⋅±= ,,α (4)

Las bobinas de Helmholtz se orientarán con su eje en la dirección del campo magnético terrestre HB (componente horizontal), y la brújula se colocará en el centro de las bobinas, de forma que estará sometida a un campo total aproximadamente uniforme

IR

NKBBBB HCH 2

0μ+=+= (5)

Consideraremos el campo HB como positivo. Pero nótese que el campo CB de las bobinas puede ser positivo o negativo según sea el signo de I, es decir el sentido de la corriente que circula por ellas.

El valor local de HB depende notablemente de la proximidad de estructuras de hierro, del edificio o de la propia mesa de trabajo, y será la segunda incógnita del problema.

Montaje experimental.

1) Construcción de las bobinas de Helmholtz (figura 3).

* Retira el papel protector de la cinta adhesiva de doble cara que hay pegada en la superficie exterior de cada cilindro de metacrilato. Esta superficie adhesiva va a facilitar el bobinado de las espiras.

* Dejando libres unos 30 cm de hilo de cobre, con su banana, comienza a bobinar 10 vueltas de hilo sobre la cinta adhesiva de un cilindro. Para que no se despegue el punto inicial del bobinado, emplea un trozo de cinta adhesiva normal. Procura que las sucesivas espiras sean bien circulares y estén bien apretadas3 unas a otras. Sugerencia: tras dar cada vuelta de hilo, empuja lateralmente con las uñas para apretarla contra las anteriores.

* Asegura el punto final de las espiras con cinta adhesiva. Comprueba que has contado bien el número de espiras, N/2 = 10.

* Deja libres unos 20 cm de hilo entre las dos bobinas y repite el proceso de bobinado de 10 espiras sobre el segundo cilindro de metacrilato.

* Si has realizado correctamente el proceso anterior, deberían quedar libres al final unos 30 cm de hilo de cobre, con su banana de conexión.

* Traza una línea longitudinal en la cartulina y dos líneas perpendiculares a la anterior separadas entre sí una distancia igual al radio de las espiras. Estas líneas te serán útiles como referencia visual para situar a continuación las bobinas en montaje de Helmholtz.

2 Supuesto despreciable el momento de torsión del hilo del que cuelga la brújula. 3 El hilo de cobre está esmaltado con un material aislante, de forma que no hay contacto eléctrico entre espiras adjuntas.

Fig. 3

Page 39: XXIII olimpiadas españolas de física, enunciados y soluciones de las pruebas propuestas - pag 214.pdf

XXII OLIMPIADA ESPAÑOLA DE FÍSICA Página 16 de 22

* Pega un trocito de masilla sobre cada cilindro, en la zona de principio y fin del bobinado, y pega los cilindros sobre las líneas transversales de la cartulina, con sus ejes de simetría sobre la línea longitudinal dibujada y los hilos de cobre sobre la cartulina, como se muestra en la figura 3. Las dos bobinas deben ser coaxiales, estar en planos verticales separados una distancia igual al radio de las espiras (entre los planos centrales de los dos bobinados) y la corriente debe circular en el mismo sentido por ambas.

* Como la masilla es plástica, es fácil realizar pequeños retoques de orientación o distancia entre las bobinas presionando en el sentido adecuado.

* Por último, dobla y sujeta a la cartulina con cinta adhesiva todos los hilos de cobre juntos, de forma que sean aproximadamente paralelos al eje del sistema. Como es fácil comprender, de esta forma se anula el campo magnético que produce la corriente que circula por ellos y que podría afectar a las medidas.

* Atención: maneja con cuidado el hilo de cobre porque es bastante frágil. En particular, evita las curvaturas fuertes y los nudos, que pueden provocar que se rompa, con la consiguiente pérdida de tiempo por tener que empezar de nuevo. Por otra parte, ten también cuidado de no cortarte con el hilo.

2) Construcción de la brújula (figura 4).

* Une longitudinalmente los dos imanes y átalos por su centro con una lazada de hilo de algodón (también puedes aprisionar el hilo entre los dos imanes).

* Cuelga la brújula del soporte construido con el tubo y la barra de PVC. Con los imanes cerca de la mesa, sujeta el hilo a la barra con cinta adhesiva.

* Se puede ajustar la altura de la brújula girando la barra de PVC para enrollar o desenrollar hilo, y la posición horizontal desplazando la barra en el tubo.

* Es inevitable que el imán quede en equilibrio un poco inclinado respecto a la horizontal, ya que el campo local también tiene componente vertical.

3) Disposición final y conexiones eléctricas (figura 5).

* Procura situar el montaje alejado de elementos de hierro de la mesa (patas, travesaños ...).

* Sitúa la brújula en la zona central de las bobinas. La brújula marca, en equilibrio, la dirección de la componente horizontal del campo magnético local, HB . Orienta las bobinas de forma que su eje de simetría coincida con esta dirección y el centro de la brújula esté en el centro de las bobinas.

* Una vez conseguida la posición adecuada del soporte de la brújula y de las bobinas, sujeta el sistema a la cartulina y a la mesa con cinta adhesiva para evitar posteriores movimientos involuntarios.

* Las bobinas se alimentan con una pila de petaca a través de un potenciómetro (resistencia variable) que permite ajustar la tensión aplicada a las bobinas4.

* Conecta la pila y las bobinas al potenciómetro con los cables suministrados, siguiendo el esquema de la figura 6, donde los puntos negros indican los puntos de conexión con bananas.

* Atención: para conseguir un buen contacto eléctrico, las lengüetas terminales de la pila deben introducirse entre el metal y el plástico de los conectores (no dentro del terminal metálico, que es demasiado ancho).

4 Como puedes observar, el potenciómetro lleva montada una resistencia de protección para evitar corrientes elevadas que

agotarían rápidamente la pila.

Fig. 4

Fig. 5

Page 40: XXIII olimpiadas españolas de física, enunciados y soluciones de las pruebas propuestas - pag 214.pdf

XXII OLIMPIADA ESPAÑOLA DE FÍSICA

Página 17 de 22

* La corriente I se mide con el polímetro actuando como amperímetro, conectado en serie en el circuito. Emplea la escala de 200 mA.

* Atención: para evitar que se agote la pila, mantén el circuito abierto cuando no estés midiendo.

* Antes de empezar a medir, familiarízate con el manejo del potenciómetro, el polímetro y el cronómetro.

* Inicialmente se va a medir con CB positivo, es decir en el mismo sentido que HB . Teniendo en cuenta (3) y (5), esto implica que el periodo de oscilación torsional de la brújula, T, debe ser decreciente con I (es decir, a mayor I menor T ). Si en tu sistema ocurre lo contrario, invierte el sentido de la corriente, intercambiando las conexiones de la pila al potenciómetro.

* La situación anterior, según nuestro convenio de signos, corresponde a I > 0. Si el amperímetro indica una corriente negativa, intercambia los puntos de conexión en el polímetro.

* Posteriormente se realizarán medidas con I < 0, para lo que tendrás que invertir el sentido de la corriente.

* Si te encuentras con que T no depende apenas de I, hay un problema de montaje en tu circuito: o bien la corriente no está circulando por las bobinas (revisa las conexiones), o lo hace en sentidos opuestos.

* Después de hacer las medidas, deja tu montaje experimental sobre la mesa, tal y como lo has utilizado para medir, ya que el equipo de correctores lo valorará durante la prueba. Al final, después de entregar el ejercicio, desmonta el sistema y recoge sus diversos elementos en las cajas correspondientes.

Medidas y preguntas.

a) Mide el periodo T de pequeñas oscilaciones torsionales de la brújula en torno a su orientación de equilibrio para un mínimo de diez valores de I, entre 0=I e mA200=I . Explica detalladamente tu método de medida y presenta los resultados en la tabla de la hoja de respuestas. Tabula también los valores de 21 T/ , que necesitarás más adelante.

b) Invierte el sentido de la corriente, y haz varias medidas de T entre 0=I e mA80−=I . Presenta los resultados en la tabla de la hoja de respuestas.

c) Representa gráficamente en un papel milimetrado todos los puntos ( ) )/,(, 21 TIyx = .

d) Determina la pendiente, p, y la ordenada en el origen, 0y , de la recta que mejor se ajusta a estos puntos.

e) Deduce los valores de la constante K de las bobinas de Helmholtz y del campo magnético local HB .

f) Haz una estimación razonada de la incertidumbre pΔ de la pendiente obtenida en el apartado d).

g) Teniendo en cuenta lo anterior y la incertidumbre de la constante α dada en (4), haz una estimación de la incertidumbre KΔ de la constante de las bobinas que has obtenido en e).

h) En el apartado b) has medido T hasta mA80−=I . Si has probado a seguir aumentando la corriente en este sentido negativo, habrás observado que al principio la brújula oscila muy lentamente, que para una corriente suficientemente alta "pierde el rumbo", es decir da vueltas o oscila en torno a una dirección diferente, y que, para corrientes más altas, la brújula se orienta en sentido opuesto al inicial. ¿Podrías explicar este comportamiento? Teniendo en cuenta tus resultados del apartado e), ¿a partir de qué corriente

0I se espera teóricamente la inversión de sentido de la brújula? Mide experimentalmente 0I y comprueba si coinciden aproximadamente.

Pila A R protec.

Amperímetro

Potenciómetro Fig. 6

Bobinas

I

Page 41: XXIII olimpiadas españolas de física, enunciados y soluciones de las pruebas propuestas - pag 214.pdf

XXII OLIMPIADA ESPAÑOLA DE FÍSICA Página 18 de 22

Solución prueba experimental. Bobinas de Helmholtz

a) El periodo de oscilación puede medirse con buena precisión cronometrando el tiempo de, por ejemplo, diez oscilaciones completas de la brújula, tras darle un pequeño impulso torsional. La incertidumbre de esta medida puede estimarse en 0,1 s, de forma que la incertidumbre del periodo T es del orden de 0,01 s. Esta incertidumbre puede reducirse aumentando el número de oscilaciones de la medida o realizando varias medidas y promediando.

En la siguiente tabla se recogen los valores de T y 21 T/ , que se necesitarán más adelante, medidos a intervalos aproximadamente regulares de 20 mA entre I = -80 mA e I = 200 mA. Los valores presentados son el promedio de cuatro medidas de diez oscilaciones.

I (mA) -80,5 -59,8 -40,1 -20,4 0,0 20,1 40,1 60,2 80,2 100,1 120,2 138,8 160,3 180,3 197,4

T (s) 1,703 1,415 1,238 1,107 1,025 0,945 0,887 0,822 0,785 0,750 0,722 0,690 0,666 0,644 0,625

1/T 2 (s-2) 0,345 0,499 0,653 0,816 0,952 1,120 1,271 1,480 1,623 1,778 1,918 2,100 2,255 2,411 2,560

b) Las medidas con intensidades negativas ya se han presentado en la tabla anterior.

c) A continuación se presenta la gráfica pedida, con el aspecto que tendría dibujada en papel milimetrado.

0,0

0,5

2,0

1,0

1,5

-0,10 0,05 0,10 0,15 0,20

( )22 −− sT

2,5

0,0 I (A)

-0,05

A

B

Page 42: XXIII olimpiadas españolas de física, enunciados y soluciones de las pruebas propuestas - pag 214.pdf

XXII OLIMPIADA ESPAÑOLA DE FÍSICA

Página 19 de 22

d) En la gráfica anterior también se ha trazado la recta que mejor se ajusta a los puntos experimentales. La

pendiente, p, y la ordenada en el origen, 0y , de esta recta pueden determinarse a partir de las coordenadas de dos puntos de dicha recta. Para mejorar la precisión del resultado interesa tomar dos puntos alejados, por ejemplo los puntos A y B indicados en la gráfica, elegidos cerca de los extremos de la recta y coincidentes con cruces en la cuadrícula, para facilitar la lectura precisa de sus coordenadas.

( ) ( )-2s0750A11250 ,;,y;x AA −=

( ) ( )-2s6752A21250 ,;,y;x BB =

AB

AB

xx

yyp

−= → 1-2As008 −= ,p

La ordenada en el origen de la recta puede deducirse de las coordenadas de uno de los puntos auxiliares (o también puede leerse directamente en la gráfica)

0ypxy AA += → AA pxyy −=0 → -20 s9750,=y

Nota: un ajuste analítico por el método de mínimos cuadrados conduce a un resultado muy similar:

1-2As0018 −= ,p , -20 s97360,=y

e) De acuerdo con las expresiones (3) y (5) del enunciado, se espera que la dependencia de 21 T/ con I sea lineal, con pendiente y ordenada en el origen

R

NKp

20μ

α= → N

pRK

αμ0

2=

HBy α=0 → α

0yBH =

Con los datos del enunciado y el valor del radio de las espiras5, R = 4,5 cm, se obtiene

7160,=K

T10442 5−⋅= ,HB

f) Para hacer una estimación de la incertidumbre de la pendiente vamos a trazar las rectas que, con pendientes máxima y mínima, se ajustan razonablemente a los puntos experimentales. Para ello hay que tener en cuenta la dispersión de los puntos respecto a la recta de mejor ajuste, que es nuestro caso es en promedio inferior al radio de los puntos dibujados. También es necesario tener en cuenta la incertidumbre estimada para los propios puntos experimentales. No vamos a tener en cuenta los posibles errores por falta de calibración del amperímetro, ya que no tenemos datos al respecto, pero sí podemos hacer una estimación de la incertidumbre de los valores de 21 T/ obtenidos tras cronometrar el periodo de oscilación. Con el método de medida empleado (promedio de cuatro medidas de diez oscilaciones) una estimación razonable para la incertidumbre del periodo es

s0050,=ΔT

La incertidumbre de 21 T/ puede calcularse numéricamente para cada punto a partir de los valores de TTT Δ+=max y TTT Δ−=min o, de una forma más elegante, tomando incrementos (en valor absoluto)

32 21TT

TΔ=⎟

⎠⎞

⎜⎝⎛Δ

5 Este es el radio del cilindro de metacrilato, que puede medirse con la regla con una incertidumbre del orden de 0,5 mm.

El radio medio real de las espiras es algo superior debido al grosor de la cinta adhesiva y al diámetro del cable de cobre, que es de 0,3 mm. Esta pequeña fuente de error puede despreciarse frente a la incertidumbre de la constante α.

Page 43: XXIII olimpiadas españolas de física, enunciados y soluciones de las pruebas propuestas - pag 214.pdf

XXII OLIMPIADA ESPAÑOLA DE FÍSICA Página 20 de 22

De una forma u otra es fácil comprobar que la incertidumbre de los primeros puntos, los correspondientes a corrientes negativas, es muy pequeña, inferior al tamaño de los puntos dibujados. Para corriente creciente la incertidumbre va aumentando y, por ejemplo, en el punto de corriente más alta alcanza el valor máximo

2

mA1972 s0401 −

==⎟

⎠⎞

⎜⎝⎛Δ ,

IT

En total, la "barra de error" del último punto no alcanza dos cuadritos a cada lado en la escala de la gráfica dibujada. Teniendo esto en cuenta, y la ya citada escasa dispersión de los puntos experimentales respecto a la recta óptima, es razonable considerar las rectas de pendientes máxima y mínima que a continuación se presentan, construidas manteniendo fijo el punto auxiliar A y con desviaciones de 2s0250,± (un cuadrito) en la coordenada By del segundo punto auxiliar

Con este criterio, es inmediato obtener los valores máximo y mínimo estimados para la pendiente, y su incertidumbre6

→−

=Δ⎪⎭

⎪⎬⎫

=

=−

2As927

As08812-

1-2minmax

min

max

,

, ppp

p

p 1-2As080 −=Δ ,p

g) En el apartado e) se ha obtenido la constante K aplicando la expresión

N

pRK

αμ0

2=

Para calcular la incertidumbre de K es necesario "propagar" las incertidumbres de la pendiente p y de la constante α.

6 Un cálculo (no ponderado) da como resultado una incertidumbre Δp = 0,1 s-2 A-1 con un nivel de confianza del 95%.

0,0

0,5

2,0

1,0

1,5

-0,10 0,05 0,10 0,15 0,20

( )22 −− sT

2,5

0,0 I (A)

-0,05

A

B

Page 44: XXIII olimpiadas españolas de física, enunciados y soluciones de las pruebas propuestas - pag 214.pdf

XXII OLIMPIADA ESPAÑOLA DE FÍSICA

Página 21 de 22

Un método sencillo y rápido, aunque no muy exacto, de calcular los valores máximo y mínimo de K consiste en ponerse en el "peor de los casos", es decir en combinar el valor máximo de p en el numerador con el mínimo de α en el denominador, y viceversa

02506920

2

74202

0

0 ,

,

,

max

minmin

min

maxmax

=Δ→

⎪⎪⎭

⎪⎪⎬

==

==K

N

RpK

N

RpK

αμ

αμ

Como las dos fuentes de error consideradas son independientes, es más razonable, aunque también algo más laborioso, calcular independientemente sus influencias en la incertidumbre de K

( ) ( ) 00702221

21

00,minmaxminmax =Δ=Δ=−=−=Δ

pp

KpN

RppN

RKKK pp αμαμ

( ) 0180211221

21

200

,maxmin

minmax =Δ=Δ≈⎟⎠⎞

⎜⎝⎛ −=−=Δ

αα

αα

μααμαα KNRp

NpR

KKK

Una estimación razonable de la incertidumbre total de K sería la suma de estos dos contribuciones

250,=Δ+Δ=Δ αKKK p

Pero, teniendo de nuevo en cuenta que las dos fuentes de error son independientes, es más correcto calcularla en la forma

→=Δ+Δ=Δ 019022 ,αKKK p 020,≈ΔK

El resultado final del experimento sería

020720 ,, ±=K

Nota 1: el valor de αKΔ es bastante mayor que el de pKΔ . Por ello, al redondear como es habitual a una única cifra significativa el resultado final, desaparece prácticamente la influencia de pΔ En otras palabras, si se quisiese mejorar la precisión del valor de K, sería prioritario conocer α con menos incertidumbre.

Nota 2: el valor teórico de la constante K, supuesto que el cable conductor es muy delgado, es

715505

823 ,

/==K

h) La brújula se orienta en la dirección y sentido del campo magnético horizontal. En nuestro caso, cuando la corriente I es positiva, el campo de las bobinas CB lleva el mismo sentido que el terrestre HB , pero cuando la corriente I es negativa CB lleva sentido opuesto a HB . Mientras CH BB > , el campo total sigue llevando el sentido de HB (esta es la región de corrientes negativas en la que se ha medido). Pero para una corriente negativa suficientemente intensa CB superará a HB y el campo resultante tendrá sentido inverso, por lo que la brújula girará 180º y se orientará, en equilibrio, en sentido opuesto al inicial.

Cuando el campo total es muy débil, el movimiento de la brújula está influido apreciablemente por el momento de torsión del hilo del que cuelga. Por ejemplo, si el hilo está inicialmente retorcido, la brújula empieza a girar y parece volverse loca.

El punto de inversión corresponde obviamente a campo total nulo. Teniendo en cuenta la expresión (5) del enunciado, esto ocurre para una corriente 0I que cumpla

NK

RBII

R

NKBB H

H0

000 2

02 μ

μ−=↔=+= A12200 ,−=I

Este valor también puede leerse directamente en la gráfica de 21 T/ frente a I, en el punto de corte de la recta de ajuste con el eje de abscisas, pues según (3) 0=B implica 01 2 =T/ .

Page 45: XXIII olimpiadas españolas de física, enunciados y soluciones de las pruebas propuestas - pag 214.pdf

PATROCINADORES:

COLABORADORES:

Page 46: XXIII olimpiadas españolas de física, enunciados y soluciones de las pruebas propuestas - pag 214.pdf

REAL SOCIEDAD ESPAÑOLA DE FÍSICA

Problema Experimental

Page 47: XXIII olimpiadas españolas de física, enunciados y soluciones de las pruebas propuestas - pag 214.pdf

Prueba experimental.

Plano inclinado1

Objetivos.

Se va a estudiar experimentalmente el descenso de una esfera por un plano inclinado de pendiente variable. En concreto se va a determinar el factor geométrico que diferencia la aceleración de descenso en este experimento de la que tendría un cuerpo que deslizase sin fricción por un plano inclinado. Materiales.

- Carril de aluminio con perfil en U. - Listón de madera con manguito roscado embutido y cinta adhesiva de doble cara. - Topes de plástico y aluminio. - Tornillo con cabeza en forma de gancho. - Esfera de acero. - Cinta métrica. - Cronómetro. Montaje; procedimiento experimental.

• Quita el papel que recubre la cinta adhesiva de doble cara pegada al listón de madera, y pega en su centro el carril de aluminio, como se indica en la figura 1. Ten cuidado de no doblar ni mellar el carril, ya que cualquier imperfección influiría en los resultados de la prueba.

• Enrosca el tornillo en el manguito embutido en el listón, con el gancho del mismo lado que el carril.

• Coloca el tope de aluminio en la acanaladura del carril más próxima al tornillo, y el de plástico en la del otro extremo. Estos topes sirven para marcar el comienzo y el final de la carrera de la esfera.

• Sitúa el montaje en una zona de la mesa que te permita trabajar y tomar notas con comodidad. La cinta adhesiva de doble cara de la parte inferior del listón permite fijar el dispositivo a la mesa, para evitar que se desplace mientras realizas las medidas experimentales. Ten en cuenta que tu mesa de trabajo puede no ser horizontal, de forma que un cambio de posición puede causar un cambio de desnivel.

• Procura mantener limpios el carril y la esfera, para evitar irregularidades y fuerzas de adherencia.

• Conviene que inicialmente la cabeza del tornillo esté orientada en paralelo al carril, como se muestra en la figura 1. Recuerda hacia donde está inicialmente orientado el lado abierto del gancho, porque tendrás que modificar el desnivel del carril girando un número entero de vueltas el tornillo.

• Vas a medir el tiempo de descenso de la esfera por el carril para diferentes desniveles. Antes de realizar medidas definitivas es conveniente que adquieras práctica con el manejo del cronómetro.

• Ten en cuenta que la esfera debe partir con velocidad inicial nula. Por tanto, no debes empujarla cuando la sueltes, ni presionarla contra el tope de partida, que podría darle un impulso inicial.

• Para determinar el tiempo de descenso con cada desnivel, es conveniente que realices un mínimo de cinco medidas y tomes como resultado el valor medio.

1 Esta prueba experimental está inspirada en la propuesta en la Olimpiada Argentina de Física de 2003.

Fig. 1

Page 48: XXIII olimpiadas españolas de física, enunciados y soluciones de las pruebas propuestas - pag 214.pdf

Modelo teórico.

En un experimento idealizado, supongamos un cuerpo que desciende deslizando sin fricción por un plano inclinado de ángulo α respecto a la horizontal (figura 2). La aceleración con que desciende el cuerpo es

Lhgga == αsen (1)

La aceleración puede determinarse experimentalmente midiendo el tiempo t que tarda el cuerpo en recorrer una cierta distancia s, partiendo del reposo. Como el movimiento es uniformemente acelerado

221 tas =

O sea

2

2t

sa = (2)

El experimento podría plantearse como una práctica de laboratorio para determinar la aceleración de la gravedad, g, con el siguiente método:

• Para un valor de s fijo y conocido, se cronometran los tiempo de descenso por el plano para diversos valores de h. Aplicando (2) se calcula la aceleración en cada caso.

• Para que los tiempos de descenso sean altos y puedan cronometrarse manualmente con buena precisión relativa, interesa que h sea pequeña, por lo que esta altura es difícil de medir con precisión. En nuestro dispositivo experimental no es necesario conocer el valor de esta altura, como se verá a continuación. Basta con realizar sucesivas medidas incrementando h en una cantidad constante, correspondiente a una vuelta del tornillo. Si la altura inicial es 0h y se gira en sentido ascendente n vueltas el tornillo, de paso de rosca d, la altura alcanzada es

dnhh n += 0 (3)

• Se representan gráficamente los puntos experimentales (x, y) = (n, a). Teniendo en cuenta (1) y (3), estos puntos deberían ajustarse a una línea recta2 con pendiente gd/L, independientemente del valor de la altura inicial 0h .

• Por tanto, supuesto que d y L son conocidas, para obtener g bastaría con determinar la pendiente de la recta que mejor se ajusta a los puntos experimentales anteriores.

En un experimento real es difícil eliminar el rozamiento entre el cuerpo y el plano inclinado. Las pérdidas energéticas por fricción podrían minimizarse mediante un colchón de aire que impidiese el contacto directo entre el cuerpo y el plano. Pero es mucho más sencillo y económico emplear una esfera rígida que desciende rodando sin deslizar. En estas circunstancias, el punto de contacto de la esfera con el plano tiene velocidad nula (no hay deslizamiento relativo), la fuerza de rozamiento no realiza trabajo y no se pierde energía por fricción.

Pero la aceleración del movimiento del centro de la esfera ya no es la dada en (1), puesto que la energía potencial gravitatoria inicial no sólo se convierte en energía cinética de traslación (movimiento del centro de la esfera) sino además en energía cinética de rotación (giro de la masa de la esfera en torno a su centro). Por ello, la aceleración del centro de la esfera se reduce en un cierto factor F > 1.

LF

hga = (4)

2 La dependencia prevista no es exactamente lineal porque L aumenta con h. Si se trabaja con ángulos α pequeños, como va

a ser nuestro caso, esta variación es despreciable y puede considerarse L aproximadamente constante.

α

s

L

h

Fig. 2

a

Page 49: XXIII olimpiadas españolas de física, enunciados y soluciones de las pruebas propuestas - pag 214.pdf

El principal objetivo de esta prueba experimental no es determinar el valor de la aceleración de la gravedad, que es bien conocido, sino el valor del factor geométrico F del dispositivo experimental empleado3, y hacer una estimación de su incertidumbre.

Medidas y preguntas.

1) Mide el recorrido de la bola, s, y la distancia entre los puntos de apoyo del plano sobre la mesa, L, (figura 3). Anota los resultados en la hoja de respuestas.

2) Girando el tornillo, ajusta la altura h hasta conseguir que el tiempo de descenso de la bola esté entre 4 y 5 s. Ésta va a ser la situación inicial de tu serie de medidas, es decir 0hh = .

Mide varias veces el tiempo de descenso de la bola, calcula su valor medio y la aceleración correspondiente. Anota tus medidas y resultados en la tabla de la hoja de respuestas.

Repite el proceso anterior incrementando h en sucesivas vueltas completas del tornillo, es decir para valores nh como los dados en (3) con n = 1, 2, 3..., hasta que el tiempo de descenso sea inferior a 2,3 s.

3) Representa gráficamente en un papel milimetrado los puntos experimentales a (en ordenadas) frente a n (en abscisas).

4) Obtén la pendiente, p, y la ordenada en el origen, c, de la recta que mejor se ajusta a estos puntos.

5) Deduce los valores del factor geométrico, F, y de la altura inicial, 0h .

6) Haz una estimación de la incertidumbre (margen de error) de la pendiente de la recta, pΔ . Calcula la incertidumbre transmitida al valor del factor geométrico, pFΔ .

7) Si la incertidumbre del paso de rosca del tornillo es mm010,=Δd , calcula la incertidumbre transmitida al valor del factor geométrico, dFΔ .

8) Teniendo únicamente en cuenta las dos fuentes de error anteriores, calcula la incertidumbre total de F.

Datos:

Aceleración de la gravedad: 2m/s819,=g

Paso de rosca del tornillo: d = 0,70 mm

3 Ayuda: 1 < F < 2.

s

h

Fig. 3

L

Page 50: XXIII olimpiadas españolas de física, enunciados y soluciones de las pruebas propuestas - pag 214.pdf

Prueba experimental. Plano inclinado Solución 1) Dimensiones de un plano inclinado típico (puede haber ligeras variaciones de un montaje a otro):

s = 35,3 cm L = 48,6 cm

2) Tabla de medidas:

n Tiempos de descenso, t (s) t (s) a (m/s2)

0 4,44 4,48 4,42 4,50 4,48 4,464 0,0354 1 3,92 3,98 3,98 3,95 3,98 3,962 0,0450 2 3,60 3,61 3,58 3,57 3,63 3,598 0,0545 3 3,29 3,25 3,29 3,30 3,30 3,286 0,0654 4 3,10 3,00 3,08 3,06 3,06 3,060 0,0754 5 2,85 2,86 2,92 2,92 2,93 2,896 0,0842 6 2,75 2,73 2,75 2,73 2,75 2,742 0,0939 7 2,57 2,61 2,60 2,61 2,62 2,602 0,1043 8 2,48 2,50 2,44 2,45 2,48 2,470 0,1157 9 2,36 2,36 2,38 2,42 2,37 2,378 0,1248

10 2,30 2,25 2,26 2,30 2,30 2,282 0,1356 11 2,23 2,24 2,18 2,24 2,19 2,216 0,1438

3) En la siguiente gráfica se representan los puntos (x, y) = (n, a) obtenidos

4) En la gráfica anterior también se ha trazado la recta que mejor se ajusta a los puntos experimentales. La pendiente, p, y la ordenada en el origen, c, de esta recta pueden determinarse a partir de las coordenadas de dos puntos alejados sobre dicha recta, por ejemplo los puntos A y B indicados en la gráfica.

( ) ( )2m/s0400500 ,;,; =AA yx

( ) ( )2m/s15005011 ,;,; =BB yx

0,02

0,04

0,06

0,08

0,10

0,12

0,14

0,16

0 2 4 6 10 12 n

a (m/s2)

A

B

8

Page 51: XXIII olimpiadas españolas de física, enunciados y soluciones de las pruebas propuestas - pag 214.pdf

AB

AB

xx

yyp

−= → 2m/s01000,=p

La ordenada en el origen de la recta puede deducirse de las coordenadas de uno de los puntos auxiliares (o también puede leerse directamente en la gráfica)

cpxy AA += → AA pxyc −= → 2m/s0350,=c

Nota: un ajuste analítico por el método de mínimos cuadrados conduce a un resultado muy similar

2m/s009960,=p , 2m/s03510,=c

5) Teniendo en cuenta las expresiones (3) y (4) del enunciado se espera una dependencia a(n) de la forma

( )ndhLF

ga += 0

La pendiente y la ordenada en el origen de esta recta son

LF

dgp = ,

LF

hgc 0=

Por tanto, los valores de F y 0h pueden obtenerse de la forma

Lp

dgF = → 411,=F

g

LFch =0 → mm520 ,=h

6) La incertidumbre de p puede estimarse trazando las dos rectas que, con pendientes máxima y mínima, se ajustan razonablemente a los puntos experimentales, teniendo en cuenta su dispersión. Como los primeros puntos están bien alineados y la dispersión es apreciablemente mayor para aceleraciones altas1, una estimación razonable para las incertidumbres de los puntos auxiliares A y B podría ser

0≈Δ Ay 2m/s00150 ,≈Δ By

1 Los errores típicos de los tiempos de descenso son del orden de 0,01 s. Al propagar esta incertidumbre a la aceleración, se

obtienen valores para Δa entre 0,0002 m/s2, para las aceleraciones más bajas, y 0,0015 m/s2 para las más altas. Estas incertidumbres son inferiores al tamaño de los puntos dibujados en la gráfica, excepto en los dos últimos, por lo que no se han representado barras de error.

0,02

0,04

0,06

0,08

0,10

0,12

0,14

0,16

0 2 4 6 10 12 n

a (m/s2)

A

B

8

Page 52: XXIII olimpiadas españolas de física, enunciados y soluciones de las pruebas propuestas - pag 214.pdf

Por tanto

010140,max,

max =−

−=

AB

AB

xx

yyp

009860,min,

min =−

−=

AB

AB

xx

yyp

2

minmax ppp

−=Δ → 2m/s000140,=Δp

Nota: Cálculos analíticos que no detallaremos aquí permiten comprobar que esta incertidumbre es razonable. En concreto corresponde a un nivel de confianza bastante alto, del orden del 85%.

La incertidumbre transmitida al factor geométrico F puede calcularse numéricamente en la forma

4331,min

max ==Lp

dgF

3931,max

min ==Lp

dgF

El resultado se ha expresado con una única cifra significativa, pues nos basamos en estimaciones de baja precisión.

Esta incertidumbre también puede obtenerse tomando incrementos, en valor absoluto, en la expresión que relaciona F con p

Lp

dgF = → 020

2,=Δ=

Δ=Δ p

pF

Lp

pdgFp

Nótese que, como es bien conocido en dependencias de tipo producto o cociente, se transmite el error relativo: ppFFp // Δ=Δ .

7) Como mm010,=Δd , se espera que el paso de rosca esté comprendido entre mm690 ,min =d y mm710,max =d . Por tanto

4331,maxmax ==

Lp

dgF

3931,minmin ==

Lp

dgF

Como en el apartado anterior, esta incertidumbre también puede calcularse tomando incrementos. F es directamente proporcional a p, y es inmediato demostrar que de nuevo se transmite el error relativo, es decir

dd

F

Fd Δ=Δ

→ 020,=Δ dF

8) La incertidumbre total del factor geométrico, teniendo únicamente en cuenta las dos fuentes de error anteriores, podría estimarse sumando las dos contribuciones

040,=Δ+Δ=Δ dp FFF

Pero, debido a que estas fuentes de error son independientes, es más correcto considerar

028022 ,=Δ+Δ=Δ dp FFF → 030,=ΔF

→ 02002

,minmax =−

=ΔFF

Fp → 020,=Δ pF

→ 02002

,minmax =−

=ΔFF

Fd → 020,=Δ dF

Page 53: XXIII olimpiadas españolas de física, enunciados y soluciones de las pruebas propuestas - pag 214.pdf

En total, el resultado principal del experimento puede expresarse en la forma

030411 ,, ±=F

Nota final: Los estudiantes que participan en la OEF no han estudiado todavía la Dinámica del sólido rígido, por lo que no saben plantear analíticamente el problema de una esfera que desciende rodando sin deslizar por un plano inclinado. Dejamos como ejercicio para estos estudiantes demostrar, cuando tengan conocimientos suficientes, que el valor esperado para el factor geométrico F en nuestro dispositivo es

22

2

521

bR

RF−

+=

donde R es el radio de la esfera y b la semianchura del carril.

En el montaje experimental empleado, R = 1,25 cm y b = 3,5 mm, con lo que se obtiene

F = 1,434

Este valor teórico queda dentro del margen de incertidumbre del resultado experimental. En todo caso, no es de extrañar que se obtenga un valor de F algo inferior al teórico, puesto que siempre existen pequeñas pérdidas energéticas. Por ejemplo, se escucha durante el descenso de la bola un claro sonido (energía sonora) debido a pequeñas irregularidades y falta de rigidez del carril de aluminio.

R

2b

Page 54: XXIII olimpiadas españolas de física, enunciados y soluciones de las pruebas propuestas - pag 214.pdf

REAL SOCIEDAD ESPAÑOLA DE FÍSICA

Problema Teórico 1

Page 55: XXIII olimpiadas españolas de física, enunciados y soluciones de las pruebas propuestas - pag 214.pdf

Problema 1. Una introducción al arte de navegar.

Alicante es una bella ciudad mediterránea que vive de cara al mar. Su magnífico puerto es un hervidero de barcos de recreo, yates espectaculares y otros más modestos. Los hay de muchos tipos y tamaños, pero de todos ellos destacan los veleros por su elegante belleza. La vocación marinera de Alicante fue recompensada cuando en 2008 fue seleccionada como puerto de salida de la más importante competición náutica: la vuelta al mundo a vela, conocida desde 2001 como la Volvo Ocean Race. La regata partió en Octubre de Alicante y, tras circunnavegar el globo, terminó en San Petersburgo en junio de 2009.

¿Cómo y por qué avanza un velero? Hasta los de tierra adentro sabemos contestar a esta pregunta: “porque el viento los empuja”. Esto es evidente cuando el viento viene por la “popa” (aunque ésta no es la condición más favorable para impulsar un velero moderno). Pero, ¿y si el viento es de costado? Esto requiere alguna discusión en la que la Física tiene la palabra y de ello va a tratar este problema.

Por supuesto, el problema está basado en un modelo físico sencillo de velero elemental con una única vela, la mayor, es decir sin vela delantera o foque. Supondremos también que el velero navega en la dirección (rumbo) este a oeste con un viento que sopla de norte a sur.

Los únicos elementos activos del velero que se van a considerar inicialmente son la vela y la orza (figura 1). La orza es una especie de vela rígida situada bajo el casco, completamente sumergida y con un gran peso que contribuye a la estabilidad del velero.

La vela y la orza actúan como el ala de un avión, salvo que sus planos son aproximadamente verticales y, en el caso de la orza, el fluido que la circunda es el agua.

En la figura 2 se muestra la planta de un velero cuya vela recibe viento con una velocidad avr relativa al barco, que es la velocidad con que se percibe el viento desde el velero (viento aparente). Dicho viento produce dos fuerzas aerodinámicas: una de sustentación S

r perpendicular a avr , y otra de resistencia, R

r, en la dirección de avr . La

componente xS de la sustentación es la responsable de hacer avanzar a la nave, mientras que la componente yS tiende a desplazarlo de costado, lo que, naturalmente, no interesa. De evitarlo se encarga la orza, situada longitudinalmente debajo del casco. Como se muestra en la figura 3, cuando el eje del velero forma un pequeño ángulo δ (abatimiento) con la dirección de movimiento del velero (rumbo), sufre también unas fuerzas, hidrodinámicas en este caso, de sustentación S ′

r perpendicular a la velocidad V

r del velero, y de resistencia R′

r

paralela a dicha velocidad y en sentido opuesto, en la que también influye el casco del buque y el propio abatimiento.

Fig. 1

Vela mayor

Timón

Orza

avr

Sr

Rr

Fig. 2

yS

xS X

Y

Vela Vr

Fig. 3

δ

Sr′

R′r

Orza

Page 56: XXIII olimpiadas españolas de física, enunciados y soluciones de las pruebas propuestas - pag 214.pdf

Una vez establecidas estas nociones elementales de náutica, el problema que se plantea es el siguiente.

a) Supongamos que un velero está navegando con un viento que sopla del Norte con una velocidad nudos.150 =v En estas condiciones navega hacia el Oeste a nudos18=V de velocidad y con un abatimiento δ (figura 4).

Determine la velocidad avr del viento respecto al velero (viento aparente). Calcule su módulo av y su dirección β . (1,5 Puntos)

b) La superficie de la vela de nuestro velero con el viento a disposición y el reglaje (trimado) escogido, produce una sustentación N1005 3⋅= ,S . La eficiencia de las actuales velas es muy elevada, de forma que la resistencia que presentan es de tan sólo un 10% de la sustentación. Por lo tanto, en nuestro caso, N1005 2⋅= ,R .

Determine y calcule el módulo de la fuerza de sustentación S ′ que tiene que generar la orza para que el velero siga manteniendo su rumbo exactamente hacia el Oeste. (1,25 Puntos)

c) El conjunto del casco en contacto con el agua (carena) y la orza son menos eficientes que la vela. En consecuencia, el “precio a pagar” es una fuerza de resistencia, R′ que puede estimarse en este caso como la cuarta parte de la sustentación S ′ .

Determine y calcule el módulo de la resistencia debida al conjunto vela y orza, RRRt ′+=rrr

, y el ángulo θ que forma tR

r con la dirección del movimiento del velero. (3 Puntos)

d) Los veleros de recreo, no destinados a competición, suelen estar provistos de un motor para facilitar las maniobras y, naturalmente, para poder navegar en ausencia de viento. Supongamos que, en estas condiciones, un velero de masa kg2500=m navega con una velocidad constante, nudos03,V =′ , con su motor desarrollando una potencia P igual al 90% de su potencia máxima, CV15=maxP .

Calcule la fuerza de resistencia, rF que el agua opone al avance del barco. (2 Puntos)

e) El patrón maniobra el timón para cambiar el rumbo, de forma que el barco describe un arco de circunferencia de radio m10=a . En estas circunstancias y considerando que el módulo de la velocidad del barco permanece constante, nudos03,V =′ , la fuerza neta que el agua hace sobre el barco, tF

r, tiene dos componentes: una, cF ,

que le obliga a describir la trayectoria circular y otra de resistencia igual a la anterior rF más 4/FF cr =′ .

Determine y calcule la fuerza cF . (1,25 Puntos)

f) Indique razonadamente si el motor del barco tiene suficiente potencia para realizar la maniobra indicada en el apartado anterior. (1 Punto)

Nota: El nudo es una unidad náutica de velocidad que equivale a 1 milla/hora.

1 milla (náutica) = 1,852 km

1 CV = 735,5 W

Vr

Fig. 4

δ Oeste

0v

Sur

avr β X (Este)

Y (Norte)

Page 57: XXIII olimpiadas españolas de física, enunciados y soluciones de las pruebas propuestas - pag 214.pdf

Solución Problema 1. Una introducción al arte de navegar.

a) Si las velocidades del velero y del viento respecto al mar, que se considera en reposo, son Vr

y 0vr respectivamente, la velocidad del viento respecto al velero avr verificará que

Vvvarrr

−= 0

De la figura 5, en la que está representada esta composición vectorial, se deduce que

0

arctgvV

=β ⇒ º50=β

y el viento soplará respecto al velero con una velocidad

220 Vvva += ⇒ nudos23=av

b) En la figura 6 se representan las fuerzas que actúan sobre el velero debidas a la vela y la orza. Para evitar que el velero altere su rumbo es necesario que la fuerza S ′

r de

sustentación de la orza compense las componentes, en la dirección Sur-Norte, de la sustentación S

r de la vela y de su

resistencia Rr

. Es decir

ββ cossen RSS +=′ ⇒ N1024 3⋅=′ ,S

c) La resistencia total del conjunto vela-orza es RRRt ′+=rrr

. En componentes,

( )( ) ⎪⎭

⎪⎬⎫

−=

+′=

β

β

cos

sen

RR

RRR

yt

xt ⇒

( ) ( )22 cossen ββ RRRRt −++′=

De acuerdo con el enunciado, N10014 3⋅=′=′ ,/SR y N1005 2⋅= ,R , por lo tanto

( ) N1041 3⋅= ,R xt y ( ) N1023 2⋅−= ,R yt ⇒ N1051 3⋅= ,Rt

Como se indica en la figura 7, el ángulo que forma tRr

con la dirección Oeste-Este (eje X) es

( )( )xt

yt

R

Rarctg=θ ⇒ º13−=θ

El pequeño valor de este ángulo significa que se está navegando a la caña, sin tener prácticamente que trabajar de timón, como debe ser.

0vr

Vr

avr β

Fig. 5

Fig. 6

avr

Sr

Rr

R′r

S ′r

β

δVr

X (Este)

Y (Norte)

Fig. 7

tRr

R′r

β

δ X

θRr

Vr

Y

Page 58: XXIII olimpiadas españolas de física, enunciados y soluciones de las pruebas propuestas - pag 214.pdf

d) Cuando el velero navega a motor con velocidad constante nudos03,V =′ la potencia, P, que genera el motor tiene que ser igual a la disipada por la fuerza de resistencia, rF . Es decir

VFP r ′= ⇒ VPFr′

=

Los datos en el S.I. son

W10995735159090 3⋅=⋅⋅== ,,,P,P max , m/s513600

100085213 ,,V =⋅⋅

=′

por lo que resulta

N1046 3⋅= ,Fr

e) La fuerza responsable de que el barco describa una trayectoria circular de radio a, es la fuerza centrípeta cF

aVmFc

2′= ⇒ N,Fc21006 ⋅=

f) Esta fuerza cF es, de nuevo, una fuerza de sustentación hidrodinámica, perpendicular a la velocidad y, por consiguiente, lleva asociada la fuerza de resistencia rF ′ que se añade a la que actúa sobre el barco cuando se mueve de forma rectilínea. De acuerdo con el enunciado 4/FF cr =′ , luego la resistencia total es 4/FF cr + . Por tanto, la potencia necesaria para realizar esta maniobra es

VFFP crnec ′⎟

⎠⎞

⎜⎝⎛ +=

4 ⇒ CV14W1001 4 =⋅= ,Pnec

El motor del velero es capaz de realizar la maniobra puesto que

maxnec PP < .

Page 59: XXIII olimpiadas españolas de física, enunciados y soluciones de las pruebas propuestas - pag 214.pdf

REAL SOCIEDAD ESPAÑOLA DE FÍSICA

Problema Teórico 2

Page 60: XXIII olimpiadas españolas de física, enunciados y soluciones de las pruebas propuestas - pag 214.pdf

Problema 2. El experimento de Cavendish.

Henry Cavendish (1731–1810) fue un notable físico y químico británico. Trabajó en prácticamente todas las áreas de la física de su tiempo, destacando particularmente en sus investigaciones sobre la electricidad y la determinación de parámetros de la Tierra. Concretamente, vamos a analizar su célebre experimento para “pesar la Tierra”.

Desde tiempos de Newton (1643-1727) se conocía que la fuerza de atracción gravitatoria entre dos cuerpos era proporcional al producto de sus masas e inversamente proporcional al cuadrado de la distancia, pero se desconocía la constante de proporcionalidad y también la masa de la Tierra. Cavendish, cuando realizó su famoso experimento, utilizó una balanza de torsión que previamente había diseñado y fabricado John Michell, que murió antes de poder probarla. La figura 1 es una fotografía de una moderna balanza de este tipo que se utiliza para realizar prácticas de laboratorio.

a) Una balanza de torsión consiste en una ligera varilla con dos esferas de masa m en sus extremos, que se mantiene horizontal cuando está suspendida por su punto medio O mediante un hilo sujeto por su extremo superior P, como se muestra en perspectiva en la figura 2. Supondremos que la semilongitud de la varilla es a y que su masa es despreciable frente a las de las esferas. En estas condiciones, cuando se aparta la varilla del equilibrio manteniéndola siempre horizontal y girándola un pequeño ángulo θ, las esferas se mueven describiendo una trayectoria circular de radio a.

Si en un cierto instante la velocidad angular del movimiento de las esferas es dtd /θω = , determine la expresión del módulo del momento angular 0L de las dos esferas respecto al centro O de la varilla. (1,5 Puntos)

b) Al girar un ángulo θ respecto a su posición de equilibrio el sistema varilla-esferas, el hilo del cual está suspendido se opone a que lo “retuerzan” ejerciendo un momento de torsión, τ , que es proporcional al ángulo girado, siempre que sea pequeño. Es decir,

θτ k−=

donde k es la llamada constante de torsión del hilo y el signo tiene en cuenta la oposición del hilo al giro.

En consecuencia, si se aparta el sistema un pequeño ángulo θ respecto a la posición de equilibrio y después se deja libre, realizará oscilaciones torsionales armónicas en torno a dicha posición de equilibrio.

Demuestre1 que el periodo de dichas oscilaciones armónicas viene dado por: ( ) 2/122 /8 kamT π= (2 Puntos)

1 Recuerde que el momento respecto a un punto de las fuerzas que actúan sobre un sistema es igual a la variación temporal del momento angular respecto al mismo punto dtLdM /00

rr= .

Fig. 1

m

m θ

θ

P

O

a

a

Hilo

Fig. 2

Posición de equilibrio

Hilo

Page 61: XXIII olimpiadas españolas de física, enunciados y soluciones de las pruebas propuestas - pag 214.pdf

c) Si kg7300,m = y cm0,90=a y se observa que el periodo de las oscilaciones es s423=T

Calcule el valor de la constante de torsión del hilo, k. (0,5 Puntos)

d) A continuación, sobre cada esfera se aplican fuerzas Fr

y FFrr

−=′ respectivamente, como se indica en la figura 3, en la que se muestra la balanza vista desde arriba. El sistema alcanzará un nuevo estado de equilibrio correspondiente a un pequeño ángulo 0θ , cuando el momento que ejercen dichas fuerzas se equilibre con el momento de torsión del hilo. Dado que

0θ es pequeño puede suponerse que las fuerzas son siempre perpendiculares a la varilla.

Determine la expresión del módulo de la fuerza F aplicada en cada extremo, en función de a, k y 0θ . (1 Punto)

e) En el experimento de Cavendish, la fuerzas Fr

y FFrr

−=′ aplicadas en las bolas de masa m eran las correspondientes fuerzas de interacción gravitatoria, gF

r que ejercían otras bolas de

gran masa M colocadas a unas distancias b, como se muestra en la figura 4.

Obtenga la expresión de la masa de la Tierra, TM , en función de M, m, a, b, 0θ , del radio de la Tierra TR y de la aceleración de la gravedad g. (3 Puntos)

f) Con los datos adicionales siguientes:

kg158=M ; rad1099 40

−⋅= ,θ ; cm023,b = ; m10376 6⋅= ,RT ; m/s819,g =

Calcule la masa de la Tierra, TM , y el valor de la constante de Gravitación Universal G. (1 + 1 Puntos)

m O

Fr

FFrr

−=′

0θ 0θ m

Fig. 3

O

Fig. 4

m

m b

b

gF

gF

M

M

Page 62: XXIII olimpiadas españolas de física, enunciados y soluciones de las pruebas propuestas - pag 214.pdf

Solución Problema 2. El experimento de Cavendish.

a) Cuando al sistema varilla-bolas se aparta del equilibrio, los centros de las esferas describen trayectorias circulares idénticas de radio a, como se muestra en la figura 5. Si la velocidad angular es ω, el módulo de las velocidades lineales de cada bola es av ω= . El momento angular del sistema respecto a O es la suma de los momentos angulares de cada bola, es decir

vamL 20 = ⇒ ω20 2 amL =

b) De acuerdo con la nota a pie de página del enunciado,

τ=dt

dL0 ⇒ θω kdtdam −=22 (1)

Como la aceleración angular es dtd /ωα = , de (1) resulta que

θΩθα 222

−=−=am

k con 22 am

k=Ω

Es decir, la aceleración angular α es proporcional (con signo menos) al ángulo girado θ , lo que significa que ( )tθ es una función armónica con el tiempo, cuyo periodo viene dado por Ωπ /2=T . Por tanto

kamT

228π=

c) Con los datos del enunciado, kg7300,m = , cm0,90=a y s423=T , la constante k de torsión es

Nm102,61 -4⋅=k

d) De acuerdo con la figura 3, el módulo del momento resultante de las fuerzas que actúan sobre las bolas tiene que estar equilibrado con el momento de torsión. Esto es

02 θkaF = ⇒ a

kF2

0θ=

e) Ahora gFF = es la fuerza de interacción gravitatoria entre las bolas de masas M y m que se encuentran más

próximas. Sin tener en cuenta la interacción entre las bolas grandes y las pequeñas más lejanas

a

kb

mMG2

02

θ= (2)

Aunque se supone que G no es conocida, sí se sabe que la aceleración de la gravedad g viene dada por

2T

T

RMGg = (3)

donde TT RM y son la masa y el radio de la Tierra respectivamente. Eliminando G entre (2) y (3) y despejando TM se obtiene

m O m

Fig. 5

v

v

a

a

Page 63: XXIII olimpiadas españolas de física, enunciados y soluciones de las pruebas propuestas - pag 214.pdf

0

2

22θbk

mMaRgM TT =

f) Con los datos adicionales siguientes:

kg158=M ; rad1099 40

−⋅= ,θ ; cm0,23=b ; m10376 6⋅= ,RT ; m/s81,9=g

kg1006 24⋅= ,MT

Por otra parte, conocida ya TM , la constante de Gravitación Universal G puede deducirse de (3) y resulta ser

2211 /kgNm1066 −⋅= ,G

Page 64: XXIII olimpiadas españolas de física, enunciados y soluciones de las pruebas propuestas - pag 214.pdf

XX Olimpiada Española de Física

Problema Experimental

Page 65: XXIII olimpiadas españolas de física, enunciados y soluciones de las pruebas propuestas - pag 214.pdf

Prueba experimental.

Oscilaciones amortiguadas de un péndulo de agua

Objetivos

Se van a estudiar experimentalmente las oscilaciones de la columna de agua contenida en un tubo cilíndrico, doblado en forma de U. Debido a la fricción del agua con las paredes del tubo, esta oscilación es amortiguada y al cabo de unas pocas oscilaciones se alcanza el equilibrio. En concreto, se van a determinar experimentalmente el radio efectivo del tubo y el coeficiente de amortiguamiento. Materiales

- Tubo de goma de 170 cm de longitud. - Listón de madera. - Sargentos y tacos de madera para sujetar el listón a la pata de la mesa. - 4 pinzas para sujetar el tubo al listón. - Cinta adhesiva y tijeras. - Jeringa graduada de 60 cm3. - Cinta métrica. - Cronómetro. - Botella de agua. - Vaso de plástico. Montaje; procedimiento experimental

• El listón de madera se coloca vertical, apoyado en el suelo y en el lateral de la mesa. Se colocan los tacos de madera entre el listón y la pata de la mesa y se sujetan con los sargentos, como indica la figura 1.

• El tubo se sujeta al listón mediante las pinzas, como se indica en la figura 2. Si es necesario, se puede emplear cinta adhesiva para terminar de sujetar el tubo y darle la forma deseada en U.

• Debe procurarse que el tubo mantenga su sección circular, es decir que no se doble en la parte inferior curvada ni se aplaste con las pinzas. Además, los dos laterales del tubo deben quedar verticales.

• Es necesario que quede un trozo de tubo libre en la parte superior de los dos lados, uno de ellos suficientemente largo para poder soplar por él.

• El volumen deseado de agua se introduce en el tubo mediante una jeringa graduada. Deben evitarse en lo posible las burbujas de aire dentro de la jeringa y en la columna de agua dentro del tubo.

• Para forzar la oscilación de la columna de agua, soplar por un extremo del tubo hasta que el agua alcance el nivel deseado y tapar con un dedo el otro extremo. Al quitar el dedo que cierra el tubo comienza la oscilación de la columna de agua. El desplazamiento inicial del agua debe ser suficiente para que puedan observarse cuatro oscilaciones completas antes de que el sistema alcance el equilibrio.

• Atención: cuando se sopla por un extremo del tubo, debe evitarse que el agua rebose por el otro. Si ocurriese esto, sería necesario vaciar el tubo y volver a empezar la medida pues se desconocería el volumen de agua que queda dentro del tubo.

• Antes de realizar medidas es conveniente adquirir práctica con el método de forzar la oscilación del agua y con el manejo del cronómetro.

Fig. 1

Fig. 2

Page 66: XXIII olimpiadas españolas de física, enunciados y soluciones de las pruebas propuestas - pag 214.pdf

Modelo teórico

En equilibrio, el nivel de agua en los dos lados del tubo es el mismo (figura 3.a). Cuando se sopla por un extremo del tubo el sistema se desequilibra (figura 3.b) y, al liberar el sistema, el nivel x del agua oscila en torno al de equilibrio. No es difícil demostrar que, en ausencia de fricción, esta oscilación es armónica, es decir

tAx 0ωcos= (1)

con periodo de oscilación 00 2 ωπ /=T dado por

VgR

T2

20

2π= (2)

donde g es la aceleración de la gravedad, R el radio del tubo y V el volumen de agua.

En ausencia de fricción, la amplitud de oscilación, A, sería constante en el tiempo. Un tratamiento más realista debe tener en cuenta la fricción del agua con las paredes del tubo. Si se considera una fuerza de fricción proporcional a la velocidad del agua, se obtiene

teAx t ωγ cos−= 0 (3)

donde γ es el llamado coeficiente de amortiguamiento. El resultado (3) representa una oscilación armónica con frecuencia angular

T/πω 2= y amplitud A exponencialmente decreciente (figura 4)

teAA γ−= 0 (4)

En particular, la amplitud al cabo de n oscilaciones completas, es decir en nTt n = , es (figura 4)

ntn eAA γ−= 0 (5)

Si el amortiguamiento es débil ( 0ωγ << ), como es nuestro caso, la frecuencia ω coincide aproximadamente con la frecuencia

0ω en ausencia de amortiguamiento, de forma que la expresión (2) para el periodo de oscilación sigue siendo aproximadamente válida.

Pero, debido también a la fricción del agua con las paredes, la columna de agua no oscila "en bloque", es decir como un todo1. Como consecuencia, el radio efectivo del tubo es inferior al real, y por tanto el periodo de oscilación es mayor que el previsto en ausencia de fricción, de forma que

VgR

T

ef2

2 2π= (6)

con RRef < , y 0TT > .

En la primera parte de esta prueba experimental se van a realizar una serie de medidas para determinar los radios efectivo, efR , y real, R, del tubo. En nuestro montaje, estos dos radios son claramente diferentes, es decir la diferencia entre ambos es mayor que sus incertidumbres experimentales. En la segunda parte se obtendrá el coeficiente de amortiguamiento γ.

1 La capa de agua en contacto con las paredes prácticamente no se mueve.

2R

V

x

Fig. 3.a Fig. 3.b

gr

0A−

0

0A

0 1t 2t 3t 4t

t

teA γ−0

Fig. 4

x

1A 2A

Page 67: XXIII olimpiadas españolas de física, enunciados y soluciones de las pruebas propuestas - pag 214.pdf

Medidas y preguntas 1ª parte. Determinación de Ref y R.

1.a) Mediante la jeringa graduada, añade agua dentro del tubo de forma que el volumen de agua sea sucesivamente V = 30, 40, 50 ...100 cm3. En cada uno de estos casos:

• Mide con el cronómetro el periodo de oscilación del agua en torno a su nivel de equilibrio. Sugerencia: mide cuatro veces el tiempo de cuatro oscilaciones completas (4T1, 4T2, 4T3 y 4T4) y deduce el periodo T promediando los resultados. Presenta tus medidas y resultados en la tabla 1 de la hoja de respuestas.

• Para cada V, marca con bolígrafo en los dos lados del tubo la posición del nivel de agua en equilibrio. La distancia entre estas dos marcas, que se medirá más tarde cuando se vacíe y estire el tubo, permitirá determinar su radio R. Ten cuidado de que estas marcas no se borren.

1.b) Representa gráficamente en un papel milimetrado los puntos experimentales 2T (en ordenadas) frente a V (en abscisas).

1.c) Obtén la pendiente de la recta que mejor se ajusta a estos puntos.

1.d) Deduce el valor del radio efectivo del tubo, efR .

1.e) Haz una estimación de la incertidumbre (margen de error) del radio efectivo, efRΔ .

1.f) Desmonta con cuidado un lado del tubo y vacía el agua en un vaso. Desmonta completamente el tubo y estíralo sobre la mesa, mide la distancia L entre las marcas simétricas que has hecho para cada V y deduce en cada caso el radio R del tubo. Anota los valores de L y R en las columnas correspondientes de la tabla 1 en la hoja de respuestas.

1.g) Calcula el valor medio de R y haz una estimación de su incertidumbre.

2ª parte. Determinación de γ.

Vuelve a sujetar el tubo en U al listón de madera. Introduce 60 cm3 de agua y marca en el listón el nivel de agua en equilibrio. Haz otra marca unos 20 cm por encima. Ésta será la posición inicial del nivel de agua, 0A , en todas las medidas posteriores. Antes de medir debes adquirir práctica en conseguir enrasar el nivel de agua con esta marca, soplando por el otro lado hasta alcanzar un nivel un poco más alto, tapando el tubo con el dedo y dejando entrar un poco de aire hasta que el nivel alcance la marca.

2.a) Mide las amplitudes A1, A2 y A3 al cabo de una, dos y tres oscilaciones completas, respectivamente, es decir en Tt =1 , Tt 22 = y Tt 33 = (el periodo T para este volumen de agua ya ha sido medido previamente). Repite

la observación de la oscilación amortiguada todas las veces que sea necesario para, mediante aproximaciones y marcas sucesivas, determinar estas amplitudes con suficiente precisión. Presenta tus medidas en la tabla 2 de la hoja de respuestas.

2.b) Transforma la expresión (4) para obtener una relación lineal entre una función de la amplitud, F(A), y el tiempo t.

2.c) Representa gráficamente los puntos experimentales correspondientes, F(A) en función de t. Ajusta estos puntos a una línea recta.

2.d) Deduce el coeficiente de amortiguamiento, γ.

Page 68: XXIII olimpiadas españolas de física, enunciados y soluciones de las pruebas propuestas - pag 214.pdf

Prueba experimental. Solución 1.a) Ejemplo de medidas reales con un tubo de radio nominal 5 mm.

Tabla 1

V (cm3) 4T1 (s) 4T2 (s) 4T3 (s) 4T4 (s) T (s) T 2 (s2) L (cm) R (mm)

30 3,69 3,75 3,75 3,69 0,930 0,865 39,5 4,917 40 4,32 4,25 4,25 4,37 1,074 1,154 53,0 4,901 50 4,91 4,87 4,84 4,84 1,216 1,479 66,0 4,911 60 5,25 5,34 5,22 5,25 1,316 1,733 79,5 4,901 70 5,75 5,71 5,71 5,78 1,434 2,057 92,0 4,921 80 6,18 6,12 6,28 6,16 1,546 2,391 106,0 4,901 90 6,57 6,6 6,5 6,47 1,634 2,669 118,5 4,917 100 6,93 6,96 6,97 6,91 1,736 3,012 131,5 4,920

1.b) Se presenta la gráfica pedida, con un aspecto parecido el que tendría dibujada en papel milimetrado.

1.c) Los puntos experimentales se ajustan bien a una dependencia lineal que pasa por el origen, como prevé la ecuación (6) del enunciado. La pendiente de esta recta puede obtenerse fácilmente y con buena precisión observando las coordenadas de un punto de la recta alejado del origen. Por ejemplo, el extremo derecho de la recta tiene coordenadas (V; 2T ) = (110 cm3; 3,26 s2), luego la pendiente de la recta es

3

2

cm110

s263,=p , 324 ms10962 −⋅= ,p

Nota: un ajuste por mínimos cuadrados conduce a una pendiente 324 ms109652 −⋅= ,p .

0,0

0,5

1,0

1,5

2,0

2,5

3,0

3,5

0 10 20 30 40 50 60 70 80 90 100 110

V (cm3)

T 2 (s2)

y = 0,02965x

Page 69: XXIII olimpiadas españolas de física, enunciados y soluciones de las pruebas propuestas - pag 214.pdf

1.d) De acuerdo con (6), la pendiente de esta recta es

2

2

efgRp π= → m106542 3-⋅== ,

pgRef

π , mm654,=efR

1.e) La incertidumbre del radio efectivo depende de la incertidumbre de la pendiente. Esta última puede estimarse trazando las rectas que con pendientes máxima y mínima se ajustan razonablemente a los puntos experimentales, teniendo en cuenta su dispersión en torno a la recta óptima2. En la siguiente figura se presenta una estimación gráfica de estas rectas.

Las pendientes de estas dos rectas se obtienen como en el apartado 1.c.

3243

2ms10013

cm110

s313 −⋅== ,,

maxp ; 3243

2ms10922

cm110

s213 −⋅== ,,

minp

Los correspondientes radios efectivos resultan

mm6142 ,max

min, ==pg

Refπ ; mm6842 ,

minmax, ==

pgRef

π

Por tanto

2

min,max, efefef

RRR

−=Δ , mm040,=Δ efR

Nota: la incertidumbre de la pendiente p puede calcularse analíticamente. Con un nivel de confianza alto, del 95%, se obtiene una incertidumbre relativa del 1,2%. Como el radio efectivo es proporcional a 21 /−p , el error relativo transmitido al radio es del 0,6%, de forma que mm030,=Δ efR . Como suele ocurrir, el método de estimar las pendientes máxima y mínima sobrevalora las incertidumbres.

2 La incertidumbre de cada valor experimental de T

2 (barra de error) puede estimarse mediante el error probable de las varias medidas realizadas. El resultado es inferior o del orden del radio de los puntos dibujados en la gráfica.

V (cm3)

T 2 (s2)

0,0

0,5

1,0

1,5

2,0

2,5

3,0

3,5

0 10 20 30 40 50 60 70 80 90 100 110

Page 70: XXIII olimpiadas españolas de física, enunciados y soluciones de las pruebas propuestas - pag 214.pdf

1.f) Los valores de L medidos se recogen en la penúltima columna de la Tabla 1. El radio se calcula en cada caso teniendo en cuenta que

LRV 2π= → L

VRπ

=

Los valores obtenidos para R se presentan en la última columna de la Tabla 1.

1.g) El promedio y el error probable3 de esta serie de datos son

mm911 4, =R , mm0030,=ΔR

2.a) Tabla 2. Amplitudes tras varias oscilaciones para V = 60 cm3

2.b) Tomando logaritmos en (4) se obtiene una dependencia lineal entre ln(A) y t, con pendiente –γ.

tAA γ−= )ln()ln( 0

2.c) Los puntos experimentales se ajustan bien a la dependencia prevista, como puede observarse en la siguiente gráfica

La pendiente de la recta puede obtenerse a partir de las coordenadas de los puntos extremos

11 s580s054972370 −− −=

−−= ,

,,,

p

2.d) La pendiente de esta recta es –γ, de forma que

1s580 −= ,γ

3 Para un nivel de confianza del 95%, el margen de incertidumbre del radio se eleva a 0,007 mm

n nt (s) nA (cm) )ln( nA

0 0,000 20,0 3,00

1 1,316 8,8 2,18

2 2,632 4,2 1,44

3 3,948 2,0 0,69

t (s)

ln(A)

y = -0,581x + 2,972

0,0

0,5

1,0

1,5

2,0

2,5

3,0

3,5

0,0 0,5 1,0 1,5 2,0 2,5 3,0 3,5 4,0 4,5 5,0

Page 71: XXIII olimpiadas españolas de física, enunciados y soluciones de las pruebas propuestas - pag 214.pdf

XX Olimpiada Española de Física

Problema Teórico nº 1

Page 72: XXIII olimpiadas españolas de física, enunciados y soluciones de las pruebas propuestas - pag 214.pdf

Problema 1. Física en los encierros de San Fermín. ¿Pero, hay Física en los encierros? Claro que la hay, mucha y muy compleja. En este ejercicio se pretende poner de

manifiesto algún aspecto físico de este popular festejo. Naturalmente, será necesario utilizar modelos muy simples; de lo contrario, el problema sería prácticamente irresoluble.

La longitud del recorrido del encierro, desde la salida de los corralillos de Santo Domingo hasta la entrada de la plaza de toros es de 848,6 m y la duración promedio de los últimos treinta años, es de 3,55 min.

a) Determine la velocidad media, V, de un toro en un encierro rápido y limpio como el del 8 de julio de 2008, cuya duración fue de 2 min 17 s.

Un tramo singular del encierro es la cerrada curva con la que se inicia la calle Estafeta. Su pavimento es prácticamente horizontal y allí los toros suelen derrapar dando lugar a situaciones peligrosas. La figura 1 es una fotografía a vista de pájaro de la famosa curva. En ella, superpuesta, se muestra la hipotética trayectoria de un toro y se supone que la curva es un arco de circunferencia de radio R.

b) Haga una estimación de R utilizando la escala que también se muestra en la fotografía.

Siendo coherentes con la “vista de pájaro” de la figura 1, podemos considerar al toro como una partícula puntual que traza la curva de la Estafeta con una velocidad de módulo constante V, la calculada en el apartado (a). Además, suponemos que el toro no llega a “derrapar” pero está en el límite, es decir, que no desliza por muy poco.

c) Con estos supuestos, haga una estimación del coeficiente de rozamiento μ entre las pezuñas del cornúpeta y el pavimento de la curva de la Estafeta.

Supongamos ahora que un toro de masa kg512=M (que visto de cerca no es precisamente una partícula puntual…) que se mueve con la velocidad V del apartado (a), embiste a un despistado forastero de masa kg75=m que se encontraba parado en la calle (no lo empitona, simplemente choca con él). Los dibujos a y b de la figura 2 corresponden respectivamente a los instantes inmediatamente anterior y posterior al choque y se supone que el intervalo de tiempo tδ transcurrido entre ellos es muy corto. Como consecuencia del choque, la velocidad del toro se reduce en un 10 %.

d) ¿Cuál es la velocidad v del forastero inmediatamente después del choque?

e) ¿Cuánto ha variado la energía mecánica del toro toroEΔ ? ¿Se transfiere toroEΔ íntegramente al forastero? Razone su respuesta.

f) Si estimamos como “duración del choque” un tiempo s30,t =δ , ¿cuál es la fuerza media F con que el toro empuja al forastero durante el choque?

Fig. 2 a) Inmediatamente antes del choque

V

b) Inmediatamente después del choque

vV´

Fig. 1.- La curva de la Estafeta

Page 73: XXIII olimpiadas españolas de física, enunciados y soluciones de las pruebas propuestas - pag 214.pdf

Problema 1. Física en los encierros de San Fermín.

Solución

a) Como la longitud del recorrido es m6848,L = y el tiempo es s137s17min2 ==T , la velocidad media es

km/h322m/s196 ,,V ==

b) Para medir el radio de curvatura de la “curva de la Estafeta”, se puede trazar sobre la fotografía de la figura 1 del enunciado una circunferencia que se “ajuste” lo mejor posible a la trayectoria, como se muestra en la figura 3. Con una regla se mide el diámetro de dicha circunferencia y la longitud de la escala. El radio de curvatura R será

m0,510escala la de Longitud

diametro21

==R

c) El toro, considerado como una partícula puntual, describe una trayectoria circular con velocidad constante. En consecuencia, si llamamos rF a la fuerza de rozamiento, se verificará

rFRVM =

2 (1)

Si el toro no “derrapa” pero está en límite de hacerlo, la fuerza de rozamiento tiene que estar muy próxima a su valor máximo MgNFr μμ == , siendo M la masa del toro. Sustituyendo en (1), resulta

MgRVM μ=

2 ⇒

gRV 2

=μ ⇒ 780,=μ

d) Considerando como sistema mecánico al conjunto toro-forastero, en el proceso del choque entre ambos actúa como fuerza exterior, paralela al suelo, la fuerza de rozamiento. Luego, en la dirección horizontal, no se conserva el momento lineal del sistema. Si llamamos Pi y Pf a los momentos lineales inicial y final del sistema, el teorema del impulso permite escribir

IPP if =−

Donde MVPi = , mv´MVPf += e ∫=t

r dtFIδ

0

Fig. 3.- La curva de la Estafeta

Page 74: XXIII olimpiadas españolas de física, enunciados y soluciones de las pruebas propuestas - pag 214.pdf

siendo m la masa del forastero, V´ y v las velocidades del toro y del forastero después del choque.

Ahora bien, aunque la fuerza de rozamiento puede variar en el intervalo de tiempo tδ , nunca puede llegar a superar su valor máximo. Por ello y teniendo en cuenta el pequeño valor de tδ que indica el enunciado, su impulso puede considerarse nulo y entonces resulta que

0≈− if PP ⇒ mvMVMV += ´

O lo que es igual,

( )´VVmMv −=

Como el enunciado nos dice que “la velocidad del toro se reduce en un 10 %, se tiene que

( ) 10,V

´VV=

Y teniendo en cuenta las masas de los protagonistas del choque, el módulo de la velocidad del forastero resulta

m/s24680 ,V,v ==

e) Dado que todo el proceso se lleva a cabo sin variación apreciable de energía potencial gravitatoria, la variación de la energía mecánica del toro es

( )22

21 V´VMEtoro −=Δ

Naturalmente es negativa, ya que V,´V 90= , (el toro tiene que perder energía). En valor absoluto la perdida es

kJ91,Etoro =Δ

¿Se transfiere toroEΔ íntegramente al forastero? No, el toro no es un “sólido rígido”. En el breve “encuentro” con el forastero sufrirá alguna deformación que conlleva trabajo de las fuerzas interiores. Este trabajo se realiza a expensas de una fracción de la energía mecánica del cornúpeta.

Si se considera ahora al forastero como sistema mecánico, durante el corto tiempo tδ que dura el choque, actúa como fuerza externa la que ejerce el toro sobre el sufrido forastero. Esta fuerza es nula antes y después del choque pero no lo es durante los s,t 30=δ que dura el encontronazo. Es pues una fuerza de percusión F(t) que puede llegar a tener valores muy altos en el intervalo de tiempo tδ . El impulso de esta fuerza es igual a la variación del momento lineal del forastero que es mv , por lo tanto

( )∫=t

dttFmvδ

0

f) Si sustituimos ( )tF por su valor medio en el intervalo tδ ,

∫∫ ==ttdtFdtFmv

δδ

00 ⇒

tmvFδ

= ⇒ N1011 3⋅= ,F

Page 75: XXIII olimpiadas españolas de física, enunciados y soluciones de las pruebas propuestas - pag 214.pdf

Problema Teórico nº 3

Page 76: XXIII olimpiadas españolas de física, enunciados y soluciones de las pruebas propuestas - pag 214.pdf

Problema 3. El fracaso del átomo clásico.

En los albores de las investigaciones sobre el átomo, a principios del siglo XX, se pensaba que éste tenía una estructura similar a un sistema planetario con el núcleo en el centro, muy pesado y cargado positivamente, y los ligeros electrones girando a su alrededor, ligados por la atracción coulombiana.

Pronto se descubrió que las cosas no podían ser tan simples: las cargas eléctricas cuando se mueven con aceleración, como los electrones orbitando alrededor del núcleo, pierden energía en forma de radiación electromagnética, por lo que los electrones atómicos se precipitarían hacia el núcleo en un tiempo muy breve. Está claro que no sucede así, puesto que la materia normal es estable y ustedes están aquí.

Este problema tiene por objeto hacer una estimación, clásica y no relativista, del tiempo que tardaría en aniquilarse un átomo de hidrógeno según las teorías clásicas de la mecánica y de la radiación, partiendo del hecho de que la energía de ionización de un átomo de hidrógeno en su estado fundamental o no excitado es eV613,Eioniz = .

1) Suponiendo que la órbita del electrón es circular de radio r, obtenga las expresiones de la energía total del electrón E, de su velocidad v, de su aceleración a y de su periodo de revolución T, en función de la energía total E0 del átomo de hidrógeno en el estado no excitado y del radio r0 de la órbita circular en dicho estado.

2) Determine y calcule E0 y r0 así como la velocidad, la aceleración y el periodo en esta órbita, v0, a0, y T0, respectivamente.

Cuando un electrón se mueve con aceleración a, la potencia radiante que emite viene dada por la fórmula de Larmor:

23

2

32 a

cekP =

en la que k es la constante de Coulomb, c la velocidad de la luz y e la carga elemental

3) Al perder energía, el electrón irá describiendo órbitas de radio cada vez menor. Halle la expresión de la potencia emitida P en función del radio r de la órbita circular del electrón, de r0 y de E0.

Puede comprobar que tanto ( )rE como ( )rP tienden a infinito cuando 0→r . Este absurdo físico es consecuencia de que en las antiguas teorías se idealizaba al electrón y al protón considerándolos como partículas puntuales. Pasemos por alto estos inconvenientes y vayamos al objetivo de este ejercicio:

4) ¿Cuál es el tiempo τ que le costaría a un electrón “caer” sobre el núcleo desde la órbita del estado fundamental de radio r0?

AYUDA: La potencia radiada por el electrón (fórmula de Larmor) tiene que ser igual a la variación (con signo negativo) de la energía mecánica del electrón por unidad de tiempo, dtdE /− . Esta variación representa una pérdida de energía, de ahí el signo negativo.

Le resultarán útiles las siguientes expresiones:

dtdr

drdE

dtdE

= ⇒ drdrdE

dtdEdt

/1

=

El tiempo τ lo obtendrá integrando entre los valores inicial y final de t y de r. (La integral es inmediata).

DATOS: J1060,1eV1 19−⋅= ; masa del electrón kg10119 31−⋅= ,me ; constante de Coulomb 229 CmN10998 −⋅= ,k ;

velocidad de la luz -18 sm10003 ⋅= ,c ; carga elemental C10601 19−⋅= ,e .

CONCLUSIÓN: Si ha llegado al final del problema comprobará que la vida de un átomo según la teoría clásica es muy corta. El átomo clásico es un fracaso. La teoría cuántica es la que describe correctamente la estabilidad de la materia, prediciendo además las propiedades de los átomos.

e p

H

r

v (r)

Page 77: XXIII olimpiadas españolas de física, enunciados y soluciones de las pruebas propuestas - pag 214.pdf

Problema 3. El fracaso del átomo clásico.

Solución

1) La energía mecánica del electrón en una órbita de radio r es

rekvmE e

22

21

−= (1)

Como la fuerza que actúa sobre el electrón es central y describe una órbita circular, su velocidad angular es constante, por lo que su aceleración es sólo centrípeta y se verificará

rvm

rek e

2

2

2= ⇒

rekvme

22 = ,

sustituyendo en (1) r

keE2

21

−= (2)

En el estado fundamental, 0rr = , luego

( ) 00

2

0 21 E

rkerE =−= ⇒ 00

2 2 rEke −= (3)

rr

EE 00= (4)

rm

rErm

kevee

002

2 2−== ⇒

rmrEv

e

002−=

r

va2

= ⇒ 2002

rmrEa

e−= (5)

v

rT π2= ⇒ 23

0022 /e r

rEmT

−= π

2) Como la energía de ionización es la mínima energía que hay que comunicar a un átomo para arrancarle un electrón (que alcance el infinito con velocidad cero), se puede escribir

00 =+ ionizEE ⇒ ionizEE −=0 ⇒ J10182eV613 180

−⋅−=−= ,,E

De (3), el valor de 0r es

0

2

0 2Eker −= ⇒ m10295 11

0−⋅= ,r

Si en el resto de expresiones anteriores hacemos 0rr = se obtiene

em

Ev 00

2−= ⇒ m/s10192 6

0 ⋅= ,v

0

00

2rm

Eae

−= ⇒ 2220 m/s10059 ⋅= ,a

Page 78: XXIII olimpiadas españolas de física, enunciados y soluciones de las pruebas propuestas - pag 214.pdf

00

0 22 r

EmT e

−= π ⇒ s10521 16

0−⋅= ,T

3) Llevando a la fórmula de Larmor las anteriores expresiones (3) y (5), se obtiene

432

30

30 1

316

rcmrEP

e−= (6)

4) La potencia radiada tiene que ser igual a la variación (con signo negativo) de la energía mecánica del electrón por unidad de tiempo, es decir:

dtdEP −=

Luego, teniendo en cuenta la “ayuda”

drdrdE

dtdEdt

/1

= ⇒ drdrdE

rPdt

)(1

−=

Derivando en (4) 200

rrE

drdE

−=

Teniendo en cuenta además la expresión (6) queda

drrrE

mcdt e 220

20

23

163

−=

Integrando desde un instante inicial, 0=t en el que 0rr = , hasta un instante τ=t en el que 0=r , esto es

∫∫ −=0 2

20

20

23

0 0163

re drr

rEmcdt

τ ⇒ 02

0

23

161 r

Emc e=τ ⇒ s10561 11−⋅= ,τ

Page 79: XXIII olimpiadas españolas de física, enunciados y soluciones de las pruebas propuestas - pag 214.pdf

REAL SOCIEDAD ESPAÑOLA DE FÍSICA

XIX OLIMPIADA ESPAÑOLA DE FÍSICA

ENUNCIADOS Y SOLUCIONES DE LAS PRUEBAS PROPUESTAS

Mieres (Asturias), 4 a 7 de abril de 2008

Page 80: XXIII olimpiadas españolas de física, enunciados y soluciones de las pruebas propuestas - pag 214.pdf

XIX OLIMPIADA ESPAÑOLA DE FÍSICA

Página 2 de 22

Problema 1. Anemometría sónica.

Hoy en día, los Centros Meteorológicos disponen de aparatos muy sofisticados para medir la velocidad del viento que, además y simultáneamente, miden la temperatura del aire. El ejercicio que se propone hace referencia a uno de estos aparatos, el anemómetro sónico, que a diferencia de los convencionales carece de partes móviles como son las cazoletas giratorias, veleta, etc. El funcionamiento de este tipo de anemómetro se basa en la medida de tiempos de propagación de señales sonoras.

Como introducción, y para facilitar la resolución del ejercicio, se propone en primer lugar el conocido problema del movimiento de una canoa en un río.

Una canoa que navega a una velocidad constante c respecto al agua, realiza una trayectoria rectilínea desde un punto A hasta otro B separados una distancia L y después, realiza el trayecto inverso de B hasta A. Los tiempos que tarda en hacer los recorridos son 1t′ y 2t′ respectivamente, siendo 12 tt ′>′ porque existe una corriente de velocidad av

r que tiene la dirección de A a B, tal como

se muestra en la figura 1.

a) Deduce las expresiones analíticas que permitan calcular c y av (módulo de avr

) en función de 1t′ y de 2t′ y de la distancia L.

Supón ahora, y en todo lo que sigue, que la canoa es una señal sonora que se propaga en el aire con una velocidad c y que la velocidad av

r es la del viento. b) Si la velocidad de propagación del sonido es m/s1044,3 2⋅=c y la del viento es

km/h0,100=av , calcula los tiempos 1t ′ y 2t′ tomando cm0,20=L

Considera que el viento “no sopla” en la dirección AB sino que lo hace de forma que avr

tiene dos componentes, Nv y Lv , perpendicular y paralela a AB, respectivamente, tal como se indica en la figura 2.

c) Como sólo se puede determinar la componente longitudinal de la velocidad del viento, deduce la expresión de Lv en función de la distancia L y de los tiempos 1t y 2t que transcurren, respectivamente, desde que la señal sonora es emitida en A y detectada en B y viceversa.

d) Teniendo en cuenta que avc >> , deduce la expresión de la velocidad del sonido, c, en función de L, 1t y 2t .

La velocidad de propagación del sonido en el aire, c, depende de la densidad del aire ρ y de su módulo de compresibilidad B, según la expresión:

ρBc =

En la propagación de una onda sonora, cada elemento de volumen del aire, V, experimenta variaciones periódicas de la presión, pΔ , que dan lugar a variaciones, VΔ , de su volumen. El módulo de compresibilidad representa el factor de proporcionalidad entre dichas variaciones, es decir, se define como

VVBp Δ

−=Δ

Ahora bien, la relación pV ΔΔ / depende del tipo de transformación termodinámica que experimente el gas (aire) contenido en el elemento de volumen.

Dada la baja conductividad térmica de los gases y la rapidez con que se producen las variaciones de presión y de volumen, estos procesos deben considerarse adiabáticos. En consecuencia, el modelo adiabático de propagación del sonido es el más indicado para aire seco, por lo que, si γ es el índice adiabático del aire, se tiene

0=Δ+Δ VppV γ

e) Considerando que el aire, de masa molecular media aM , se comporta como un gas perfecto, obtén la expresión de c en función de la temperatura T del aire.

f) ¿A qué temperatura la velocidad del sonido tendrá el valor indicado en el apartado b?

mol)J/(K315,8=R kg/mol108,28 3−⋅=aM 40,1=γ

B

A

avr

Fig. 1

L

B

A

avr

Fig. 2

LLv Nv

Page 81: XXIII olimpiadas españolas de física, enunciados y soluciones de las pruebas propuestas - pag 214.pdf

XIX OLIMPIADA ESPAÑOLA DE FÍSICA

Página 3 de 22

Nota: El problema propuesto constituye el fundamento de los anemómetros sónicos, ampliamente usados en la actualidad y descritos por V. E. Suomi y J. A. Businger (Geophys. Research Papers No 59, Vol. III, 1, 1959).

Puesto que sólo se puede determinar la componente longitudinal de la velocidad del viento, vL, en función de los tiempos de vuelo de la señal, es necesario disponer de tres dispositivos análogos que midan dicha componente en tres direcciones no coplanarias para que la velocidad del viento quede completamente determinada, como se puede apreciar en la fotografía de la figura 3.

La dirección del viento puede medirse con una exactitud de º1± y su velocidad con %1± en el rango de 0 a 215 km/h. Asimismo, miden la velocidad del sonido con la misma exactitud en el rango de 300 a 380 m/s.

Dada la dependencia de la velocidad del sonido con la temperatura, estos anemómetros permiten medirla con una exactitud de %1± en el rango comprendido entre -30ºC y 50ºC.

Solución Problema 1

a) Consideremos dos sistemas de referencia, uno fijo ligado a los puntos A y B y otro ligado al agua que se mueve respecto al primero con velocidad av

r. Las velocidades de la barca en ambos sistemas de referencia están relacionadas por:

avcvrrr

+=′ (1) Si llamamos 1v

r′ y 2vr′ a las velocidades con las que se mueve la barca respecto

al primer sistema en los recorridos citados, y dado que todos los vectores velocidad tienen la misma dirección, sus módulos verifican

a

avcvvcv

−=′+=′

2

1

Por otra parte, 11 / tLv ′=′ y 22 / tLv ′=′ , luego

avc

Lt+

=′1 , avc

Lt−

=′2 (2)

Lógicamente, 12 tt ′>′ . Sumando y restando las expresiones (2) se obtienen las relaciones buscadas

21

122 tt

ttLva ′′′−′

= , 21

122 tt

ttLc′′

′+′=

b) Con los datos del enunciado, de (1) se obtiene:

ms538,0s1038,5 41 =⋅=′ −t , ms633,0s1033,6 4

2 =⋅=′ −t

c) La relación entre las velocidades es la dada en (1) pero ahora los vectores tienen direcciones diferentes. En los trayectos AB y BA, los correspondientes diagramas de velocidades son los dados en la figura 5.

Las direcciones de las velocidades 1vr y 2vr son la de AB,

deberá cumplirse que

βββββ ==⇒== 2121 sensen ccvN

Por otra parte, de los diagramas de la figura 5, se deduce

L

Lvcvvcv

−=+=

ββ

coscos

2

1 (3)

siendo 11 / tLv = y 22 / tLv =

sustituyendo estas últimas expresiones en (3) y restando ambas ecuaciones se obtiene inmediatamente vL en función de los tiempos de ida y de vuelta de la señal

⎟⎟⎠

⎞⎜⎜⎝

⎛ −=

21

122 tt

ttLvL (4)

Fig. 3

1βcr

Nv

1vr

2βcr 2v

r

BA→ AB→

Fig. 5

Lvavr

avr

Nv

Lv

cr 1v

r′

BA →

cr 1v

r′

AB →Fig. 4

avr

avr

Page 82: XXIII olimpiadas españolas de física, enunciados y soluciones de las pruebas propuestas - pag 214.pdf

XIX OLIMPIADA ESPAÑOLA DE FÍSICA

Página 4 de 22

d) Si sumamos las expresiones (3), con las velocidades de la señal en función de los tiempos de vuelo, resulta

⎟⎟⎠

⎞⎜⎜⎝

⎛ +=

21

212

costtttLc β (5)

Como el ángulo β es desconocido, para poder eliminarlo, el teorema de Pitágoras aplicado a los triángulos rectángulos de la figura 5, nos proporciona la siguiente ecuación

2222 cos Nvcc −=β (6)

Sustituyendo en (5)

2

21

212

224 ⎟⎟

⎞⎜⎜⎝

⎛ +=−

ttttLvc N

Ya que c >> va, también c >> vN por lo que

2

21

212

24 ⎟⎟

⎞⎜⎜⎝

⎛ +≈

ttttLc ⇒

21

212 tt

ttLc +≈ (7)

d’) Otra forma de determinar c.

Del diagrama A → B de la figura 5 se deduce que

Lvcv += βcos1 ⇒ Lvvc −= 1cos β

Elevando al cuadrado y teniendo en cuenta (6)

122

122 2 vvvvvc LLN −+=− ⇒ 1

2221

2 2 vvvvvc LNL −++=

Pero, 222aNL vvv =+ y como c >> va, se puede escribir, recordando que 11 / tLv =

LvtL

tLc

121

22 2−≈

Sustituyendo el valor de Lv de la expresión (4), se deduce que

21tt

Lc ≈ (8)

Nota:

Las expresiones (7) y (8) son, a primera vista, muy diferentes pero ambas válidas. En efecto la ecuación (7) puede expresarse así en función del cociente entre de las medias aritmética y geométrica de 1t y 2t

( )2121

21 12/tttt

ttLc +≈

Como ambas medias son prácticamente coincidentes cuando, como es el caso que nos ocupa, 21 tt ≈ , resulta que

21

1tt

Lc ≈

que coincide con la expresión (8).

De (7) y (8) se llega también a

21

2tt

Lc+

Page 83: XXIII olimpiadas españolas de física, enunciados y soluciones de las pruebas propuestas - pag 214.pdf

XIX OLIMPIADA ESPAÑOLA DE FÍSICA

Página 5 de 22

Dado que 21 tt ≈ , puede deducirse esta expresión de forma independiente sin más que escribir que

tLc ≈

donde t es el tiempo medio de ida y vuelta de la señal sonora

e) En virtud de la información que aporta el enunciado de este apartado, la velocidad del sonido puede escribirse como

VpVc

ΔΔ

−=ρ

(9)

Naturalmente, un aumento de presión ( 0>Δp ) implica una disminución del volumen ( 0>ΔV ) y viceversa, es decir, la relación Vp ΔΔ / tiene que ser necesariamente negativa, cualquiera que sea la transformación termodinámica que sufra el

elemento de volumen.

En este caso, y de nuevo tal como sugiere el enunciado para aire seco, el modelo de transformación más indicado es el adiabático en el que se verifica que

V

pVp γ

−=ΔΔ

Sustituyendo en (9)

ρ

γ pc = (10)

Asumiendo que el aire se comporta como un gas perfecto, llamando M a la masa de aire encerrado en el volumen V y T a su temperatura absoluta, se tiene que

RTMMnRTpV

a== ⇒

aMRTp

Sustituyendo en (19) se obtiene la expresión de la velocidad de propagación del sonido en el aire, en función de la temperatura

TM

Rca

γ= (11)

f) La velocidad del sonido indicada en el apartado (b) es m/s1044,3 2⋅=c . La temperatura correspondiente, de acuerdo con

(11), vendrá dada por

R

McT aγ

2= ⇒ K1093,2 2⋅=T

Page 84: XXIII olimpiadas españolas de física, enunciados y soluciones de las pruebas propuestas - pag 214.pdf

XIX OLIMPIADA ESPAÑOLA DE FÍSICA

Página 6 de 22

Problema 1. Tabla de Respuestas

Apartado Expresiones analíticas Resultados numéricos Puntos

a 21

122 tt

ttLva′′

′−′= , 21

122 tt

ttLc′′

′+′= 0,5 + 0,5

b ms5380s10385 41 ,,t =⋅=′ −

ms6330s10336 42 ,,t =⋅=′ −

0,5 + 0,5

c ⎟⎟⎠

⎞⎜⎜⎝

⎛ −=

21

122 tt

ttLvL 3

d 21

212 tt

ttLc +≈ ó

21

1tt

Lc ≈

ó 21

2tt

Lc+

2

e TM

Rca

γ=

2

f K10932 2⋅= ,T 1

Page 85: XXIII olimpiadas españolas de física, enunciados y soluciones de las pruebas propuestas - pag 214.pdf

XIX OLIMPIADA ESPAÑOLA DE FÍSICA

Página 7 de 22

Problema 2. La rueda hidráulica.

La rueda hidráulica es uno de los más antiguos artilugios para producir trabajo útil. Como antecedentes de las modernas turbinas, estos dispositivos captan energía de una corriente de agua y la transforman en trabajo mecánico para mover molinos, batanes, martillos pilones en las ferrerías, etc. y para bombear agua, como es el caso de las norias, objeto de este ejercicio.

En España han existido infinidad de norias y algunas todavía subsisten, como ejemplo verdaderamente notable es la de Albarán (Murcia). En el río Ebro, una noria da nombre a un antiguo monasterio cisterciense: el Monasterio de Rueda. Con motivo de la celebración en España de la Exposición Internacional de 2008 que tendrá lugar en Zaragoza y para hacer honor a su lema: el agua, se instalará una noria de gran tamaño y enteramente de madera, construida en Siria donde son famosas las existentes en Hama, en el río Orantes, que datan del siglo XII y aún permanecen en funcionamiento.

Una noria hidráulica destinada a la elevación de agua es esencialmente una rueda con paletas (álabes), colocada con su eje horizontal, como se muestra en la figura 1. La corriente de un río, acequia o canal ejerce una fuerza sobre los álabes sumergidos de su parte inferior que producen la rotación de la rueda. En el perímetro de la rueda se disponen unos pequeños recipientes (cangilones o arcaduces) que se llenan de agua al estar sumergidos y cuando, debido al giro de la rueda, alcanzan la parte superior vierten su contenido en un canal. De esta forma elevan agua a una altura aproximadamente igual al diámetro de la rueda.

Una reproducción de una noria romana, con cangilones de barro en forma de cántaros, se encuentra en el parque de María Cristina de Algeciras (figura 2) y en la fotografía de la figura 3 aparecen unas norias de Hama; la mayor es la que está siendo reproducida en la EXPO2008 y que en la figura 4, está representada esquemáticamente y a escala. (Foto y esquema, cortesía de EUROTEC, empresa encargada de su proyecto y construcción)

Fig. 1

Paletas (álabes) Cangilones

Canal superior

Corriente de agua

Fig. 2

Fig. 3

Fig. 4 v

ω

Page 86: XXIII olimpiadas españolas de física, enunciados y soluciones de las pruebas propuestas - pag 214.pdf

XIX OLIMPIADA ESPAÑOLA DE FÍSICA

Página 8 de 22

En la noria representada en la figura 4, objeto de este ejercicio, el agua de la acequia que se mueve con una velocidad media v, “choca” con el álabe plano, de área A, que se encuentra en su parte inferior y que se desplaza con una velocidad constante u (u < v).

Responde a las siguientes cuestiones suponiendo que la noria se comporta como un mecanismo ideal, sin tener en cuenta ningún tipo de pérdidas energéticas.

1º Si llamamos tΔ al tiempo que dura el choque, la cantidad de agua mΔ que golpea al álabe es la contenida en el elemento de volumen xAΔ , como se muestra en la figura 5a. Como consecuencia de este proceso, la masa mΔ pasa a tener una velocidad horizontal u, igual a la del álabe (figura 5b).

Como aplicación del teorema del impulso y de la cantidad de movimiento, deduzca la expresión de la fuerza FΔ que el agua ejerce sobre el álabe en función de las velocidades u y v, del área A y de la densidad del agua ρ .

2º Determina el trabajo WΔ que realiza la fuerza FΔ sobre el álabe en el intervalo de tiempo tΔ .

3º Determina el rendimiento del proceso, definido como cE/W ΔΔη = , en donde cEΔ es la energía cinética de la masa mΔ contenida en el elemento de volumen xAΔ , que es la máxima energía que se puede transferir del agua al álabe.

4º El rendimiento anterior depende de la velocidad u del álabe. Determina el valor 0u de la velocidad del álabe para la cual dicho rendimiento es máximo. ¿Cuál es el valor del máximo rendimiento?

5º Si el radio de la noria es R, ¿con qué velocidad angular 0ω debe girar para que el rendimiento sea máximo?

6º La noria que se va a construir para la Expo2008, tiene un diámetro mR 5,162 = , la superficie efectiva de sus álabes planos es 250 m,A = y se pretende elevar un caudal de agua /sm1051 32−⋅= ,G . Determina la expresión analítica de la velocidad media de la corriente de agua, v, que provoca el giro de la noria, en condiciones de rendimiento máximo, y calcula su valor. Toma como valor de la aceleración de la gravedad: 2m/s89,g = .

Solución Problema 2

1º La variación de la componente horizontal del momento lineal de la masa mΔ contenida en el elemento de volumen xAΔ , durante el choque con el álabe es igual al impulso que éste ejerce sobre dicho elemento de volumen:

( )vumt'F −= ΔΔΔ

Como xAm ΔρΔ = y, en virtud del principio de acción y reacción, la fuerza FΔ que el agua ejerce sobre el álabe es igual a 'FΔ− , se tiene

( )uvxAtF −= ΔρΔΔ

Además, como tvx ΔΔ = se obtiene

( )uvvAF −= ρΔ

2º El trabajo WΔ que FΔ realiza sobre el álabe en el tiempo tΔ es

'xFW ΔΔΔ =

A A u

A A u v

“antes del choque” “después del choque”

Fig. 5 a Fig. 5 b

Page 87: XXIII olimpiadas españolas de física, enunciados y soluciones de las pruebas propuestas - pag 214.pdf

XIX OLIMPIADA ESPAÑOLA DE FÍSICA

Página 9 de 22

Como tu'x ΔΔ = es la distancia recorrida por el álabe en el tiempo que dura el choque, el trabajo viene dado por

( ) tuvvuAW ΔρΔ −=

3º Para el cálculo del rendimiento es necesario determinar la energía cinética de la masa mΔ . Esta energía es

32221

21

21 vtAvtvAvmEc ΔρΔρΔΔ ===

Por lo que el rendimiento es

22

v)uv(u −

4º El rendimiento será máximo para el valor u = u0 que haga 0=dudη

0=dudη ⇒ 02 =− ouv

Con lo que vu21

0 = y 50,=η

5º La velocidad angular de la noria esRu

=ω . Si el rendimiento ha de ser el máximo, la velocidad angular deberá ser

Rv

20 =ω

6º La energía disponible es el trabajo realizado sobre el álabe en condiciones de rendimiento máximo, 20 /vu =

tvAW uu ΔρΔ 341

0 ==

Para elevar una masa de agua MΔ a una altura h = 2R es necesario consumir una energía

gRME 2ΔΔ =

que podemos poner en función del caudal G recordando que tGM ΔρΔ = , con lo que

tgRGgRME ΔρΔΔ 22 ==

Igualando la energía disponible con la consumida (balance energético) se tiene

tgRGtvA ΔρΔρ 241 3 =

Que nos permite obtener la velocidad de la corriente de agua que provoca el giro de la noria en rendimiento máximo en función del caudal de agua elevada

318 /

AGgRv ⎟

⎠⎞

⎜⎝⎛=

Sustituyendo los valores dados en el enunciado, se obtiene el siguiente valor

km/h9,6m/s72 == ,v

Page 88: XXIII olimpiadas españolas de física, enunciados y soluciones de las pruebas propuestas - pag 214.pdf

XIX OLIMPIADA ESPAÑOLA DE FÍSICA

Página 10 de 22

Problema 2. Tabla de Respuestas

Apartado Expresiones analíticas Resultados numéricos Puntos

1 ( )uvvAF −= ρΔ 2

2 ( ) tuvvuAW ΔρΔ −= 2

3 22

v)uv(u −

=η 1

4 vu21

0 = 50,=η 1 + 0,5

5 Rv

20 =ω 0,5

6 318 /

AGgRv ⎟

⎠⎞

⎜⎝⎛= km/h9,6m/s72 == ,v 2 + 1

Page 89: XXIII olimpiadas españolas de física, enunciados y soluciones de las pruebas propuestas - pag 214.pdf

XIX OLIMPIADA ESPAÑOLA DE FÍSICA

Página 11 de 22

Problema 3. Paseando una espira por un campo magnético.

Una espira cuadrada de lado b, resistencia eléctrica R y masa m, está montada sobre un carrito de madera que puede moverse sobre una superficie horizontal con rozamiento despreciable. Un ciudadano empuja el carrito, como se muestra en la figura 1, haciendo que la espira atraviese la región sombreada en la figura, de longitud L = 2b, en la que existe un campo magnético B uniforme, perpendicular al plano de la espira y dirigido hacia adentro. El ciudadano tiene que esforzarse para que durante todo el recorrido la velocidad de la espira sea siempre la misma, V.

En la figura se han incluido unos ejes de coordenadas para tener bien definida la posición de la espira en todo momento. Considera como coordenada x la distancia del eje Y al borde derecho de la espira.

a) Determina el flujo magnético )(xΦ que atraviesa la espira en los siguientes rangos de x:

a1) ( )0≤xΦ a2) ( )bx ≤≤0Φ a3) ( )bxb 2≤≤Φ a4) ( )bxb 32 ≤≤Φ a5) ( )xb ≤3Φ

b) Haz una representación gráfica de )(xΦ en el rango bx −= a bx 4= .

c) Determina el valor absoluto de la fuerza electromotriz ε (x), inducida en la espira en los siguientes rangos de x:

c1) ε ( )0≤x c2) ε ( )bx ≤≤0 c3) ε ( )bxb 2≤≤ c4) ε ( )bxb 32 ≤≤ c5) ε ( )xb ≤3

d) Representa gráficamente la intensidad de la corriente inducida que circula por la espira en función de x. Considera que la intensidad es positiva cuando recorre la espira en el sentido contrario al de las agujas del reloj.

e) Determina la fuerza que el ciudadano tiene que ejercer sobre la espira para que su velocidad permanezca constante e igual a V en los siguientes rangos de x:

e1) ( )0≤xF e2) ( )bxF ≤≤0 e3) ( )bxbF 2≤≤ e4) ( )bxbF 32 ≤≤ e5) ( )xbF ≤3

Ahora la protagonista es una ciudadana que en vez de arrastrar el carrito con la espira, le da un empujón imprimiéndole una velocidad v0, como se muestra en la figura 2.

f) ¿Cuál es la mínima velocidad v0 que la ciudadana debe imprimir al carrito para que la espira entre completamente en la región del campo magnético?

En este último apartado, seguramente te serán útiles las siguientes relaciones:

vdxdv

dtdx

dxdv

dtdv

=⋅=

b

x = 0 x = 2b X

Fig. 1

B⊗ V

Y

b

x = 0 x = 2b X

Fig. 2

B⊗ 0v

Y

Page 90: XXIII olimpiadas españolas de física, enunciados y soluciones de las pruebas propuestas - pag 214.pdf

XIX OLIMPIADA ESPAÑOLA DE FÍSICA

Página 12 de 22

Solución Problema 3

a) El flujo de un campo magnético uniforme que atraviesa perpendicularmente una superficie de área S es: SB=Φ .

a1) ( ) 00 =≤xΦ

a2) ( ) xbBbx =≤≤0Φ

a3) ( ) 22 bBbxb =≤≤Φ

a4) ( ) ( )xbbBbxb −=≤≤Φ 332

a5) ( ) 03 =≤ xbΦ

b) La representación correspondiente es:

c) De acuerdo con la ley de Faraday-Lenz,

ε dtdSB

dtd

−=Φ

−=

el valor absoluto de la fem inducida en la espira es:

c1) ε ( ) 00 =≤x

c2) ε ( ) VbBbx =≤≤0

c3) ε ( ) 02 =≤≤ bxb

c4) ε ( ) VbBbxb =≤≤ 32

c5) ε ( ) 03 =≤ xb

d) La representación gráfica de la intensidad que circula por la espira (I = ε / R) , de acuerdo con el convenio de signos establecido en el enunciado, es:

( )xΦ

X

-b 0 b 2b 3b 4b

Bb2

( )xI

X

-b 0 b 2b 4b

RVbB

3b

RVbB

Page 91: XXIII olimpiadas españolas de física, enunciados y soluciones de las pruebas propuestas - pag 214.pdf

XIX OLIMPIADA ESPAÑOLA DE FÍSICA

Página 13 de 22

e) Cuando la espira está en 0≤x , bxb 2≤≤ o xb ≤3 la fuerza de interacción con el campo magnético es nula; sin embargo, tanto en bx ≤≤0 , como en bxb 32 ≤≤ la fuerza de Lorentz es IBbF = , horizontal y dirigida hacia la izquierda.

Por lo tanto, sólo en ambos tramos, para que la espira se mueva con velocidad constante V, es decir, con aceleración nula, el ciudadano deberá “esforzarse” aplicando una fuerza igual y de sentido contrario a Fm.

Como en dichos tramos el valor absoluto de la intensidad de la corriente inducida es

R

VbBI =

Resulta que la fuerza con la que el ciudadano deberá empujar es:

R

VbBF22

= “Horizontal y hacia la derecha”

f) Una vez que la ciudadana “lanza” el carrito con la espira, el movimiento es uniforme hasta que comience a entrar en el campo. A partir de ahí, la fuerza de Lorentz debida a la intensidad inducida, que actúa sobre el lado derecho de la espira, frenará su movimiento.

Si v es la velocidad de la espira cuando está a la distancia x, e I la intensidad de la corriente inducida en ese instante, el módulo de la fuerza de Lorentz vendrá dada por:

IbBfm =

siendo dt

dR

I Φ1=

Por otra parte,

xbB=Φ ⇒ vbBdtdxbB

dtd

==Φ ⇒ v

RbBfm

22=

Recordando que fm se opone a la velocidad, que “frena” la espira, en virtud de la segunda ley de Newton,

vRbB

dtdvm

22−=

Ahora es cuando viene bien tener en consideración la “ayuda” del final del apartado d ya que con ella se puede escribir,

vRmbBv

dxdv 22

−= ⇒ dxRmbBdv

22−=

Si 0v es la mínima velocidad con la que la ciudadana lanza el carrito para que entre enteramente en la región del campo

magnético y se pare, o lo que es lo mismo que v = 0 en x = b, integrando la anterior ecuación, se obtiene

∫∫ −=b

vdx

RmbBdv

0

220

0 ⇒

RmbBv

32

0 =

I V Fm B⊗ V Fm B⊗ I

Fig. 3 Fig. 4

Page 92: XXIII olimpiadas españolas de física, enunciados y soluciones de las pruebas propuestas - pag 214.pdf

XIX OLIMPIADA ESPAÑOLA DE FÍSICA

Página 14 de 22

Problema 3. Tabla de Respuestas

Apartado Respuestas Puntos

a

a1 a2 a3 a4 a5

( ) 00 =≤xΦ ( ) xbBbx =≤≤0Φ

( ) 22 bBbxb =≤≤Φ ( ) ( )xbbBbxb −=≤≤Φ 332 ( ) 03 =≤ xbΦ

0,25 0,50 0,50 0,50 0,25

b

( )xΦ

X

b 0 b 2b 3b 4b

Bb2

1,00

c

c1 c2 c3 c4

c5

ε ( ) 00 =≤x ε ( ) VbBbx =≤≤0 ε ( ) 02 =≤≤ bxb ε ( ) VbBbxb =≤≤ 32 ε ( ) 03 =≤ xb

0,25 0,50 0,50 0,50 0,25

d

( )xI

X

b 0 b 2b 4b

RVbB

3b

RVbB

1,00

e

e1

e2

e3

e4

e5

( ) 00 =≤xF

( )R

VbBbxF22

0 =≤≤ dirección horizontal y sentido hacia la derecha

( ) 02 =≤≤ bxbF

( )R

VbBbxbF22

32 =≤≤ dirección horizontal y sentido hacia la derecha

( ) 03 =≤ xbF

0,25

0,50

0,50

0,50

0,25

f

RmbBv

32

0 = 2,00

Page 93: XXIII olimpiadas españolas de física, enunciados y soluciones de las pruebas propuestas - pag 214.pdf

XIX OLIMPIADA ESPAÑOLA DE FÍSICA

Página 15 de 22

Prueba experimental. 20/50 puntos

ESTUDIO EXPERIMENTAL DE UN GENERADOR DE CORRIENTE

Introducción; objetivos Según la ley de Faraday, cuando cambia el flujo magnético a través de un circuito se induce en él una fuerza electromotriz (fem) proporcional a la velocidad de variación del flujo. Por ejemplo, si se hace girar una espira conductora frente a un imán, se induce una fem y por tanto circula corriente eléctrica por la espira. Un motor eléctrico contiene imanes permanentes y un bobinado conductor, de forma que, si se hace girar mediante algún dispositivo mecánico, funciona como generador eléctrico y es capaz de alimentar un circuito externo. Un porcentaje elevado de la energía mecánica suministrada se transforma en energía eléctrica, aunque hay pérdidas debidas a la fricción. En esta prueba experimental se propone estudiar las características de un pequeño motor de corriente continua que se hace funcionar como generador, haciéndolo girar mediante un hilo del que se cuelgan pesas. Materiales - Motor/generador de corriente continua. Resistencia interna (del bobinado), r = 8,8 Ω. - Listón de madera y brida para sujetar el motor. - Polea que se inserta a presión en el eje del motor. - Tijeras, hilo y clip. - Arandelas que se usan como pesas. Masa de las arandelas: grandes 11,6 g; pequeñas 5,9 g. - Pila de petaca. - Cinta métrica. - Cronómetro. - 2 resistencias de 15 Ω. - Multímetro. - Escuadra metálica y sargento. 3 pinzas metálicas. Montaje • El listón de madera se coloca vertical, apoyado en el borde de la mesa, y se sujeta mediante la escuadra, una

pinza y el sargento como indica la figura 1. • El hilo se ata al borde de la polea de plástico como muestra la figura 2. En el extremo inferior del hilo se ata el

gancho para colgar las pesas, que se construye doblando el clip. La longitud del hilo debe ser suficiente para que las pesas puedan llegar al suelo, después de descender 1,2 m aproximadamente. La polea se inserta a presión en el eje del motor, de forma que quede bien fijada.

• El motor se sujeta en la parte alta del listón mediante una brida que, a su vez, se sujeta mediante dos pinzas (figura 3). El eje del motor debe quedar horizontal, con el hilo colgando por el exterior de la mesa.

Fig. 1 Fig. 2 Fig. 3

R

Page 94: XXIII olimpiadas españolas de física, enunciados y soluciones de las pruebas propuestas - pag 214.pdf

XIX OLIMPIADA ESPAÑOLA DE FÍSICA

Página 16 de 22

• Los terminales del generador se conectan a una resistencia R = 7,5 Ω, (dos resistencias de 15 Ω en paralelo).

Para realizar la conexión basta con enrollar los cables del generador a las patas de la resistencia. • Para medir la caída de potencial en la resistencia, se conecta en paralelo el multímetro, en la escala de 2 V de

tensión continua. • En total (figura 4), el motor accionado mediante las pesas es un generador

de fem ε y resistencia interna r = 8,8 Ω, que alimenta una resistencia externa R = 7,5 Ω, en la que se mide la tensión VΔ con un voltímetro.

• Para medir la velocidad uniforme de caída de las pesas, basta cronometrar el tiempo que tardan en recorrer una distancia de, por ejemplo, un metro, entre una marca realizada a lápiz en el listón y el suelo.

• Enrollar el hilo en la polea a mano es tedioso. Es más rápido y práctico alimentar el motor con la pila de petaca, tocando con sus terminales algo abiertos las patas de la resistencia.

• Comprobaciones, ajustes y pruebas preliminares: - Verifica el correcto funcionamiento del cronómetro y del voltímetro. - Es necesario que el eje del motor esté horizontal, de modo que, cuando se enrolla el hilo en la polea al

alimentar con la pila, las vueltas de hilo queden distribuidas uniformemente y no se superpongan en exceso.

- Empleando dos o tres arandelas como pesas se puede comprobar el funcionamiento del dispositivo y adquirir práctica en las medidas de VΔ y del tiempo de caída de las pesas.

Modelo teórico Cuando las masas colgadas hacen girar la polea unida al generador, un bobinado compuesto por muchas espiras gira en el seno del campo magnético producido por unos imanes. Al aplicar la ley de Faraday se obtiene una fem inducida ε proporcional a la velocidad angular de rotación, ω. Con una geometría adecuada se consigue además que esta fem sea aproximadamente constante. En nuestro montaje, la ω de rotación del bobinado es a su vez proporcional a la velocidad v con que desciende la masa colgada, siendo la relación entre ambas el radio de la polea. En total, es de esperar una proporcionalidad directa entre ε y v.

vk=ε (1)

donde k es una constante característica del generador, esencialmente dependiente del radio de la polea, de la geometría del bobinado y de la intensidad del campo magnético de los imanes. Idealmente, el generador transforma energía mecánica (energía potencial gravitatoria de la masa colgada) en energía eléctrica, que se disipa en la resistencia del circuito (efecto Joule). Cuando la masa m desciende con velocidad v, la potencia aportada por la fuerza gravitatoria es directamente proporcional a esta velocidad

vgmPmec =

Sin embargo, la potencia eléctrica disipada en el circuito crece con el cuadrado de la velocidad

222v

rRk

rRIPelec +

=+

== εε

La masa m empieza a caer partiendo del reposo. En los primeros instantes su velocidad es creciente ya que la Pmec suministrada es superior a la Pelec disipada, y la diferencia entre ambas se convierte en energía cinética de m, es decir en un aumento de v. Como Pelec crece con v2 y Pmec con v, el incremento de velocidad es cada vez menor hasta que, al cabo de un tiempo suficientemente largo, se igualan Pelec y Pmec y m desciende con velocidad uniforme1 (velocidad límite). En estas circunstancias, se cumple

elecmec PP = → 22v

rRkvgm

+= →

( )m

k

rRgv

2

+=

1 En nuestro montaje la velocidad límite se alcanza al poco de soltar m, como es fácil comprobar experimentalmente.

VΔr

ε gene

rado

r

R I

Fig. 4

Page 95: XXIII olimpiadas españolas de física, enunciados y soluciones de las pruebas propuestas - pag 214.pdf

XIX OLIMPIADA ESPAÑOLA DE FÍSICA

Página 17 de 22

En un generador real existe además una fricción mecánica que frena la rotación del sistema. Para vencer esta fricción es necesario que la masa colgada supere un cierto valor mínimo 0m , característico de cada generador2. La potencia mecánica asociada a esta masa, vgm 0 , se pierde en calentamiento del sistema por fricción. En consecuencia, un balance energético más realista conduce a una velocidad límite de descenso proporcional a la masa eficaz, 0mm − .

( ) ( )02

mmk

rRgv −

+= (2)

Uno de los objetivos de esta prueba experimental es comprobar este modelo teórico, y en concreto determinar los valores de las constantes características del generador, k y 0m .

Medidas y preguntas

1) Cuelga sucesivamente del hilo las arandelas necesarias para que los valores de la masa m sean los indicados en la primera columna de la tabla 1 de la hoja de respuestas. Para cada valor de m, mide y anota en la columna adecuada de la tabla 1: • El tiempo que tarda m en descender una altura h = 1,00 m. Es conveniente repetir tres veces la medida (t1, t2

y t3) y calcular el valor medio, t . • La caída de potencial VΔ en la resistencia R. Como su valor puede variar ligeramente durante el descenso

de m, es conveniente anotar los valores máximo y mínimo observados y tomar el promedio, VΔ , para cálculos posteriores.

2) A partir de las medidas anteriores, calcula para cada m, y anota en las columnas correspondientes de la tabla 2, la velocidad v de la masa, la intensidad I que circula por el circuito y la fem ε del generador.

3) Aplicando (1), calcula en cada caso el valor de la constante k del generador y anótalo en la última columna de la tabla 2. Calcula el valor medio de k y haz una estimación de su incertidumbre (margen de error), kΔ .

4) Representa gráficamente en el papel milimetrado los puntos experimentales (x, y) = (m, v).

5) Ajusta estos puntos a una línea recta.

6) Deduce de este ajuste los valores de las dos constantes características del generador3, k y 0m .

7) Haz una estimación de las incertidumbres de estas dos constantes.

8) Deduce la expresión analítica de la intensidad que circula por el circuito, I, en función de las constantes internas del generador (k y 0m ), y de los parámetros externos (m y R). En particular, ¿cómo depende I de R?

9) Mediante las medidas que consideres oportunas, comprueba experimentalmente la dependencia prevista de I con R. Explica detalladamente las medidas que has realizado y preséntalas en la tabla 3 de la hoja de respuestas. Discute tus resultados.

Sugerencia: trabaja con una masa suspendida fija, por ejemplo m = 23,2 g.

2 La fuerza de rozamiento dinámica (cuando hay deslizamiento relativo) es inferior a la fuerza de rozamiento estática máxima

(límite de equilibrio, sin deslizamiento). Por ello, en nuestro sistema, la masa que hay que colgar para que el generador empiece a girar partiendo del reposo puede ser apreciablemente superior a m0.

3 Puede haber discrepancias del orden del 5% entre los valores de k obtenidos en los apartados 3 y 6, debidas principalmente a la dispersión de los valores reales de R+r en los diversos montajes experimentales. Para cálculos posteriores, es preferible tomar el valor de k obtenido en el apartado 6.

Page 96: XXIII olimpiadas españolas de física, enunciados y soluciones de las pruebas propuestas - pag 214.pdf

XIX OLIMPIADA ESPAÑOLA DE FÍSICA

Página 18 de 22

Solución Prueba experimental. 1) Ejemplo de medidas reales:

Tabla 1

2) Aplicando thv /= , RVI /Δ= y ( )IrR +=ε se obtiene

Tabla 2

3) En la última columna de la tabla anterior se presentan los valores de la constante vk /ε= obtenidos en cada caso. El valor medio es

Vs/m 4,71=k

Una estimación razonable para la incertidumbre de esta constante, obtenida como promedio de n = 8 determinaciones, es su error típico4

( )

212

1

/

)( ⎥⎥

⎢⎢

−=Δ

∑nn

kkk

i

Se obtiene

Vs/m 0,04=Δk

4 Si se toma como incertidumbre el error típico, el nivel de confianza es inferior al 65% pues se han realizado menos de 10

medidas. Para aumentar este nivel al 95% habría que duplicar con creces este margen de error. Sin necesidad de buscar en unas tablas de error, un resultado razonable con aproximadamente el 95% de confianza sería ( )Vs/m1074 ,, ±=k

m (g) t1 (s) t2 (s) t3 (s) t (s) maxVΔ (V) minVΔ (V) VΔ (V)

17, 5 14,40 14,37 14,44 14,40 0,15 0,15 0,15 23,2 8,72 8,75 8,72 8,73 0,25 0,23 0,24 29,1 6,90 6,40 6,50 6,60 0,34 0,32 0,33 34,8 5,37 5,34 5,28 5,33 0,42 0,39 0,405 40,7 4,56 4,29 4,53 4,46 0,52 0,49 0,505 46,4 3,84 3,62 3,69 3,72 0,59 0,57 0,58 52,3 3,22 3,28 3,28 3,26 0,68 0,66 0,67 58,0 2,82 2,88 2,78 2,83 0,75 0,73 0,74

m (g) v (m/s) I (A) ε (V) k (Vs/m)

17, 5 0,0694 0,020 0,33 4,76 23,2 0,115 0,032 0,52 4,54 29,1 0,152 0,044 0,72 4,75 34,8 0,188 0,054 0,89 4,74 40,7 0,224 0,067 1,09 4,86 46,4 0,269 0,077 1,26 4,68 52,3 0,307 0,089 1,46 4,76 58,0 0,354 0,099 1,61 4,55

Page 97: XXIII olimpiadas españolas de física, enunciados y soluciones de las pruebas propuestas - pag 214.pdf

XIX OLIMPIADA ESPAÑOLA DE FÍSICA

Página 19 de 22

4) Se presenta la gráfica pedida, con un aspecto similar al que tendría dibujada en papel milimetrado.

5) En la gráfica anterior también se ha trazado la recta que más aproximadamente pasa por los puntos experimentales. La pendiente y la ordenada en el origen pueden calcularse con buena precisión a partir de dos puntos alejados de esta recta. Pueden tomarse, por ejemplo, los puntos extremos de dicha recta, de coordenadas

( ) ( )05000011 ,;,; −=yx , ( ) ( )362006022 ,;,; =yx

Por tanto, la pendiente, p, y la ordenada en el origen, c, de la recta son5

g060m/s4120

12

12,

,=

−=

xx

yyp → 11 kgsm876 −−⋅= ,p

1yc = → m/s0500,−=c 6) Según la ecuación (2) del enunciado, la pendiente y la ordenada en el origen de la gráfica anterior son,

respectivamente,

( )

2k

rRgp

+= , 0mpc −=

Por tanto, las constantes k y 0m del generador pueden deducirse en la forma

( ) 21 /

⎟⎠⎞

⎜⎝⎛ +

=p

rRgk ,

pcm −=0 (3)

Se obtiene Vs/m824,=k , g370 ,=m

5 Aplicando el método de mínimos cuadrados se obtiene p = 6,866 m·s-1kg-1 y c = -0,0496 m/s.

-0,05

0,00

0,05

0,10

0,15

0,20

0,25

0,30

0,35

0 5 10 15 20 25 30 35 40 45 50 55

v (m/s)

m (g)

0,40

-0,1060

Page 98: XXIII olimpiadas españolas de física, enunciados y soluciones de las pruebas propuestas - pag 214.pdf

XIX OLIMPIADA ESPAÑOLA DE FÍSICA

Página 20 de 22

7) Las incertidumbres de la pendiente y de la ordenada en el origen de la gráfica v(m) pueden estimarse trazando las rectas que con pendientes máxima y mínima se ajustan razonablemente a los puntos experimentales, dentro de su margen de incertidumbre. En nuestro caso, la dispersión de los puntos experimentales respecto a la recta óptima es pequeña, del orden del radio de los puntos dibujados. Teniendo en cuenta, además, que ambas rectas deben pasar por el punto promedio de los experimentales, ( )vm , , en la siguiente gráfica se presenta una estimación razonable de estas rectas.

Observando sobre la gráfica las coordenadas de los dos puntos extremos de cada una de estas dos rectas, como en el apartado 5, se obtienen sus pendientes y sus ordenadas en el origen

( ) 11- kgsm007

g060m/s05503650 −⋅=

+= ,

,

,,maxp m/s0550 ,min −=c

( ) 11- kgsm736

g060m/s04503590 −⋅=

+= ,

,

,,minp m/s0450 ,max −=c

Por tanto, una estimación de las incertidumbres de p y c podría ser6

1-1kgsm130 −⋅=Δ ,p m/s0050,=Δc

Los correspondientes valores máximo y mínimo de las constantes k y 0m se obtienen aplicando (3) con los valores de p y c ajustados para cada una de estas dos rectas

( )

Vs/m87421

,/

minmax =⎟⎟

⎞⎜⎜⎝

⎛ +=

p

rRgk , g970 ,

max

minmax, =−=

p

cm

( )

Vs/m78421

,/

maxmin =⎟⎟

⎞⎜⎜⎝

⎛ +=

p

rRgk , g760 ,

min

maxmin, =−=

p

cm

6 Un cálculo estadístico conduce a que los errores típicos de p y c son, respectivamente, 0,11 m·s-1kg-1 y 0,004 m/s.

-0,05

0,00

0,05

0,10

0,15

0,20

0,25

0,30

0,35

0 5 10 15 20 25 30 35 40 45 50 55

v (m/s)

m (g)

0,40

-0,1060

Page 99: XXIII olimpiadas españolas de física, enunciados y soluciones de las pruebas propuestas - pag 214.pdf

XIX OLIMPIADA ESPAÑOLA DE FÍSICA

Página 21 de 22

Los valores máximo y mínimo estimados para 0m también pueden leerse directamente en la gráfica anterior, pues, teniendo en cuenta la ecuación (2) del enunciado, corresponden a las intersecciones de las dos rectas con la ordenada v = 0.

En total, los resultados para k y 0m , incluyendo una estimación de sus incertidumbres, serían

( ) Vs/m050824 ,, ±=k ( ) g60370 ,, ±=m

Nótese que las incertidumbres relativas de k y 0m , son del orden del 1% y del 8%, respectivamente, por lo que la principal fuente de error para cálculos posteriores es la incertidumbre de 0m .

8) La intensidad que circula en régimen estacionario por el circuito es, teniendo en cuenta la ley de Ohm y las

ecuaciones (1) y (2) del enunciado

( ) ( )02

mmk

rRg

rRk

rR

vk

rRI −

++

=+

=+

= ε

Simplificando, queda

( )

k

gmmI 0−

= (4)

Esta corriente no depende de la resistencia del circuito. Por ello, tal y como indica el título de esta prueba experimental, el sistema opera como un generador de corriente. Nótese que la constante k del generador describe la relación entre el peso motor efectivo y la intensidad que suministra el generador, por lo que en el SI las unidades de k podrían expresarse como N/A.

9) En particular, para m = 23,2 g la ecuación (4) prevé una corriente

I = 32,4 mA

Propagando las incertidumbres de k y 0m obtenidas en el apartado 7, se obtiene una incertidumbre para esta corriente

21,=ΔI mA.

Como se dispone de dos resistencias de 15 Ω, puede trabajarse experimentalmente con tres valores de la resistencia de carga del generador: R1 = 7,5 Ω si se conectan en paralelo, R2 = 15 Ω si se conecta una sola resistencia y R3 = 30 Ω si se conectan en serie. En la tabla 3 se presentan medidas de la caída de potencial ΔV en cada caso y de la intensidad deducida como I = ΔV/R. También podría medirse directamente la intensidad usando el múltímetro como amperímetro, conectado en serie con la resistencia y el generador.

El acuerdo entre las previsiones del modelo y los resultados experimentales es correcto, dentro del margen de incertidumbre estimado.

Tabla 3

R (Ω) maxVΔ (V) minVΔ (V) VΔ (V) I (mA)

7,5 0,25 0,23 0,24 32 15 0,52 0,48 0,50 33 30 1,01 0,97 0,99 33

Page 100: XXIII olimpiadas españolas de física, enunciados y soluciones de las pruebas propuestas - pag 214.pdf

XIX OLIMPIADA ESPAÑOLA DE FÍSICA

Página 22 de 22

Prueba experimental. Hoja de respuestas 1) Tabla 1 (3 puntos)

2) Tabla 2 (2 puntos)

3) Vs/m 4,71=k Vs/m 0,04=Δk (2 puntos) 4) Gráfica en papel milimetrado adjunto. (2 puntos)

5) Parámetros de la recta ajustada: 6) Vs/m824,=k g370 ,=m (2 puntos)

7) Vs/m050 ,=Δk g600 ,=Δm (2 puntos)

8) Expresión analítica: I(k, 0m , m, R) =( )

( )Rfk

gmm≠

− 0 (2 puntos)

9) Tabla 3 (3 puntos)

m (g) t1 (s) t2 (s) t3 (s) t (s) maxVΔ (V) minVΔ (V) VΔ (V)

17, 5 14,40 14,37 14,44 14,40 0,15 0,15 0,15 23,2 8,72 8,75 8,72 8,73 0,25 0,23 0,24 29,1 6,90 6,40 6,50 6,60 0,34 0,32 0,33 34,8 5,37 5,34 5,28 5,33 0,42 0,39 0,405 40,7 4,56 4,29 4,53 4,46 0,52 0,49 0,505 46,4 3,84 3,62 3,69 3,72 0,59 0,57 0,58 52,3 3,22 3,28 3,28 3,26 0,68 0,66 0,67 58,0 2,82 2,88 2,78 2,83 0,75 0,73 0,74

m (g) v (m/s) I (A) ε (V) k (Vs/m)

17, 5 0,0694 0,020 0,33 4,76 23,2 0,115 0,032 0,52 4,54 29,1 0,152 0,044 0,72 4,75 34,8 0,188 0,054 0,89 4,74 40,7 0,224 0,067 1,09 4,86 46,4 0,269 0,077 1,26 4,68 52,3 0,307 0,089 1,46 4,76 58,0 0,354 0,099 1,61 4,55

R (Ω) maxVΔ (V) minVΔ (V) VΔ (V) I (mA)

7,5 0,25 0,23 0,24 32 15 0,52 0,48 0,50 33 30 1,01 0,97 0,99 33

Pendiente = 11 kgsm876 −−⋅, Ordenada en el origen = m/s0500,−

(2 puntos)

Page 101: XXIII olimpiadas españolas de física, enunciados y soluciones de las pruebas propuestas - pag 214.pdf

EJERCICIOS PROPUESTOS Y SOLUCIONES

R.S.E.F.

Page 102: XXIII olimpiadas españolas de física, enunciados y soluciones de las pruebas propuestas - pag 214.pdf

Página 1 de 18

XVIII OLIMPIADA ESPAÑOLA DE FÍSICA. JAÉN 2007R.S.E.F.

Prueba teórica 1. Feel free, feel zero-g! 10/50 Puntos

El título de este problema hace alusión a la campaña que, desde hace algunos años, promueve la Agencia Espacial Europea (ESA) y que permite que grupos de jóvenes estudiantes europeos realicen experimentos diseñados por ellos mismos en condiciones de gravedad aparente nula. La foto de la figura 1 corresponde a un grupo de la Universidad de Zaragoza en la campaña 2006, a bordo de un avión Airbus A300 preparado para tal fin (figura 2).

La descripción de este tipo de vuelos, comúnmente denominados “parabólicos”, se representa esquemáticamente en la figura 3 y es la siguiente: en un principio el avión vuela horizontalmente a su velocidad máxima hasta un punto A. Después se eleva, y cuando alcanza con un ángulo de 45º una altura, hB ≈ 25.000 ft1 sobre el nivel del mar (punto B), reduce la potencia de los motores hasta un mínimo suficiente para contrarrestar la disipación de energía producida por la resistencia del aire. En esta primera fase del vuelo AB, que dura un tiempo tAB = 20 s, los pasajeros sienten que su “peso” casi se duplica. A partir de B el vuelo puede considerarse libre, ¡feel free! y, por tanto, la trayectoria que describe es parabólica (de ahí el nombre que reciben estos vuelos). El vértice de la parábola (punto C) se encuentra a una altura, hC ≈ 28.000 ft. Posteriormente, ya iniciado el descenso del avión, en el punto D, situado a una altura similar a la de B, se incrementa de nuevo la potencia de los motores para permitir que en el punto E el aparato recupere el vuelo horizontal. Durante el trayecto B-C-D tanto los pasajeros como la carga transportada se encuentran como si la gravedad se hubiese anulado, ¡feel zero-g! Sin embargo, durante el trayecto DE, cuya duración es también análoga a la del trayecto AB, sienten de nuevo que su peso casi se duplica.

1 En aeronáutica se usan asombrosamente las unidades anglosajonas ( imperial units). La equivalencia del

pie, “foot” o abreviadamente ft es 1ft = 0,30480 m

Fig. 1 Fig. 2

Feel free, Feel zero-g! Student Parabolic Flight

Page 103: XXIII olimpiadas españolas de física, enunciados y soluciones de las pruebas propuestas - pag 214.pdf

Página 2 de 18

XVIII OLIMPIADA ESPAÑOLA DE FÍSICA. JAÉN 2007R.S.E.F.

Estas maniobras se repiten 30 veces en cada vuelo, que tiene una duración total de unas dos horas, brindando la oportunidad de realizar interesantes experiencias en ingravidez, imposibles de conseguir en laboratorios en Tierra.

El concepto de “gravedad aparente” al que antes se ha hecho referencia, requiere cierta explicación. Por esta razón, antes de plantear las cuestiones relativas al problema del “vuelo parabólico”, se propone resolver el siguiente ejercicio:

Del extremo inferior de un dinamómetro sujeto al techo de un ascensor se suspende un cuerpo de masa m = 1 kg. Como la escala del dinamómetro nos indica, en newtons, la fuerza que el resorte ejerce sobre la masa suspendida, cuando el ascensor está en reposo la indicación numérica de dicha escala coincidirá con el valor numérico de la aceleración de la gravedad.

Más en general, la indicación en la escala del dinamómetro cuando la masa suspendida de su extremo es m = 1 kg, nos proporciona el valor numérico de lo que se denomina gravedad aparente, ga.

Según esto, ¿cuál es la gravedad aparente en los siguientes casos: A1) Ascensor que, partiendo del reposo, inicia un movimiento de subida con aceleración constante a. A2) Ascensor que, moviéndose hacia arriba, frena con aceleración constante a. A3) Ascensor que, partiendo del reposo, inicia un movimiento de bajada con aceleración constante a. A4) Ascensor que, moviéndose hacia abajo, frena con aceleración constante a.

Con referencia al “vuelo parabólico”, deduce las expresiones analíticas y estima los valores correspondientes de las siguientes magnitudes: B1) La velocidad del avión en el punto B, vB. B2) Los valores de la gravedad aparente media, gAB y gDE, en los trayectos AB y DE,

respectivamente. B3) El tiempo del que disponen los estudiantes para realizar sus experiencias con

gravedad aparente nula en cada maniobra. Nota: Considera que el valor de la aceleración de la gravedad en puntos próximos a la superficie terrestre es g0 = 9,81 m/s2.

Solución

Ejercicio previo La escala del dinamómetro marca la fuerza (elástica), Fe, que el resorte ejerce sobre

la masa suspendida m, cuyo peso es mg0. A1) Cuando el ascensor sube con aceleración constante a, mamgFe =− 0

Page 104: XXIII olimpiadas españolas de física, enunciados y soluciones de las pruebas propuestas - pag 214.pdf

Página 3 de 18

XVIII OLIMPIADA ESPAÑOLA DE FÍSICA. JAÉN 2007R.S.E.F.

Como m = 1 kg, la indicación numérica de la escala del dinamómetro nos dará el valor de la gravedad aparente. Por tanto,

agm/Fg ea +== 0 (1)

A2) Cuando el ascensor sube y frena con aceleración constante a, ( )ammgFe −=− 0

Luego: agm/Fg ea −== 0 (2)

A3) Cuando el ascensor baja con aceleración constante a, ( )ammgFe −=− 0

Luego: agm/Fg ea −== 0 (3)

A4) Cuando el ascensor baja y frena con aceleración constante a, mamgFe =− 0

Por tanto, agm/Fg ea +== 0 (4)

Ejercicio del “vuelo parabólico” B1) El trayecto del avión desde A hasta B, en el que

gana una altura AB hh − , se realiza a costa de la potencia que generan sus motores. Por ello, en este trayecto no se conserva la energía mecánica y en consecuencia no se puede deducir la velocidad en B a partir de la velocidad en A que, por cierto, ni siquiera es un dato del problema. Sin embargo, y para todos los efectos, en el punto B es como si el avión parara sus motores e iniciara un vuelo libre, con una velocidad inicial vB que forma un ángulo ϕ con la horizontal (el bien conocido “tiro oblicuo”), como se muestra en la figura 2. Omitiendo detalles y comentarios, las expresiones básicas del tiro oblicuo son las siguientes: Componentes de la aceleración del “proyectil” (en nuestro caso, del propio avión):

0

0x

y

aa g

=⎧⎨ = −⎩

Componentes de la velocidad: 0

cossen

x B

y B

v vv v g t

ϕϕ

=⎧⎨ = −⎩

Trayectoria: ( )

( ) 20

cos1sen2

B

B

x t v t

y t v t g t

ϕ

ϕ

=⎧⎪⎨

= −⎪⎩

En el punto C la velocidad del avión es horizontal, 0yv = . Si tBC es el tiempo que dura el vuelo entre B y C, tendremos,

xB

C

Fig. 2

y

vB

ϕ h

g

Page 105: XXIII olimpiadas españolas de física, enunciados y soluciones de las pruebas propuestas - pag 214.pdf

Página 4 de 18

XVIII OLIMPIADA ESPAÑOLA DE FÍSICA. JAÉN 2007R.S.E.F.

0

seng

vt B

BCϕ

= (5)

Como ( )BCy t h= , siendo C Bh h h= − , resulta

02senB

g hv

ϕ=

Según el enunciado, h = hC - hB = 3,0·103 ft = 910 m, ϕ = 45º y g0 = 9,81 m/s2 ⇒

190m/s 680km/hBv = =

B2) La componente vertical de la aceleración media del avión en el trayecto AB es el cociente entre la diferencia de las componentes verticales de la velocidad en B y en A (que es nula) y el tiempo de vuelo, tAB = 20 s, entre A y B,

senBAB

AB

vat

ϕ= ⇒ 0

2 680m/s76 g,,aAB ==

Por lo que, teniendo en cuenta (1), la gravedad aparente en el avión será

0AB ABg g a= + ⇒ 0senB

ABAB

vg gt

ϕ= + ⇒ 071 g,g AB =

En el descenso DE del avión la aceleración vertical media es el cociente entre la diferencia de las componentes verticales de la velocidad en E (que es nula) y en D, que por simetría es la misma que en B pero en sentido contrario, dividido por el tiempo de vuelo entre ambos puntos que, según el enunciado, es prácticamente el mismo que entre A y B. Por lo tanto,

AB

BDE t

va ϕsen= ⇒ 0

2 680m/s76 g,,aDE ==

De acuerdo con la expresión (4), la gravedad aparente en DE es

DEDE agg += 0 ⇒ 0senB

DEAB

vg gt

ϕ= + ⇒ 071 g,gDE =

B3) Durante el vuelo libre parabólico, trayecto BCD, la aceleración del avión es – g0, por lo que la gravedad aparente es nula

( )0 0 0BCDg g g= + − =

En este trayecto el interior del avión se convierte en un recinto que permite experimentar en estado de ingravidez durante un tiempo, tg=0 , doble del que trascurre entre B y C, dado en (5). En definitiva,

BCg tt 20 == ⇒ 00

sen2 B

g

vt

= = ⇒ 0 27sgt = =

Page 106: XXIII olimpiadas españolas de física, enunciados y soluciones de las pruebas propuestas - pag 214.pdf

Página 5 de 18

XVIII OLIMPIADA ESPAÑOLA DE FÍSICA. JAÉN 2007R.S.E.F.

Prueba teórica 2. Un modelo de molécula de HCl 10/50 Puntos

La molécula de cloruro de hidrógeno está formada por los iones, Cl– y H+. Como la masa del primero es mucho mayor que la del segundo, podemos adoptar como modelo sencillo que el Cl–

está en reposo en x = 0 y que el H+ puede moverse a lo largo del eje X, tal como se representa en la figura 1a.

Consideraremos que el HCl está en estado gaseoso para que cualquiera de sus moléculas esté poco perturbada por la presencia de otras vecinas. En estas condiciones, el Cl– de una molécula

ejerce una fuerza de atracción electrostática sobre el H+. Pero para que el sistema (la molécula de HCl) permanezca en equilibrio con los iones separados una distancia xe, (fig. 1b), es necesario además que sobre el H+ se ejerza una fuerza de repulsión, muy fuerte a distancias cortas y que decrezca rápidamente a distancias interiónicas grandes comparadas con la de equilibrio.

Por ello, debido a la presencia del Cl–, el H+ tiene una energía potencial2 que viene dada por la siguiente expresión:

( )2

9

e BU x kx x

= − +

donde 9 2 2

0

1 8,987 10 N m /C4

kπε

= = ⋅ , 191,602 10 Ce −= ⋅ y B es

una constante positiva. Esta energía potencial se representa en la figura 2.

1) Teniendo en cuenta que en el equilibrio la fuerza sobre el H+ debe ser nula, determina la distancia de equilibrio xe.

2) Deduce que la expresión de la energía potencial, U(x), puede escribirse en función de xe de la siguiente forma

( )8

29

19

exU x kex x

⎛ ⎞= − +⎜ ⎟

⎝ ⎠ (1)

El ión H+ no está nunca en reposo en la posición de equilibrio ex x= . Para “entretenerse” realiza pequeñas oscilaciones en torno a xe. Esto significa que cuando la distancia de separación es ex x≈ se comporta como si el H+, de masa mH, estuviese unido a un muelle de constante K y, por lo tanto, su energía potencial (elástica) sería

2 Recuerda que la diferencia de energía potencial se define a partir de la expresión dUdxFdW −== .

xe

x = 0 x

Cl- H+

HCl

U

x

X

Fig. 1a

Fig. 1b

Fig. 2

X

Page 107: XXIII olimpiadas españolas de física, enunciados y soluciones de las pruebas propuestas - pag 214.pdf

Página 6 de 18

XVIII OLIMPIADA ESPAÑOLA DE FÍSICA. JAÉN 2007R.S.E.F.

( ) ( )212 eU x K x x cte= − + (2)

En consecuencia, la expresión de la energía potencial (1) deberá coincidir con (2) cuando exx ≈ .

3) Realiza un desarrollo en serie de Taylor3 para U(x) en torno a exx = , con las aproximaciones que estimes oportuno, y deduce el valor de la constante K de la energía potencial elástica (2).

4) Determina la frecuencia de oscilación, f , del H+ en torno a su posición de equilibrio. 5) Experimentalmente se encuentra que f = 8,66·1013 Hz. Sabiendo que la masa del H+

es 27H 1,67 10 kgm −= ⋅ , calcula el valor de xe.

6) Determina y calcula la energía, Wd, que hay que aportar a un mol de HCl para separar completamente los iones de cada una de sus moléculas (Energía de disociación). El número de Avogadro es 236,022 10AN = ⋅ moléculas/mol.

Solución

1) Si la energía potencial del H+ es ( )2

9

e BU x kx x

= − + , la fuerza que actúa sobre él es

( ) 22 10

1 9dU BF x kedx x x

= − = − +

La posición de equilibrio cumple ( ) 0dUF xdx

= − = , es decir 1/8

2

9e

Bxke

⎛ ⎞= ⎜ ⎟⎝ ⎠

2) Despejando B en la expresión anterior y sustituyendo en la expresión de U(x),

( )8

29

19

exU x kex x

⎛ ⎞= − +⎜ ⎟

⎝ ⎠, como se trataba de demostrar.

3) Empleando la expresión del desarrollo de Taylor que se indica en la nota a pie de página, el desarrollo de U(x) resulta

( ) ( ) ( ) ( )2

22

12

e e

e e ex x

dU d UU x U x x x x xdx dx

⎛ ⎞⎛ ⎞= + − + −⎜ ⎟⎜ ⎟⎝ ⎠ ⎝ ⎠

+…

3 El desarrollo en serie de Taylor de una función f(x) en torno a un punto x0 es una importantísima

herramienta matemática. El desarrollo es:

( ) ( )

0!1

10

xxdxdfxfxf

=⎟⎠⎞

⎜⎝⎛+= ( )

02

2

0 !21

xxdxfdxx

=⎟⎟⎠

⎞⎜⎜⎝

⎛+− ( ) ...xx +− 2

0

Page 108: XXIII olimpiadas españolas de física, enunciados y soluciones de las pruebas propuestas - pag 214.pdf

Página 7 de 18

XVIII OLIMPIADA ESPAÑOLA DE FÍSICA. JAÉN 2007R.S.E.F.

Como ( ) 0/ =exdxdU , el primer término del desarrollo es el cuadrático en x- xe. Los

siguientes dependen de potencias crecientes por lo que son despreciables frente al anterior, dado que 0ex x− ≈ , por lo que

( ) ( ) ( )2

22

12

e

e ex

d UU x U x x xdx

⎛ ⎞= + −⎜ ⎟

⎝ ⎠

Comparando con la expresión (1) de la energía potencial elástica

=K2 8

22 3 11

2 10 ed U xkedx x x

⎛ ⎞= − +⎜ ⎟

⎝ ⎠ ⇒

2 2

2 3

8

e ex

d U kedx x

⎛ ⎞=⎜ ⎟

⎝ ⎠ ⇒

2

3

8

e

keKx

=

4) Si K es la constante del muelle equivalente, la frecuencia angular de las oscilaciones de una masa m es mK /=ω . En nuestro caso m es la masa del ión hidrógeno, mH, luego la frecuencia de las oscilaciones será

Hm

Kf π21= ⇒

2

3H

1 82 e

kefm xπ

=

5) Despejando xe de la expresión anterior,

1/ 32

2 2H

2e

kexf mπ

⎛ ⎞= ⎜ ⎟

⎝ ⎠ ⇒ nm155,0m1055,1 10 =⋅= −

ex

Este resultado es coherente con el valor de la longitud de enlace4 del HCl, 101,2746 10 mex −= ⋅ .

6) En la figura 2 del enunciado se pone de manifiesto que la energía potencial del ión hidrógeno sólo será nula cuando se encuentre muy alejado del origen, en el que se supone que está el ión Cl–. Esto equivale a decir “matemáticamente”, que será nula cuando la distancia tienda a infinito. Por tanto, la energía wd que es preciso aportar a la molécula es igual a ( )eU x− , es decir

( )8 2

29

1 89 9

ed e

e e e

x kew U x kex x x

⎛ ⎞= − = − − + =⎜ ⎟

⎝ ⎠ ⇒ 181,32 10 Jdw −= ⋅

Luego, para disociar un mol de HCl, la energía necesaria será,

d A dW N w= ⇒ 28

9d Ae

keW Nx

= ⇒ 27,96 10 kJ/moldW = ⋅

4 CRC Handbook of Chemistry and Physics. 83rd Edition. 2002.

Page 109: XXIII olimpiadas españolas de física, enunciados y soluciones de las pruebas propuestas - pag 214.pdf

Página 8 de 18

XVIII OLIMPIADA ESPAÑOLA DE FÍSICA. JAÉN 2007R.S.E.F.

Prueba teórica 3. Un prototipo de “gato” termodinámico 10/50 Puntos

Un estudiante aficionado a la física y a la tecnología

ha ideado un dispositivo capaz de funcionar como un gato que permita levantar cuerpos a pequeñas alturas.

El dispositivo consiste en un tubo cilíndrico vertical con secciones diferentes; en la parte superior tiene un radio 1 9,00cmr = y en la inferior 2 7,00 cmr = , tal como se representan en la figura 1. Dentro del tubo hay dos émbolos de masas 1 4,00kgM = y 2 0,900kgM = , unidos mediante una cadena inextensible, de longitud m00,1=L y masa 0,100kgcm = . Los émbolos, que ajustan perfectamente en el tubo, pueden deslizar sin fricción. Todos los materiales con los que se ha construido el sistema son perfectos aislantes del calor.

Mediante la llave S se puede igualar la presión del espacio comprendido entre los émbolos con la atmosférica del exterior, 51,01 10 Paatp = ⋅ . Con la llave S abierta, la base inferior de M1 se apoya sobre unos pequeños pivotes que tienen como objeto, entre otros, dejar espacio para alojar una resistencia eléctrica de calefacción que se alimenta con una batería ε cuando se cierra el interruptor I.

Se supone que en el estado inicial (que es el representado en la figura 1), la temperatura de todo el sistema es la ambiente, 23,00 10 KaT = ⋅ . A continuación, se cierra la llave S y se mantiene cerrada en todo lo que sigue. Considera que el aire se comporta como un gas perfecto diatómico5 de densidad -31,29 kg mρ = .

1) Determina la masa de aire, airem , encerrada entre los émbolos. Comprueba que esta masa es mucho menor que la del sistema deslizante (émbolos + cadena) y, por este motivo, no se considerará en el resto del problema.

2) Con objeto de levantar los émbolos (gato termodinámico), al aire encerrado entre ambos se le suministra lentamente calor mediante una resistencia eléctrica. En consecuencia, la presión interior variará. ¿Cuál es el valor de la presión crítica, pc, para la cual los émbolos comenzarán su ascenso? (Toma 9,81 m s–2 como valor de g).

3) Desde el estado inicial hasta que los émbolos comienzan a ascender,

5 Calores específicos molares de un gas ideal diatómico, a presión y a volumen constante: 25 /Rcv = ;

27 /Rc p = , donde R es la constante de los gases perfectos.

Fig. 1

M1

M2

SL

Page 110: XXIII olimpiadas españolas de física, enunciados y soluciones de las pruebas propuestas - pag 214.pdf

Página 9 de 18

XVIII OLIMPIADA ESPAÑOLA DE FÍSICA. JAÉN 2007R.S.E.F.

a) ¿Qué tipo de proceso termodinámico ha tenido lugar? b) ¿Cuál es la temperatura, 1T , del aire al comenzar el ascenso?

c) ¿Cuánto calor, 1Q , habrá sido necesario suministrar para que 1M empiece a ascender?

4) Una vez que 1M despega, se produce la acción útil del gato elevando este émbolo hasta una altura 20,0cmh = . Supóngase que la elevación es muy lenta para poder despreciar la energía cinética del sistema. a) ¿Qué tipo de proceso termodinámico ha tenido lugar? b) Calcula la temperatura, 2T , del gas al final de este proceso.

c) ¿Cuánto calor adicional, 2Q , habrá sido necesario suministrar al gas?

5) Si se considera como trabajo útil el necesario para levantar el émbolo M1 la altura h, calcula la relación, expresada en %, entre dicho trabajo y el calor total suministrado, lo que puede llamarse rendimiento, η , del proceso.

6) Para que el sistema evolucione lentamente, el suministro de calor se realiza mediante una resistencia 1,00kΩr = conectada a una batería, de resistencia interna despreciable y fuerza electromotriz ε = 50,0 V. Calcula el tiempo, t, que deberá estar conectada la batería durante todo el proceso.

7) Representa en un diagrama Presión-Volumen el proceso seguido por el gas (aire) desde el estado inicial hasta que 1M haya subido la altura h.

Solución

1) Despreciando el volumen de la parte ancha del tubo en la que se encuentran los pequeños pivotes y el de la conexión a la llave S, la masa de aire encerrada cuando se cierra la válvula S es

22aire airem r Lρ π= ⇒ caire mMMm ,,kg1099,1 21

2 <<⋅= −

2) Cuando la llave S se cierra, antes de comunicar calor, el aire encerrado entre los émbolos está a la presión atmosférica. Los pivotes que sujetan al conjunto móvil ejercen unas fuerzas normales cuya resultante es

( )0 1 2 cN M M m g= + +

Al conectar el interruptor I comenzará la transferencia de calor y la presión del aire interior aumentará. Para una presión atp p p= + ∆ , la resultante de las fuerzas de presión sobre los émbolos es atp A p A p A∆ − ∆ = ∆ ∆ hacia arriba, por lo que el valor de la reacción normal será

Page 111: XXIII olimpiadas españolas de física, enunciados y soluciones de las pruebas propuestas - pag 214.pdf

Página 10 de 18

XVIII OLIMPIADA ESPAÑOLA DE FÍSICA. JAÉN 2007R.S.E.F.

( )1 2 cN M M m g p A= + + − ∆ ∆ con ( )22

21 rrA −=∆ π

A medida que aumente la presión en el interior, la normal disminuirá hasta que para un valor critico de la diferencia de presiones, ∆ cp , se anule.

( )1 2 cc

M M m gp

A+ +

∆ =∆

De donde la presión crítica absoluta es

( )1 2

2 21 2( )

cc at

M M m gp p

r rπ+ +

= +−

⇒ 51,06 10 Pacp = ⋅

Como se observa, la presión sólo depende de la masa total y de la geometría del sistema móvil.

3a) Hasta que la presión alcanza el valor crítico, el volumen de aire encerrado entre los émbolos no cambia. En consecuencia, sufre un proceso lento a volumen constante.

Tipo de proceso: a volumen constante (isocoro) 3b) Como la transferencia de calor se realiza lentamente, se puede suponer que todos los

estados intermedios son de equilibrio (proceso cuasiestático). Por lo tanto, la ecuación de estado de los gases perfectos permite escribir

0

0 1

at a

c

p V nRTp V nRT

= ⎫⎬= ⎭

Donde n es el número de moles del aire encerrado, R la constante de los gases, 0V el volumen de aire inicial y 1T la temperatura del aire correspondiente a la presión pc que se trata de determinar. Dividiendo las relaciones anteriores,

1c

aat

pT Tp

= ⇒ 1 314KT =

3c) En este proceso a volumen constante, el calor 1Q que ha sido necesario aportar es

( )1 1v aQ nc T T= −

Teniendo en cuenta que 5 / 2vc R= y que 0at

a

p VnRT

= ,

2 11 2

5 12 at

a

TQ p r LT

π⎛ ⎞

= −⎜ ⎟⎝ ⎠

⇒ 1 188 JQ =

4a) A partir del estado de presión crítica, los émbolos comienzan a elevarse manteniendo la presión del aire encerrado constante e igual a pc, y aumentando el volumen y temperatura del aire encerrado. El proceso se realiza, lentamente, a presión constante.

Tipo de proceso: a presión constante (isobaro)

Page 112: XXIII olimpiadas españolas de física, enunciados y soluciones de las pruebas propuestas - pag 214.pdf

Página 11 de 18

XVIII OLIMPIADA ESPAÑOLA DE FÍSICA. JAÉN 2007R.S.E.F.

4b) Razonando de forma análoga que en el apartado 3b, tendremos

( )

0 1

0 2

c

c

p V nRTp V V nRT

= ⎫⎪⎬+ ∆ = ⎪⎭

Donde 2T es la temperatura final del sistema y V∆ la variación total del volumen de aire correspondiente a la elevación de los émbolos la altura h, cuyo valor es

V h A∆ = ∆ . Dividiendo las expresiones anteriores y despejando 2T

( )2 2

1 22 1 2

2

1h r r

T Tr L

⎛ ⎞−⎜ ⎟= +⎜ ⎟⎝ ⎠

⇒ 2 356KT =

4c) Ahora el proceso es a presión constante, y el calor 2Q aportado será

( )2 2 1pQ nc T T= −

Teniendo en cuenta que 7 / 2pc R= y que no ha variado el número de moles, resulta

( )2

22 2 1

72

at

a

p rQ T TTπ

= − ⇒ 2 745 JQ =

5) Trabajo útil es el que se realiza para elevar el bloque M1 hasta una altura h

1útilW M g h=

Energía aportada total 1 2W Q Q= +

El rendimiento es:

1 2

100útilWQ Q

η = ⋅+

⇒ 1

1 2

100M g hQ Q

η = ⋅+

⇒ 0,84%η =

Realmente, el gato diseñado es deplorable. 6) El suministro de calor se hace de acuerdo con la ley de Joule:

2

1 2εQ Q tr

+ = ⇒ 1 22ε

Q Qt r+= ⇒ min22,6373 == st

7) Como hemos indicado, los procesos pueden considerarse cuasiestáticos, por lo que pueden representarse en un diagrama P-V. La primera fase del proceso, isocora, la segunda, isobara, se representan cualitativamente (y no a escala) en la figura 3.

p

V

pc

pat

Fig. 3 V0 + ∆V V0

Page 113: XXIII olimpiadas españolas de física, enunciados y soluciones de las pruebas propuestas - pag 214.pdf

Página 12 de 18

XVIII OLIMPIADA ESPAÑOLA DE FÍSICA. JAÉN 2007R.S.E.F.

Prueba experimental. 20/50 Puntos Campo magnético de un imán y campo magnético terrestre

Objetivos

Como bien sabes, el campo gravitatorio creado por una partícula decrece con el cuadrado de la distancia. Pero, ¿sabes con qué potencia de la distancia decrece el campo magnético creado por un imán? En la primera parte de esta prueba se determinará experimentalmente cómo es esta dependencia. En la segunda parte se obtendrá la intensidad del campo magnético terrestre y el momento magnético del imán empleado.

Materiales • Cuatro imanes iguales. • Papel con transportador de

ángulos impreso. • Regla. • Palillo de madera. • Cinta adhesiva. • Tijeras. • Hilo. • Cilindro de PVC. • Barra de PVC. • Goma de borrar. • Cronómetro.

Montaje a) En primer lugar hay que montar una brújula, que indicará la dirección del campo

magnético. Para ello se van a utilizar dos imanes cilíndricos, unidos longitudinalmente y colgados mediante un hilo de un soporte construido con un tubo y una barra de PVC, como se indica en la figura 1. En adelante, llamaremos "A" a esta pareja de imanes. Sujeta el hilo a la barra con un trozo de cinta adhesiva y cuelga los imanes A en el otro extremo, pellizcando el hilo entre ambos. La altura de la brújula sobre la mesa puede regularse girando la barra de PVC para enrollar o desenrollar hilo. Para poder medir en el transportador la dirección de la brújula, sujeta debajo de los imanes A un palillo largo, cortado a la longitud apropiada y bien alineado con el eje de simetría de los imanes.

b) Coloca sobre la mesa el papel con el transportador impreso y la brújula con su soporte de modo que:

• La brújula esté lo más alejada posible de la estructura de hierro de la mesa. Aleja también la otra pareja de imanes para que no influyan en la orientación de la brújula.

• El eje de rotación de la brújula (el hilo) esté exactamente encima del centro del transportador.

Fig. 1

A

B

Page 114: XXIII olimpiadas españolas de física, enunciados y soluciones de las pruebas propuestas - pag 214.pdf

Página 13 de 18

XVIII OLIMPIADA ESPAÑOLA DE FÍSICA. JAÉN 2007R.S.E.F.

• La dirección N-S marcada por la brújula (dirección del palillo) coincida con la línea 0º-180º del transportador.

• El palillo quede cerca del transportador, sin llegar a rozarlo.

• Una vez bien alineado el sistema, es conveniente sujetar sus elementos a la mesa con cinta adhesiva para evitar que se muevan accidentalmente.

c) Para terminar, une longitudinalmente los otros dos imanes (en adelante pareja "B") y pégalos con cinta adhesiva a la goma de borrar (figura 2), para que su altura sea similar a la de los imanes de la brújula. Para facilitar las medidas posteriores, es conveniente dibujar en la goma unas líneas que se crucen en el punto medio de los imanes, como se muestra en la figura 2.

Procedimiento experimental Con el montaje anterior, la brújula está inicialmente orientada en la dirección de la

componente horizontal del campo magnético de la Tierra (dirección N – S). Si se sitúan en perpendicular (dirección E – O) la pareja de imanes B, la brújula gira hasta orientarse en la dirección del campo magnético total, suma del terrestre y del producido por los imanes B.

En la primera parte de esta prueba experimental se va a medir la desviación angular de la brújula al acercar gradualmente la pareja de imanes B y, a partir de estas medidas, se determinará cómo decrece con la distancia el campo magnético que crean.

En la segunda parte, se medirá el periodo de las oscilaciones torsionales de la brújula, formada ahora por la pareja de imanes B, en presencia del campo terrestre, y se determinará el valor de dicho campo (de su componente horizontal) y del momento magnético de esta pareja de imanes.

Primera Parte. Dependencia con la distancia del campo magnético de un imán El campo magnético producido por un imán cilíndrico en un punto de su eje de

simetría lleva la dirección de dicho eje, y su módulo puede expresarse, en puntos alejados frente al tamaño del imán, en la forma

0

2m n

mBr

µπ

=

donde m es el llamado momento magnético del imán, que caracteriza su "potencia", r es la distancia al centro del imán y n es un número entero positivo que queremos determinar experimentalmente.

Dato: 270 AN104 −−⋅= πµ

Fig. 2

θ BH

Fig. 3

r Bm

N

S

E O B A

Page 115: XXIII olimpiadas españolas de física, enunciados y soluciones de las pruebas propuestas - pag 214.pdf

Página 14 de 18

XVIII OLIMPIADA ESPAÑOLA DE FÍSICA. JAÉN 2007R.S.E.F.

Con la geometría de nuestro montaje, el campo Bm producido por la pareja de imanes B, orientados en la dirección E - O, es perpendicular a la componente horizontal del terrestre, BH, de forma que la brújula se orienta a un ángulo θ con la dirección N – S dado por (véase la figura 3)

0 1tg2 n

H

mB r

µθπ

= (1)

1) Mide la desviación angular de la brújula para valores de r entre 20 cm y 40 cm, a intervalos de 2 cm. Para evitar errores sistemáticos debidos a pequeños errores de alineación, es conveniente medir las desviaciones angulares en sentidos opuestos, θ 1 y θ 2, obtenidas al orientar los imanes B en un sentido u otro sobre la dirección E – O, de forma que se invierta el sentido de Bm. Toma como valor para θ el promedio de estas dos desviaciones. Sugerencia: dibuja trazos transversales sobre la línea larga de la cruz impresa en el papel, a las distancias r en que vas a realizar las medidas. Estos trazos son fáciles de alinear con las marcas previamente realizadas en la goma, que marcan el punto medio de la pareja B de imanes. A partir de estas medidas, construye la siguiente tabla, reservando la última columna para el apartado 5.

r θ 1 θ 2 θ tgθ ln(tgθ) ln(r)

Presenta estos datos en la tabla impresa en la hoja de respuestas. 2) Transforma la ecuación (1) y demuestra que es de esperar una dependencia lineal entre

ln(tgθ) y ln(r).

3) Representa gráficamente los puntos experimentales ln(tgθ), en ordenadas, frente a ln(r), en abscisas.

4) A partir del ajuste a una línea recta de estos puntos, determina el valor de n. Ten en cuenta que n debe ser un numero entero, por lo que debes aproximar el valor obtenido al entero más próximo.

5) Completa la última columna de la tabla del apartado 1 con los valores de nr/1 .

6) Representa gráficamente los puntos tgθ frente a nr/1 . 7) A partir de la representación anterior, y teniendo en cuenta la ecuación (1), determina

el valor de m/BH.

Page 116: XXIII olimpiadas españolas de física, enunciados y soluciones de las pruebas propuestas - pag 214.pdf

Página 15 de 18

XVIII OLIMPIADA ESPAÑOLA DE FÍSICA. JAÉN 2007R.S.E.F.

Segunda Parte. Determinación6 de BH y de m. En el apartado anterior se ha determinado el valor del cociente m/BH. En esta segunda

parte se determinará el valor del producto mBH a partir del periodo de oscilación de esta pareja de imanes colgada de un hilo, formando una brújula. Una vez conocidos los valores de m/BH y mBH, se obtendrán los de m y BH.

En presencia del campo magnético terrestre, nuestra brújula marca en equilibrio la dirección N – S. Si se le da un pequeño impulso angular (en el sentido de retorcer el hilo del que cuelgan los imanes), oscila en torno a la dirección de equilibrio. Este sistema oscilante constituye un péndulo de torsión. El par de fuerzas que tiende a llevar la brújula a su orientación de equilibrio se debe a la interacción entre el campo magnético de la Tierra (componente horizontal), BH, y el momento magnético de la brújula, m. Despreciando el pequeño efecto recuperador debido a la torsión del hilo, se demuestra que el periodo T de pequeñas oscilaciones torsionales de la brújula es

2H

ITmB

π= (2)

donde I es el momento de inercia de la brújula. Esta magnitud representa la inercia de un objeto a cambiar su movimiento de rotación. Depende de la masa del objeto y de su distribución respecto el eje de rotación. Si el cuerpo es un cilindro recto de masa M, longitud L y radio R, que gira respecto a un eje perpendicular al eje principal de simetría por el punto medio (como es nuestro caso), el valor de I se obtiene en la forma

12/4/ 22 MLMRI += Emplea como brújula la pareja de imanes B colgada del soporte, como la pareja A en

la primera parte pero sin el palillo. Aleja los imanes A para que no influyan en la medida. 8) Calcula el momento de inercia de la brújula respecto el eje de rotación indicado, I.

Datos de cada uno de los dos imanes: masa Mi = 3,10 g, longitud Li = 15,0 mm y diámetro Di = 6,0 mm.

9) Coloca la brújula en, aproximadamente, la misma posición sobre la mesa que en la primera parte, para que el valor de BH no cambie apreciablemente debido a la estructura metálica de la mesa. Gira la brújula respecto a su orientación de equilibrio y suéltala para que realice pequeñas oscilaciones torsionales. Mide el periodo, T, de estas oscilaciones. Describe detalladamente el método de medida que has empleado.

10) Utiliza la ecuación (2) para calcular el valor de mBH. Combina este resultado con el valor de m/BH obtenido en el apartado 7 y calcula los valores de m y BH.

11) Haz una estimación de la incertidumbre (margen de error) del valor de T obtenido en el apartado 9. Teniendo en cuenta sólo esta fuente de error experimental, calcula la incertidumbre de los valores de m y BH obtenidos en el apartado 10.

6 La determinación precisa del campo magnético terrestre requiere hacer el experimento en un lugar

alejado de objetos y estructuras de hierro, lo que no es posible en nuestro caso. El valor que se obtenga para BH corresponderá al campo magnético local existente en la posición de la brújula.

Page 117: XXIII olimpiadas españolas de física, enunciados y soluciones de las pruebas propuestas - pag 214.pdf

Página 16 de 18

XVIII OLIMPIADA ESPAÑOLA DE FÍSICA. JAÉN 2007R.S.E.F.

Solución 1) Medidas y cálculos

Medida r (m) θ1 (º) θ2 (º) θ (º) tgθ ln(tgθ) ln(r) 1/r3 (m-3)1 0,380 6 7 6,5 0,1139 -2,172 -0,968 18,22 2 0,360 7 8 7,5 0,1317 -2,028 -1,022 21,43 3 0,340 8 9 8,5 0,1495 -1,901 -1,079 25,44 4 0,320 10 11 10,5 0,1853 -1,686 -1,139 30,52 5 0,300 12 13 12,5 0,2217 -1,506 -1,204 37,04 6 0,280 15 16 15,5 0,2773 -1,283 -1,273 45,55 7 0,260 19 19 19,0 0,3443 -1,066 -1,347 56,90 8 0,240 23 24 23,5 0,4348 -0,833 -1,427 72,34 9 0,220 29 30 29,5 0,5658 -0,570 -1,514 93,91

10 0,200 37 38 37,5 0,7673 -0,265 -1,609 125,0

2) 0 1tg2 n

H

mB r

µθπ

= (1)

Tomando logaritmos, la ecuación que se pide es

( ) ( )0ln tg ln ln2 H

m n rB

µθπ

⎛ ⎞= −⎜ ⎟

⎝ ⎠ (2)

3) Gráfica de ln(tgθ) frente a ln(r).

4) Se obtiene n = 2,99. Por tanto 3=n .

5) Los valores de 1/r3 se presentan en la última columna de la tabla del apartado 1.

6) Gráfica de tgθ frente a 1/r3.

y = -2,99x - 5,09

-2,5

-2

-1,5

-1

-0,5

0

-1,8 -1,6 -1,4 -1,2 -1,0 -0,8 ln(r )

ln(tgθ )

Page 118: XXIII olimpiadas españolas de física, enunciados y soluciones de las pruebas propuestas - pag 214.pdf

Página 17 de 18

XVIII OLIMPIADA ESPAÑOLA DE FÍSICA. JAÉN 2007R.S.E.F.

7) Según (1), la pendiente de esta gráfica es 3 30 6,08 10 m2 H

mB

µπ

−= ⋅ , de donde

4 2 3 -13,04 10 A m NH

mB

= ⋅ (3)

8) Momento de inercia. Para M = 6,20·10-3 kg, L = 30,0·10-3 m y R = 3,0·10-3 m, resulta

( ) 7 20,14 4,65 10 kg mI −= + ⋅ → 7 24,79 10 kg mI −= ⋅

9) Medida del periodo de oscilación de la brújula.

Nº oscilaciones 10 10 10 10 10 10

Tiempo (s) 9,40 9,39 9,76 9,62 9,29 9,54

Periodo, T (s) 0,940 0,939 0,976 0,962 0,929 0,954

10) 2H

ITmB

π= → 2

2

4H

ImBTπ

= (4)

→ 52,10 10 NmHmB −= ⋅ (5)

La resolución del sistema de ecuaciones formado por (3) y (5) conduce a

2

5

mA798,0

T1063,2

=

⋅= −

m

BH

11) Precisión de los resultados: El error típico del periodo de oscilación, determinado en el apartado 9 como promedio de seis medidas, es

( )2

0,007 s6 5

iT TT

−∆ = =

×∑

→ 0,950 sT =

y = 0,006075x

0

0,2

0,4

0,6

0,8

0 20 40 60 80 100 120 140

tgθ

1/r3 (m-3)

Page 119: XXIII olimpiadas españolas de física, enunciados y soluciones de las pruebas propuestas - pag 214.pdf

Página 18 de 18

XVIII OLIMPIADA ESPAÑOLA DE FÍSICA. JAÉN 2007R.S.E.F.

El número de medidas no es muy elevado, por lo que una estimación razonable de la incertidumbre del periodo obtenido podría ser7

0,01sT∆ =

Para calcular los errores transmitidos a los valores de m y BH es conveniente obtener la dependencia explícita de ambos con el periodo de oscilación. A partir de (4) y del valor numérico de m/BH ≡ k, dado en (3), se obtiene

12

12

HIB

T k

m k IT

π

π

=

=

Las incertidumbres transmitidas a m y BH pueden calcularse en la forma

72

2 3 10 TH HH

B I BB T T TT T k T

π −∂∆ = ∆ = ∆ = ∆ = ⋅

3 22

2 8 10 A mm mm T k I T TT T T

π −∂∆ = ∆ = ∆ = ∆ = ⋅

Estos márgenes de error también pueden calcularse de una forma directa evaluando BH y m para los valores extremos de T, dentro del intervalo de incertidumbre estimado

71 1 12 3 10 T2H

IBk T T T T

π −⎡ ⎤∆ = − = ⋅⎢ ⎥− ∆ + ∆⎣ ⎦

3 21 1 12 8 10 Am2

m kIT T T T

π −⎡ ⎤∆ = − = ⋅⎢ ⎥− ∆ + ∆⎣ ⎦

Los resultados finales para BH y m son, con incertidumbres transmitidas8

( )

( )

5

2

2,63 0,03 10 T

0,798 0,008 A mHB

m

−= ± ⋅

= ±

Notas: El valor del exponente n = 3 puede considerarse libre de error, ya que es entero. El error típico relativo de la pendiente de la gráfica en el apartado 6, que permite

determinar m/BH, es del orden del 0,3%. Por tanto, para el conjunto de medidas que se presentan, la influencia de esta fuente de error en el resultado final es prácticamente despreciable frente a la del periodo de oscilación.

7 Un cálculo estadístico más exacto conduce a que, con un nivel de confianza del 95%, s018,0=∆T . 8 En ambos casos, las incertidumbres relativas son del orden del 1%. Esto era de esperar pues el error

relativo de T es de este mismo orden, y tanto BH como m son inversamente proporcionales a T.

Page 120: XXIII olimpiadas españolas de física, enunciados y soluciones de las pruebas propuestas - pag 214.pdf

Página 19 de 18

XVIII OLIMPIADA ESPAÑOLA DE FÍSICA. JAÉN 2007R.S.E.F.

Tablas de respuestas de los problemas teóricos Problema teórico 1.

Apartado Expresiones analíticas Resultados numéricos Puntos

A1 agga += 0 1

A2 agga −= 0 1

A3 agga −= 0 1

A4 agga += 0 1

B1 ϕsen

2 0hgvB =

km/680m/s190 ==Bv

1 + 0,5

B2 AB

BAB t

vgg ϕsen0 += ;

AB

BDE t

vgg ϕsen0 += 071 g,g AB =

071 g,gDE = 1 + 0,5 1 + 0,5

B3 0

0sen

2g

vt Bg

ϕ== s270 ==gt 1 + 0,5

Problema teórico 2.

Apartado Expresiones analíticas Resultados numéricos Puntos

1º 81

29 /

eke

Bx ⎟⎟⎠

⎞⎜⎜⎝

⎛= 2,0

2º ( ) ⎟⎟⎠

⎞⎜⎜⎝

⎛+−= 9

82

91

xx

xkexU e 1,0

3º 3

28

exkeK = 2,0

4º 3H

2821

exmkef

π= 2,0

5º nm0,15m10551 10 =⋅= −,xe

1,0

6º ( )( )e

AeAd xkeNxUNW

98 2

=−= kJ/mol10967 2⋅= ,Wd 1,5 +0,5

Page 121: XXIII olimpiadas españolas de física, enunciados y soluciones de las pruebas propuestas - pag 214.pdf

Página 20 de 18

XVIII OLIMPIADA ESPAÑOLA DE FÍSICA. JAÉN 2007R.S.E.F.

Problema teórico 3.

Apartado Expresiones analíticas Resultados numéricos Puntos

1º kg10991 2−⋅= ,maire 0,25

( )

)( 22

21

21rr

gmMMpp catc

+++=

π

Pa10061 5⋅= ,pc 2,0 + 0,25

a)

Tipo de proceso: a volumen constante (isocoro).

0,25

b) at

ca p

pTT =1 K3141 =T 1,0 + 0,25

c) ⎟⎠

⎞⎜⎝

⎛ −= 125 12

21a

at TTrpQ π J1881 =Q 1,0 + 0,25

a) Tipo de proceso: a presión constante (isobaro). 0,25

b) ( )

⎟⎟⎠

⎞⎜⎜⎝

⎛ −+=

LrrrhTT 2

2

22

21

12 1 K3562 =T 1,0 + 0,25

c) ( )12

22

2 27 TT

TrpQ

a

at −=π J7452 =Q 1,0 + 0,25

5º 10021

1 ⋅+

=QQ

hgMη %,840=η 0,5 + 0,25

6º rε

QQt 221+= min6,22s373 ==t 0,5 + 0,25

0,5

p

V

pc

pat

V + ∆VV

Page 122: XXIII olimpiadas españolas de física, enunciados y soluciones de las pruebas propuestas - pag 214.pdf

1

R.

XVII OLIMPIADA ESPAÑOLA DE FÍSICA. TERUEL 2006

Problema Teórico 1. Un calendario fósil. 12/60 Puntos Algunos corales generan en su esqueleto finas estrías a causa de las interrupciones

diarias (día-noche) de su crecimiento. Estas estrías, posiblemente debidas a las variaciones de profundidad del mar por efecto de las mareas, se agrupan en estrechas bandas que corresponden a cada mes lunar. A su vez, las bandas mensuales se agrupan en otras, más anchas, con una periodicidad anual. Pueden apreciarse las citadas bandas en la fotografía de la figura 1, que corresponde a un coral fósil de Calceola sandalina, perteneciente al Museo Paleontológico de la Universidad de Zaragoza.

En definitiva, el sistema de estrías y bandas del esqueleto coralino equivale a un “calendario” de la época en la que el coral vivió y, a través de un estudio paleontológico, se deduce que entonces la duración del año era de unos 400 días. Por tanto, la Tierra giraba en torno a su eje con una velocidad angular mayor que en la actualidad. Puede suponerse que el periodo de rotación de la Tierra en torno al Sol no ha variado apreciablemente desde aquella época.

Por otra parte, mediciones muy precisas del tiempo de vuelo de pulsos láser emitidos desde la Tierra y reflejados en espejos colocados en la Luna, en misiones norteamericanas y de la antigua URSS, muestran que la distancia Tierra-Luna aumenta a razón de 3,8 cm/año.

La disminución de la velocidad angular de rotación de la Tierra y, en consecuencia, el paulatino aumento de la distancia entre la Luna y la Tierra, se deben a la enorme disipación de energía que se produce por la fricción del flujo y reflujo de las mareas oceánicas con los fondos marinos.

Dado que la masa de la Tierra es considerablemente mayor que la de la Luna y que la distancia entre sus centros es mucho mayor que cualquiera de sus radios, permite considerar la Luna como una partícula (puntual) de masa ML que describe una órbita

circular de radio R en torno al centro de la Tierra. En la figura 2 se representa a escala el sistema Tierra-Luna.

TL

Órbita R

ω

Fig. 2

Fig. 1

Bandas mensuales

Bandas anuales

Page 123: XXIII olimpiadas españolas de física, enunciados y soluciones de las pruebas propuestas - pag 214.pdf

2

R.

XVII OLIMPIADA ESPAÑOLA DE FÍSICA. TERUEL 2006

Suponga la Tierra esférica, con su eje de rotación perpendicular al plano de la órbita lunar, y que la pequeña velocidad con la que la Luna se aleja de la Tierra ha permanecido constante a lo largo del tiempo. Tenga en cuenta también que la Luna, vista desde la Tierra, presenta siempre la misma cara.

Con los datos que se indican al final del enunciado, a) Determine la distancia actual entre La Tierra y la Luna, R, en función de la

velocidad angular orbital de la Luna, ω, del radio de la Tierra, RT, y de la aceleración de la gravedad en la superficie de la Tierra, g. Calcule ω y R.

b) Calcule las velocidades angulares de la Tierra en torno a su eje en la actualidad, Tω , y cuando el coral vivía. Tω′ .

c) Determine la distancia entre la Tierra y la Luna, R′ , en la época en la que vivió el coral, en función de las siguientes magnitudes: R, MT , ML , RT, Tω , Tω′ y ω . Calcule el valor de R′ .

d) Haga una estimación de la “edad” del coral fósil, τ. Datos:

Masas de la Tierra y de la Luna: MT = 5,98·1024 kg; ML = 7,35·1022 kg; Radio de la Tierra: RT = 6,37·106 m Periodo de revolución de la Luna en torno a su eje: s10362 6⋅= ,TL Periodo de revolución de la Tierra en torno al Sol: s10163 7⋅= ,T Día sidéreo: s10648 4⋅= ,TT Aceleración de la gravedad en la superficie de la Tierra: 2m/s819,g =

Ayudas: De acuerdo con el modelo propuesto (Tierra rotatoria con su centro fijo y Luna de masa puntual), el momento angular del sistema respecto al centro de la Tierra es igual a la suma de los momentos angulares de la Luna en torno a la Tierra y de la Tierra en torno a su eje. Si el sistema está aislado, tal como se considera al sistema Tierra-Luna en este problema, su momento angular total debe conservarse. Para una esfera de masa m y radio a que gira con velocidad angular Ω en torno a un eje que pasa por su centro (figura 3), el módulo de su momento angular respecto al centro es 0L I= Ω , donde I

es el llamado momento de inercia respecto a dicho eje, cuyo valor es 225I m a= .

Solución a) De acuerdo con el modelo que sugiere el enunciado y representado en la figura 2,

la ecuación del movimiento de la Luna, considerada como una partícula de masa ML, que describe una órbita de radio R en torno a la Tierra es,

22 ωRM

RMMG L

LT = ⇒ 3RMG T=ω (1)

0L

Ω

Fig.3

Page 124: XXIII olimpiadas españolas de física, enunciados y soluciones de las pruebas propuestas - pag 214.pdf

3

R.

XVII OLIMPIADA ESPAÑOLA DE FÍSICA. TERUEL 2006

Como la aceleración de la gravedad es 2T

T

RM

Gg = ⇒ 3RgRT=ω , por lo que

31

2

2 /T gRR ⎟

⎟⎠

⎞⎜⎜⎝

⎛=

ω.

Dado que la Luna presenta siempre la misma cara, la velocidad angular de rotación de la Luna debe ser igual a su velocidad angular orbital, es decir

LTπω 2

= ⇒

rad/s10662 6−⋅= ,ω , por lo que el valor de R es m10833 8⋅= ,R . b) La velocidad angular actual de la Tierra en torno a su eje es 2T TTω π= . Como las

estrías del coral indican que mientras la Tierra realizaba una revolución en torno al Sol daba 400 vueltas en torno a su eje, el periodo T’ de revolución de la Tierra era,

s10907s400

10163 47

⋅=⋅

=′ ,,T

y, en consecuencia, la velocidad angular de la Tierra cuando vivía el coral era,

TT ′

=′ πω 2 ⇒ 57,95 10 rad/sTω −′ = ⋅

c) Considerando la Tierra y la Luna como un sistema aislado, su momento angular se conserva, lo que significa que el momento angular respecto al centro de la Tierra en la actualidad debe ser igual que en la época en la que el coral estaba vivo. Por lo tanto, de acuerdo con el modelo que se propone, (Tierra rotatoria con su centro fijo y Luna masa puntual),

ωωωω 22 RMIRMI LTTLTT +=′′+′ (2)

De (1), la velocidad angular orbital de la Luna, en la actualidad y cuando vivía el coral son, respectivamente,

3RgRT=ω y

3R

gRT

′=′ω

Por lo que, sustituyendo en (2), gRRMIRgRMI TLTTTLTT +=′+′ ωω .

De la “ayuda”, el momento de inercia es 52 2 /RMI TTT = , con lo que resulta,

( )2

521

⎥⎦

⎤⎢⎣

⎡−′−=′ TT

L

TTM

RMgR

gR ωω

Con los valores obtenidos para Tω , Tω′ y ω y los datos del enunciado, se obtiene

m10653 8⋅=′ ,R d) De acuerdo con el enunciado, la velocidad con la que la Luna se aleja de la Tierra

es constante y de valor -93,8cm/año 1,2 10 m/sv = = ⋅ . La “edad” del coral fósil puede estimarse como el tiempo que ha transcurrido desde que la distancia Tierra-

Luna era R´ hasta la actual R, es decir, R Rv

τ′−

= ⇒ 161,5 10 sτ = ⋅ .

Page 125: XXIII olimpiadas españolas de física, enunciados y soluciones de las pruebas propuestas - pag 214.pdf

4

R.

XVII OLIMPIADA ESPAÑOLA DE FÍSICA. TERUEL 2006

Problema T2. Electrómetro absoluto de Kelvin 12/60 Puntos Tres placas metálicas están en el vacío, colocadas como se indica en la figura 1. La

placa C es circular de radio a; la B posee un orificio circular de radio ligeramente mayor que a, y en él esta colocada la placa C. La A, de radio igual al exterior de B, está separada de las anteriores una distancia d considerablemente menor que a. Cuando las placas se conectan eléctricamente a unos bornes entre los cuales hay una diferencia de potencial ∆V, la placa B, llamada anillo de guarda, hace que el campo eléctrico en la región cilíndrica comprendida entre la C y la A sea uniforme.

Tomando como datos la diferencia de potencial ∆V, la distancia d, el radio a de la placa C y la permitividad eléctrica del vacío ε0, determine:

a) El módulo, dirección y sentido del campo eléctrico uniforme E, en la región comprendida entre las placas C y A.

b) La carga eléctrica en la placa C. c) El módulo, dirección y sentido de la fuerza que la placa A ejerce sobre la C. Considérese ahora la figura 2, que es un

esquema simplificado del instrumento con el cual Lord Kelvin en 1860, midió por primera vez la fuerza electromotriz de una asociación en serie de pilas Daniell.

El aparato consiste en una balanza, cuyo plato izquierdo, suspendido por alambres conductores, es un disco metálico que juega el papel de la placa C de la figura 1. Coplanario con el disco C hay un anillo de guarda y a una distancia d por debajo, se sitúa la placa fija A.

Para aislar al dispositivo de posibles influencias eléctricas externas se coloca dentro de una caja metálica (jaula de Faraday) conectada a tierra. La figura 3, tomada de un libro de Física de los años 30, muestra un electrómetro de este tipo sin la caja metálica.

Con este dispositivo, la batería cuya fem ε se desea medir, se conecta como se indica en la figura 2. Como la placa A ejerce una fuerza sobre la C, para mantener la balanza en equilibrio es preciso añadir pesas en el platillo derecho de la balanza.

Fig. 1

d ∆V A

C B

Fig

d

Caja áli

ε

Fig. 2

A

C B

Page 126: XXIII olimpiadas españolas de física, enunciados y soluciones de las pruebas propuestas - pag 214.pdf

5

R.

XVII OLIMPIADA ESPAÑOLA DE FÍSICA. TERUEL 2006

d) Si la masa requerida para equilibrar la balanza es kg10212 5−⋅= ,m , empleando los datos que se indican a continuación, determine la fuerza electromotriz de la batería, ε .

e) ¿Cuál es el error ∆ε en la medida de la fem debido a una imprecisión 0,5 mgm∆ = en la masa de la pesa?

Datos: Separación entre las placas: m10002 3−⋅= ,d . Radio de la placa C: m10001 1−⋅= ,a ; Permitividad dieléctrica: 21212

0 mNC10858 −−−⋅= ,ε . Aceleración de la gravedad: 19,81 msg −=

Solución a) De acuerdo con el enunciado, el anillo de guarda (placa B) asegura que el campo

eléctrico sea uniforme en la región comprendida entre la placa C y A. Por lo tanto, limitándonos a dicha región que es la de interés en el problema, la placa C por estar conectada al borne negativo, tendrá una carga negativa –Q, y, la inferior, conectada al borne positivo, tendrá la misma carga pero positiva, +Q. Ambas placas, separadas la distancia d, tal como se muestra en la figura 4, tienen una superficie 2aS π= .

El campo estará dirigido de la placa positiva a la negativa y, dado que el potencial entre las placas es ∆V, su valor será EdV =∆ , por lo que el módulo del campo es

dVE ∆

= (1)

y la dirección y sentido son las indicadas en la figura. b) Cada una de las placas, consideradas como planos cargados con densidades de carga

superficialesσ y σ− , crean sus respectivos campos. Su superposición es nula en puntos exteriores al espacio comprendido entre las placas y en los interiores es

σ=E (2)

En efecto, consideremos un punto P perteneciente al espacio entre placas y otros dos, P´ y P´´ exteriores, como se muestra en la figura 5a, y designamos por n al vector unitario perpendicular a ambas placas.

En la figuras 5b y 5c se representan los campos que crean individualmente las

∆V

F

E

0=E

0=E

0=E

F

n nE0ε

σ=

0=E

n02ε

σ−

n02ε

σ

n

F

n

F

n02ε

σ

n02ε

σ

n02ε

σ−

n

F

Page 127: XXIII olimpiadas españolas de física, enunciados y soluciones de las pruebas propuestas - pag 214.pdf

6

R.

XVII OLIMPIADA ESPAÑOLA DE FÍSICA. TERUEL 2006

placas en los puntos señalados y se indican sus valores pueden ser obtenidos mediante la ley de Gauss (aplicada a una “caja de píldoras” cilíndrica con su eje normal al plano y una base a cada lado). Por último, para obtener el campo total se suman en cada uno de los tres puntos los campos de cada placa. Es decir, se suman los valores indicados en las figuras 5b y 5c, obteniéndose los indicados en la figura 5d.

Se deduce por tanto que el campo total en la región interior es nE0ε

σ= , cuyo

módulo es el dado en la expresión (2). Por otra parte, eliminando E entre (1) y (2) se obtiene la densidad superficial de

carga de las placas,

dV∆

= 0εσ , (3)

por lo que la carga de la placa C, teniendo en cuenta que es negativa, será,

2aQ πσ−= ⇒ dVaQ ∆

−= 20 πε

c) Sobre cada una de las cargas de la placa C (Fig. 1 del enunciado) actúa una fuerza debida al campo eléctrico creado por la placa inferior, cuyo valor indicado en la figura 5b, es

nE02ε

σ=

Como el campo es uniforme y la distribución de carga (negativa) en C también lo es, la fuerza total sobre ella es

ndVanQF ∆

−=−= 2

0 22πσ

εσ

y teniendo en cuenta (3)

ndVaF

2

02

21

⎟⎠⎞

⎜⎝⎛ ∆

−== επ

d) Ahora la diferencia de potencial ∆V es la fuerza electromotriz de la batería,ε , que se pretende medir. La fuerza F desequilibra la balanza y para volverla a equilibrar será preciso colocar en el platillo derecho una masa m tal que el momento de su peso sea igual al de la fuerza. Si suponemos que los brazos de dicha balanza son iguales, se tendrá que mgF = , de donde resulta,

ε0

22

επ amgd= (4)

Sustituyendo los valores numéricos que nos dan, ε = V079,

e) El error ∆ε debido a una imprecisión ∆m de la masa m puede obtenerse de la forma siguiente,

Page 128: XXIII olimpiadas españolas de física, enunciados y soluciones de las pruebas propuestas - pag 214.pdf

7

R.

XVII OLIMPIADA ESPAÑOLA DE FÍSICA. TERUEL 2006

∆ε ( ) ( )0

20

222

21

επ

επ

a

gmmd

a

gmmd

−−

+=

Se ha empleado este razonamiento por ser el más utilizado por los estudiantes de Bachillerato. Podría haberse obtenido más directamente por medio de procedimientos habituales de propagación de errores.

Si g,,m k10005mg005 6−⋅==∆ , el resultado es ∆ε = 0,9 V

Problema Teórico 3. Oscilaciones 8/60 Puntos Una pequeña bolita de masa m descansa sobre un muelle que está oscilando

verticalmente con un movimiento armónico simple dado por y = A sen ωt. a) Deducir las expresiones de la fuerza Fb que el pistón ejerce

sobre la bolita en función del tiempo t y de la posición y. A partir de ellas, encontrar la relación que han de guardar los parámetros de este movimiento y la aceleración de la gravedad para que la bolita no se separe del pistón.

b) Siendo 2 2 , 15 cmA g Aω = = , ¿en qué posición yd e instante td se despega la bolita del pistón?

c) Para la aplicación numérica anterior, representar gráficamente, i) la fuerza por unidad de masa, /bF m , en función de la posición del pistón, y; ii), la posición de la bolita y del pistón en función del tiempo, en el intervalo 0 2t T π ω≤ ≤ = .

Solución La fuerza neta ΣF sobre la bola para que describa el movimiento dado, y la de

interacción bola – piston, Fb, son:

a) 2 2 sinbF F mg ma m y m A tω ω ωΣ = − = = − = − , ( )( )

2

2 sin

bF m g y

m g A t

ω

ω ω

= −

= −

La separación ocurre cuando se alcanza 0bF = , ya que esta fuerza no puede ser negativa. Por tanto, no hay despegue si

20 sinbF g A t tm

ω ω> ⇒ > ∀ , 2 .g Aω> 2 20 sinb d dF g y A tω ω ω= ⇒ = =

b) Se produce despegue donde (yd) y cuando (td):

2 7,5 cm2 2d

g g Ayg Aω

= = = = , 1 1arcsin 45,8 ms2 6 2dt g A

πω

= = = .

Page 129: XXIII olimpiadas españolas de física, enunciados y soluciones de las pruebas propuestas - pag 214.pdf

8

R.

XVII OLIMPIADA ESPAÑOLA DE FÍSICA. TERUEL 2006

c) Gráficas

Fuerza del pistón sobre la bolita vs posición

-2

0

2

4

6

8

10

0,0 0,2 0,4 0,6 0,8 1,0

Posición del pistón, y/A

Fuer

za, F

/m (m

/s^2

)

Posiciones en función del tiempo

-0,5

-0,4

-0,3

-0,2

-0,1

0,0

0,1

0,2

0,3

0,0 0,1 0,2 0,3 0,4 0,5 0,6 0,7 0,8 0,9 1,0

tiempo, t/T

posi

ción

y (m

)

posición de la bola posición del pistón

Despegue

Despegue

Reencuentro

Page 130: XXIII olimpiadas españolas de física, enunciados y soluciones de las pruebas propuestas - pag 214.pdf

9

R.

XVII OLIMPIADA ESPAÑOLA DE FÍSICA. TERUEL 2006

Problema Teórico 4. La pompa flotante 4/60 Puntos Un niño sopla pompas de agua jabonosa, y se observa que algunas de ellas flotan

momentáneamente en el aire en calma, que suponemos en condiciones normales. a) Encontrar la delgadez o espesor relativo de tales pompas, definido como la relación

espesor / radio. b) Determinar la masa líquida de una de ellas de 5 cm de radio.

Se supone que la masa molecular media del aire es 29, que la temperatura interior de una pompa es la normal del cuerpo humano, 37 ºC, y que la densidad del agua jabonosa es igual a la del agua pura.

Nota: No hay que tener en cuenta el efecto de la tensión superficial del líquido.

Solución

a)

( )

00

0

3 20 40 0 3

0

50 0

Peso Empuje ;

1 1 1 4

1, 291 0,12 ; 5,17·103 3000

Tl T

l T lm

l

TMPm V VRT T

TPm V V R R RM R T T T

T RR R RT R

ρρ ρ ρρ

ρ ρ ρ π ρ π

ρρ

= → + = = =

⎛ ⎞⎛ ⎞= − = − = − = ∆⎜ ⎟⎜ ⎟ ⎝ ⎠⎝ ⎠

∆⎛ ⎞∆ = − = =⎜ ⎟⎝ ⎠

b) ( ) ( ) ( )2 2 5

4 3 3 6

4 4 5,17·10

2·10 ·10 0,0517 27,6·10 ; 86,8 mg

l l l lm V S R R R R R

m

ρ ρ ρ π πρ

π π

− −

= = ∆ = ∆ =

= = =

Problema Teórico 5. Propagación de un rayo laser. 6/60 Puntos Un rayo de luz laser es una onda electromagnética que se puede considerar

monocromática y unidimensional, representada por su vector eléctrico (módulo) ( ) ( )0 0 0, sinE t x E t k xω= − . Se propaga en el aire ( 1n ) e incide normalmente sobre la

superficie de una lámina de caras planas y paralelas de índice de refracción n. Suponiendo que el rayo reflejado y el refractado tienen la misma intensidad, a) Determinar las funciones de ondas ( ) ( ), y ,r tE t x E t x , de la onda reflejada y de la

transmitida, respectivamente.

b) ¿Cuál es el valor máximo de Et después de atravesar la lámina, si su espesor es 1 mmd = y su coeficiente de absorción es 3 -110 mβ = .

c) La onda transmitida en la primera cara y la reflejada en la segunda, interfieren en su recorrido común en el interior de la lámina. Con relación a la onda resultante de esta

Page 131: XXIII olimpiadas españolas de física, enunciados y soluciones de las pruebas propuestas - pag 214.pdf

10

R.

XVII OLIMPIADA ESPAÑOLA DE FÍSICA. TERUEL 2006

interferencia, ¿a qué distancia, xmax, de la cara posterior de la lámina, se encuentran los puntos de máxima amplitud?

d) Encontrar la expresión del ángulo de incidencia, i, para el que el rayo reflejado y el refractado son normales entre sí.

Ayuda: La intensidad de una onda electromagnética es proporcional al cuadrado del valor máximo del campo E.

Solución

a)

( ) ( ) ( ) ( )

2 010 02

00 0 0

0 00 0 0 0

Siendo , y, en general, , .2

Además, , , ,

, = sin y , sin2 2

r t r t

tr t r t

r t

EI I I I E E E

cv k k k nkn v c n

E EE t x t k x E t x t nk x

ω ωω ω ω

ω ω

= = ∝ = =

= = = = = = =

+ = −

b) 3 32 10 ·10

,max 0 0 02 2 2,max 02

0 0

; 0, 432 2 2

d dt x

t tot

E E E EI e E E e e e EI E e

β ββ

−− − −−= = = = = = =

c) max max0 0

2 22 ; , 1, 2,3,...m mx m x m mk nk nkπ π πλ= = = = =

d) 2

sin sin sin cos , arctansin cos

i ri n t i n i i n

r t t iπ

= ⎫⎪= ⇒ = =⎬⎪+ = ⇒ = ⎭

Page 132: XXIII olimpiadas españolas de física, enunciados y soluciones de las pruebas propuestas - pag 214.pdf

11

R.

XVII OLIMPIADA ESPAÑOLA DE FÍSICA. TERUEL 2006

Prueba experimental Características de un pequeño motor 18/60 Puntos

Objetivo y fundamento Se trata de determinar algunas características de un pequeño motor de corriente

continua, del tipo de los utilizados en juguetería. Un motor de corriente continua es, básicamente, una bobina de hilo conductor

arrollada en torno a un eje, que interacciona con un campo magnético producido por un imán u otra bobina auxiliar. Mediante un generador (por ej., una batería), se hace pasar una corriente eléctrica continua por la bobina. Su interacción con el campo magnético consiste en un sistema de fuerzas sobre el eje y, por tanto, sobre lo que esté mecánicamente unido a él (la carga del motor), haciéndolo girar.

Desde el punto de vista energético, el motor recibe energía eléctrica mediante la corriente que se hace pasar por la bobina, y realiza trabajo mecánico mediante la rotación de su eje. Sin embargo, alguna energía se disipa por efecto Joule en el hilo con el que está construida la bobina.

Mientras el fundamento teórico es simple, la realización práctica está sujeta a una gran cantidad de condicionantes, entre ellos la geometría del conjunto, para optimizar sus características, especialmente su rendimiento energético.

Materiales suministrados - Motor de corriente continua - Polea para el motor (se

inserta a presión en el eje del motor). Su diámetro es de 6,0 ± 0,1 mm

- Portapilas para las dos pilas - Brida para sujetar el motor - 3 resistencias de 15 Ω

- 4 pinzas para las sondas de los multímetros. - Listón largo de madera - Escuadra metálica para sujetar el listón - Sargento para sujetar la escuadra a la mesa - Hilo de 0,1 mm de diámetro - Arandelas (que se usan como pesas). Las 4

arandelas grandes tienen una masa de 11,6 g, y la pequeña, de 5,9 g.

- Cinta métrica - Cronómetro - 2 multímetros - Clip - 2 pilas de 1,5 V - 3 pinzas metálicas - Tijeras

Montaje (I) Montaje mecánico del motor Se sujeta el motor mediante una brida,

que, a su vez, se sujeta mediante dos pinzas en la parte superior del listón de madera. El listón de madera se coloca vertical, apoyado en el borde de la mesa, y se sostiene mediante una escuadra sujeta al listón con una pinza. Un sargento aprieta la

Page 133: XXIII olimpiadas españolas de física, enunciados y soluciones de las pruebas propuestas - pag 214.pdf

12

R.

XVII OLIMPIADA ESPAÑOLA DE FÍSICA. TERUEL 2006

escuadra contra la mesa (ver figura y contraportada). Montaje eléctrico del circuito del motor

(circuito I, ver contraportada) Se colocan dos pilas en serie en el portapilas, que se

coloca detrás del listón. Se puede sujetar aprisionando los cables con una de las pinzas que aguantan la brida con el motor. Estas pilas son el generador que se usará para hacer funcionar el motor.

Se conecta un polímetro, empleado como voltímetro en

la escala de 20 V de corriente continua, para medir la tensión en los terminales del motor.

Se conecta uno de los terminales del portapilas a un terminal del motor (doblando los cables entre sí). El otro terminal del portapilas se conecta a uno del polímetro funcionando como amperímetro en la escala de 200 mA de corriente continua. Bastará con tocar el terminal del motor con el otro terminal del amperímetro para alimentar el motor y hacer que gire. El contacto tiene que ser firme para evitar variaciones en la intensidad.

Comprobación del montaje El motor debe girar al tocar el cable con la sonda

del amperímetro, y los instrumentos deben indicar la tensión y la intensidad.

Determinación de la resistencia interna R del motor Si el motor no se deja girar se comporta como una resistencia, R, correspondiente al

hilo que forma el bobinado. Puesto que el valor de R es pequeño, la medida directa de esta resistencia con el polímetro puede dar un error apreciable. Por ello, es conveniente un procedimiento basado en tomar varias medidas de intensidad y voltaje en un circuito que incluye el motor.

Modo de operación (circuito II) El motor se alimenta con una batería,

conectándolo en serie con un conductor de resistencia Rext. Con distintos valores de Rext, se obtienen distintas intensidades en el motor, para ello se utiliza, en cada caso, una combinación distinta (en serie o en paralelo) de 3

Circuito I

Circuito II

R

Page 134: XXIII olimpiadas españolas de física, enunciados y soluciones de las pruebas propuestas - pag 214.pdf

13

R.

XVII OLIMPIADA ESPAÑOLA DE FÍSICA. TERUEL 2006

resistencias de 15 Ω, para lo cual se enrollan sus terminales. Para cada valor de Rext se miden la intensidad y la tensión en el motor, sujetando el eje del motor para impedir que gire.

Medidas 1. Hacer una tabla de valores Rext ,V e I correspondiente a las medidas realizadas en

el circuito anterior. Puesto que el motor bloqueado se comporta como una resistencia, la ley de Ohm permite calcular su valor, R.

2. Hacer la representación gráfica V−I. 3. A partir de ella, calcular el valor de la resistencia R.

Montaje (II) Se hace pasar un hilo de una longitud similar a la del listón por la polea de plástico y se anuda, con el nudo en la parte interior, tal como muestra la figura (ver contraportada). Puede ser conveniente utilizar un clip para ayudar a pasar el hilo por los agujeros. La polea se inserta a presión en el eje del motor. Se ha de asegurar que el motor está sujeto con su eje horizontal, de modo que al hacerlo funcionar, el hilo se enrolle a lo largo de toda la anchura de la polea.

Se dobla el clip para hacer un portapesas que permita colgar las arandelas que se utilizarán como pesas. Se cuelga el portapesas de un lazo en el extremo inferior del hilo.

Relación entre la fuerza electromotriz ε’ y la velocidad angular ω del motor Cuando gira un motor, además del efecto principal (la fuerza de interacción

electromagnética), se tiene un efecto secundario inseparable del anterior: una bobina (muchas espiras acopladas) girando en un campo magnético produce una fuerza electromotriz (fem) inducida que, de acuerdo con la ley de Faraday-Lenz, se opone al paso de la corriente. Por esta razón, en los motores se habla de fuerza contraelectromotriz, que se designa con el símbolo ε’. Aquí se aprovecha la experiencia con un dispositivo técnico para hacer una comprobación científica.

Como en cualquier dispositivo práctico que efectúa transformaciones de energía, interesa el máximo rendimiento, entendido como la relación entre el beneficio obtenido (la energía o trabajo desarrollado) y el coste necesario para obtenerlo (la energía o trabajo suministrado).

Según el esquema del circuito III, la tensión V entre los terminales del motor cumple:

Circuito III

← Generador Motor →

Page 135: XXIII olimpiadas españolas de física, enunciados y soluciones de las pruebas propuestas - pag 214.pdf

14

R.

XVII OLIMPIADA ESPAÑOLA DE FÍSICA. TERUEL 2006

IRV += 'ε Modo de operación Se define previamente la altura h a la que el motor elevará la carga, y se anota este

valor. Se cuelgan distintas pesas del hilo y, al cerrar el circuito tocando un terminal del

motor con un terminal del amperímetro, el motor gira, enrolla el hilo y levanta la pesa una altura h.

Medidas Combinando las pesas del modo apropiado se puede conseguir una serie de masas

creciente m (se puede ignorar la masa del clip). 4. Hacer una tabla con los valores de la masa empleados y, para cada uno de ellos,

con las medidas obtenidas de: a) el tiempo que tarda subir la altura h b) la tensión V en los terminales del motor c) la intensidad I que circula Es conveniente medir separadamente (repitiendo la elevación) el tiempo, y la tensión e

intensidad, ya que resulta muy complicado poder hacer las tres medidas simultáneamente. Las medidas del tiempo con el cronómetro son la principal fuente de error en el

experimento, por ello, se sugiere tomar 5 medidas del tiempo para cada pesa. Para la intensidad y la tensión, 3 medidas son suficientes. Tomar más medidas de estas magnitudes, que resultaría deseable para un mejor tratamiento de los errores, puede suponer que no se disponga de tiempo suficiente para completar todas las tareas que se proponen.

Cálculos y gráficas 5. La tabla se ha de completar calculando, para cada una de las masas utilizadas, los

correspondientes valores de: • La fuerza contraelectromotriz, ε’, • la velocidad v con que se levanta la pesa (que se supondrá constante durante cada

ascenso), • el error ∆v en la velocidad, • la potencia mecánica desarrollada por el motor, Pm = mgv, • la potencia eléctrica suministrada al mismo, Pe = IV • el rendimiento de la transformación de energía, Re = Pm / Pe.

6. A partir de los valores tabulados se hará la representación gráfica de la fuerza contraelectromotriz del motor ε’ en función de la velocidad v con que asciende la pesa. Para simplificar la representación, así como para realizar el apartado 7, se puede considerar que los errores en los valores de ε’ son insignificantes frente a los errores de v.

Page 136: XXIII olimpiadas españolas de física, enunciados y soluciones de las pruebas propuestas - pag 214.pdf

15

R.

XVII OLIMPIADA ESPAÑOLA DE FÍSICA. TERUEL 2006

7. Se determinará la constante de proporcionalidad, k, entre ε’ y la velocidad angular ω con que gira el motor, así como el error ∆k en el valor de k.

8. A partir de los valores tabulados se hará la representación gráfica del rendimiento en función de la masa. Se determinará la masa para la cual el rendimiento es máximo. Soluciones Datos obtenidos en una realización correcta del experimento

Determinación de la resistencia interna, R, del motor

1. Tabla de valores medidos Rext, V e I. Usando escalas de multímetro de 20 V y 200

mA para V e I, respectivamente, se obtiene la tabla adjunta.

2. Representación gráfica V − I

3. Resistencia del hilo R (pendiente de la recta): • 8,8R = Ω , mínimos cuadrados Excel.

• 8,9R = Ω , buscando “a mano” la recta que mejor ajusta a los puntos.

• Considerar correcto 8,8 0,1R = ± Ω .

Rext (ohmios) I (A) V (V)

15 0,1185 1,04 30 0,0756 0,66 7,5 0,1635 1,44

22,5 0,0924 0,81 45 0,0558 0,49

Gráfica V- I para obtener la resistencia del hilo del motor

y = 8,8032x - 0,0021R2 = 1

-0,2

0

0,2

0,4

0,6

0,8

1

1,2

1,4

1,6

0 0,02 0,04 0,06 0,08 0,1 0,12 0,14 0,16 0,18

I (A)

V(V

)

Page 137: XXIII olimpiadas españolas de física, enunciados y soluciones de las pruebas propuestas - pag 214.pdf

16

R.

XVII OLIMPIADA ESPAÑOLA DE FÍSICA. TERUEL 2006

Relación entre la fuerza contraelectromotriz ε’ y la velocidad angular ω del motor 4. Tabla con los valores de la masa empleados y, para cada uno de ellos, con las

medidas obtenidas de: a) El tiempo que tarda en subir la altura h, b) la tensión V en los terminales del motor, c) la intensidad I que circula.

Tiempo (s) Intensidad (mA) Tensión (V) Masa (g) t1 t2 t3 t4 t5 I1 I2 I3 V1 V2 V3

5,9 1,87 1,81 1,78 1,84 1,78 33,9 32,8 33,3 3,08 3,08 3,07

11,6 1,93 1,93 1,91 1,87 1,91 48,9 47,3 47,2 3,02 3,04 3,03

17,5 2,06 2,03 2,03 2,03 2,09 59,5 60,1 59,4 3,00 3,00 3,00

23,2 2,16 2,13 2,16 2,19 2,16 73,4 73,1 72,8 2,96 2,96 2,96

29,1 2,38 2,43 2,31 2,38 2,41 86,3 85,7 86,2 2,92 2,93 2,92

34,8 2,59 2,59 2,53 2,63 2,56 98,6 98,5 98,3 2,89 2,88 2,89

40,7 2,78 2,84 2,85 2,81 2,82 110,2 111,3 111,7 2,85 2,85 2,84

46,4 3,10 3,06 3,06 3,10 3,03 125,4 125,5 126,0 2,82 2,83 2,81

52,3 3,37 3,32 3,47 3,50 3,56 137,2 138,5 136,4 2,78 2,77 2,77

La altura h es un valor particular de cada experimentador. En las medidas que siguen,

h = 0,960 ± 0,005 m

Page 138: XXIII olimpiadas españolas de física, enunciados y soluciones de las pruebas propuestas - pag 214.pdf

17

R.

XVII OLIMPIADA ESPAÑOLA DE FÍSICA. TERUEL 2006

5. Tabla de los valores medios experimentales y cálculos

Valores medios experimentales Cálculos

Masa m (g)

Tiempo medio,

t (s)

Error tiempo, ∆t (s)

Intens. media, I (mA)

Tensión media, V (V)

Veloc., mv h t=

(m/s)

Error veloc.,

∆v (m/s)

∆V en R V IR∆ = (V)

Fcem '

V IRε =

(V)

Potencia mP

mgv=

(W)

PotenciaeP IV= (W)

Rendto.e

m e

RP P

=

5,9 1,82 0,039 33,3 3,08 0,529 0,011 0,293 2,78 0,031 0,103 0,30

11,6 1,91 0,024 47,8 3,03 0,503 0,006 0,421 2,61 0,057 0,136 0,39

17,5 2,05 0,027 59,7 3,00 0,469 0,006 0,525 2,47 0,080 0,172 0,45

23,2 2,16 0,021 73,1 2,96 0,444 0,004 0,643 2,32 0,101 0,209 0,47

29,1 2,38 0,045 86,1 2,92 0,403 0,008 0,757 2,17 0,115 0,243 0,46

34,8 2,58 0,037 98,5 2,89 0,372 0,005 0,867 2,02 0,127 0,274 0,45

40,7 2,82 0,027 111,1 2,85 0,340 0,003 0,977 1,87 0,136 0,305 0,43

46,4 3,07 0,030 125,6 2,82 0,313 0,003 1,106 1,71 0,142 0,340 0,40

52,3 3,44 0,098 137,4 2,77 0,279 0,008 1,209 1,56 0,143 0,364 0,38

6. Representación gráfica de la fuerza contraelectromotriz del motor ε’ en función de la velocidad v con que asciende la pesa.

E' en función de v

y = 4,7685x + 0,2341R2 = 0,9977

0,00

0,50

1,00

1,50

2,00

2,50

3,00

0,000 0,100 0,200 0,300 0,400 0,500 0,600

v(m/s)

E'(V

)

Page 139: XXIII olimpiadas españolas de física, enunciados y soluciones de las pruebas propuestas - pag 214.pdf

18

R.

XVII OLIMPIADA ESPAÑOLA DE FÍSICA. TERUEL 2006

7. Constante de proporcionalidad, k, entre ε’ y la velocidad angular ω con que gira el motor, así como el error ∆k en el valor de k.

En primer lugar se calcula la pendiente, p, de la recta en la representación ε’–v Ajustada con Excel: p = 4,77 Vs/m. A partir de las rectas de pendiente máxima y mínima (4,96 y 4,49 Vs/m,

respectivamente) hechas “a mano” teniendo en cuenta las barras de error de v , se obtiene 4,7 0,3 Vs/m.p = ± La constante k pedida, se relaciona con la pendiente p de la gráfica y con el

radio r de la polea:

' 'k prv r

ε εω

= = = ,

resultando Vs/rad 001,0014,010)05,03)(3,07,4( 3 ±=⋅±±= −k 8. Representación gráfica del rendimiento en función de la masa.

Rendimiento en función de la masa

0,00

0,05

0,10

0,15

0,20

0,25

0,30

0,35

0,40

0,45

0,50

0 10 20 30 40 50 60

m(g)

Ren

dim

ient

o

El rendimiento es máximo para una masa 23 2 gm = ± .

Page 140: XXIII olimpiadas españolas de física, enunciados y soluciones de las pruebas propuestas - pag 214.pdf

Almería, 1 a 4 de abril de 2005

XVI OLIMPIADA ESPAÑOLA

DE FÍSICA

EJERCICIOS PROPUESTOS

Y SOLUCIONES

Page 141: XXIII olimpiadas españolas de física, enunciados y soluciones de las pruebas propuestas - pag 214.pdf

XVI OLIMPIADA ESPAÑOLA DE FÍSICA

2 Universidad de Almería, 1 a 4 de abril de 2005

En la noria del parque de atracciones Prueba teórica 1 (12 puntos)

Un buen día soleado, Pedro y Ana se divierten en el parque de atracciones. Montados en la noria, a Pedro se le plantean muchas cuestiones de Física, como las siguientes.

El radio de la noria es R = 10 m, y el motor que la acciona tiene una potencia P = 2 kW. Se desprecian todos los rozamientos.

a) ¿Cuál es la masa móvil de la noria, suponiendo que toda ella se encuentra en la periferia, si tarda 10 s en adquirir una velocidad angular de 0,2 rad /s?

b) Cuando Pedro pasa por la posición más alta, se le cae una moneda del bolsillo. ¿Cuánto tardará en llegar al suelo? ¿A qué distancia cae, medida desde la vertical del punto más alto?

c) ¿Qué velocidad angular debería tener la noria para que Ana se sintiera ingrávida, y en qué posición le ocurriría esto?

d) Describe el movimiento de Ana visto desde la cabina de Pedro, diametralmente opuesta a la suya. ¿Cuáles son su velocidad y aceleración en función del tiempo? ¿Cómo es el movimiento de Pedro respecto a Ana?

e) En un momento en que el motor esta desconectado y la noria girando a una velocidad de 0,1 rad/s, Pedro sube a la noria en marcha dando un pequeño salto lateral desde el andén. Si pesa 50 kg, ¿cuál es la variación de velocidad angular de la noria debida al salto?

Solución a) El trabajo realizado por el motor, con una potencia constante, es W = Pt. Por otra parte,

también es igual al aumento de energía cinética de la noria. Si se supone que toda la masa está en la periferia, la energía cinética de la noria es la misma que la de una partícula que se mueve con la velocidad lineal periférica, v = ωR.

212cW E mv= ∆ = ⇒ 4

2 2 2 2 2

2 2 2·2000·10 1,0·100,2 ·10

W Ptm kgv Rω

= = = =

b) El tiempo de caída es independiente de la componente horizontal de la velocidad inicial, por tanto es el mismo que si la moneda se dejara caer desde el reposo:

2 2·20 2,0 s

9,8ht

g= = =

La distancia horizontal recorrida por la moneda en su caída es la de un movimiento uniforme con velocidad igual a la de la noria en el tiempo de caída:

2·2 4,0 mh hd v t= = = .

Page 142: XXIII olimpiadas españolas de física, enunciados y soluciones de las pruebas propuestas - pag 214.pdf

XVI OLIMPIADA ESPAÑOLA DE FÍSICA

Universidad de Almería, 1 a 4 de abril de 2005 3

c) La sensación de ingravidez se da cuando el peso es la única fuerza que actúa, por lo que mg = ma, siendo a la aceleración normal o centrípeta, ω2R, de un punto de la noria:

2 2 mg m R R gω ω= ⇒ = ⇒ 9,8 0,99 rad/s.10

gR

ω = = =

d) Es también un movimiento circular con la misma velocidad angular (tarda el mismo tiempo en dar una vuelta completa), pero con un radio igual a la distancia que los separa, 2R.

2 22 4 m/s; 2 0,8 m/sAna Anav R a Rω ω= = = =

El movimiento de Pedro respecto a Ana es el mismo que el de Ana respecto a Pedro, salvo la velocidad, que es del mismo módulo y de signo contrario:

( ) ( )22 4 / ; 2 0,8 /Pedro Pedrov R m s a R m sω ω= = − = =− −

e) Como el salto es lateral, se conserva el momento angular de la noria: L = Cte. ⇒ Lf = Li. El módulo del momento angular de la noria, supuesta toda su masa en la periferia, es el módulo de la cantidad de movimiento de una partícula periférica equivalente (con toda la masa) multiplicada por el radio. Los subíndices i, f, denotan inicial y final, es decir, antes y después del salto, respectivamente:

( ) ( )( ) ( ) ,i f i f f iMMv R v R MM m M m M m

ω ω ω ω= ⇒ = ⇒ =+ ++

4500,1 5,0·10 rad/s110050i i

mMM mM m

ω ω ω −−⎛ ⎞∆ = = = − = −−⎜ ⎟ ++⎝ ⎠.

Page 143: XXIII olimpiadas españolas de física, enunciados y soluciones de las pruebas propuestas - pag 214.pdf

XVI OLIMPIADA ESPAÑOLA DE FÍSICA

4 Universidad de Almería, 1 a 4 de abril de 2005

Propulsión iónica Prueba teórica 2 (10 puntos)

Introducción

La nave SMART-1, de la Agencia Espacial Europea, entró en la zona de predominio de la gravedad lunar el 15 de Noviembre de 2004. Catorce meses antes fue colocada en órbita terrestre por un lanzador convencional Ariane-5, y desde entonces ha viajado hacia la Luna impulsada por su motor iónico. Actualmente, este mismo motor la está frenando lentamente para situarla en una órbita lunar baja, en la que desarrollará su misión científica. Durante todo ese tiempo (cerca de año y medio) el motor iónico ha funcionado de forma casi continua, aunque con un empuje muy débil.

La inmensa ventaja de este sistema de propulsión es que puede mantenerse durante un tiempo muy largo y, a igualdad de propelente gastado, se alcanza una velocidad final mucho más elevada que con un motor cohete tradicional, de forma que su rendimiento es unas diez veces superior. El secreto radica en que la velocidad de expulsión de los iones es muy superior a la de los gases de combustión de un motor cohete, que no suele pasar de los 3 km/s (con un tiempo de encendido de tan solo unos 10 minutos). Además la energía eléctrica necesaria para el funcionamiento de un motor iónico se extrae de la luz solar mediante paneles fotovoltaicos.

El principio de funcionamiento del motor iónico es sencillo: átomos neutros de gas Xenón entran a una primera cámara donde son ionizados mediante un haz de microondas, que arrancan un electrón a cada átomo. Los iones Xe+ son conducidos mediante un campo eléctrico débil a otra cámara, donde un intenso campo eléctrico los acelera hasta una alta velocidad y los expulsa al espacio exterior. Para crear este campo, se establece una diferencia de potencial entre dos rejillas, a través de las cuales pasan los iones. La nave debe permanecer eléctricamente neutra, por lo que un circuito capta los electrones producidos en la ionización y,

F

Xe

Microondas

Iones Xe+

Electrones

Cámara de ionización

Región de aceleración de los

iones Expulsión y recombinación de los

iones Depósito de “combustible”

∆V

Page 144: XXIII olimpiadas españolas de física, enunciados y soluciones de las pruebas propuestas - pag 214.pdf

XVI OLIMPIADA ESPAÑOLA DE FÍSICA

Universidad de Almería, 1 a 4 de abril de 2005 5

mediante un cátodo hueco, los expulsa también al espacio, donde se recombinan con los iones Xe+ formando de nuevo gas neutro y emitiendo un bello resplandor azulado. Los electrones son muchísimo más ligeros que los iones, por lo que su efecto de propulsión es irrelevante.

Problema La SMART-1 inició su viaje con una carga de unos 80 kg de gas Xenón. A su régimen normal de funcionamiento, el motor iónico tiene un consumo C = 0,10 kg/día de Xe, lo que le da una autonomía superior a dos años. La diferencia de potencial entre las rejillas aceleradoras es ∆V = 1,3 kV.

Calcule: a) La velocidad de expulsión de los iones, ve. b) La fuerza de empuje del motor, F. c) El número de iones expulsados por segundo N, y la intensidad de la corriente iónica

expulsada, I. d) La potencia eléctrica gastada en la ionización del gas, Pi. e) El consumo de potencia eléctrica del conjunto del motor, P.

Datos:

Masa media de un átomo de Xenón: kg101802 25−⋅= ,m

Energía de primera ionización: E+ = 12,13 eV

Carga del electrón: C106021 19−⋅= ,e

Solución Nota: los resultados numéricos se expresan con dos cifras significativas porque los valores de

∆V y C están dados así.

a) La energía cinética de cada ión es igual al trabajo efectuado sobre una carga e en la caída de potencial ∆V.

VevmE ec ∆== 2

21 ⇒

212 /

e mVe

v ⎟⎠

⎞⎜⎝

⎛ ∆= m/s1044 4⋅= ,ve

Page 145: XXIII olimpiadas españolas de física, enunciados y soluciones de las pruebas propuestas - pag 214.pdf

XVI OLIMPIADA ESPAÑOLA DE FÍSICA

6 Universidad de Almería, 1 a 4 de abril de 2005

b) El empuje es igual al momento lineal de la masa expulsada por segundo

212 /

e mVeCvCF ⎟

⎠⎞

⎜⎝⎛ ∆

==

Masa expulsada por segundo:

C = 0,10 kg/día = kg/s101571 6−⋅, ⇒ N1015 2−⋅= ,F

c) Número de átomos de Xe expulsados por segundo: mCN = ⇒ 18 15,3 10 sN −= ⋅

Carga expulsada por segundo: eNI = ⇒ m

eCI = A850,I =

d) En régimen estacionario, el número de iones expulsados por segundo coincide con el número N de átomos ionizados por segundo. La potencia (energía por segundo) consumida en la ionización será

+= ENPi ⇒ mEC

Pi

+= W10=iP

e) Para estimar la potencia empleada en la aceleración de los iones se puede razonar al menos de dos maneras:

i) Potencia eléctrica directa:

m

eCVIVPa ∆=∆=

ii) Energía cinética de los iones expulsados por segundo

a cP N E N e V V I= = ∆ = ∆

La potencia total es entonces

ia PPP += ⇒ ( )++∆= EVemCP W1011 3⋅= ,P ,

que es muy similar a los 1,19 kW declarados por la ESA.

Page 146: XXIII olimpiadas españolas de física, enunciados y soluciones de las pruebas propuestas - pag 214.pdf

XVI OLIMPIADA ESPAÑOLA DE FÍSICA

Universidad de Almería, 1 a 4 de abril de 2005 7

Un tapón oscilante Prueba teórica 3 (8 puntos)

Suponga que dispone de un matraz como el que se representa en la figura 1, constituido por un bulbo de volumen V, lleno de aire a presión atmosférica p0, y de un cuello de sección S, en el que hay un tapón de masa m y longitud L que puede deslizar sin rozamiento.

En un cierto instante se empuja ligeramente el tapón una distancia x (x << L) y, como consecuencia, el volumen de aire dentro del bulbo experimenta una disminución ∆V y su presión un aumento ∆p (figura 2). A continuación el sistema se libera y se observa que el tapón realiza un movimiento oscilatorio armónico.

a) Demuestre que la frecuencia angular, ω , de las oscilaciones del tapón es κ

ωmVS 2

= ,

donde κ es el coeficiente de compresibilidad del aire, que se define como pV

V ∆∆

−=1κ .

b) El valor del cociente ∆V / ∆p, y por tanto el del coeficiente de compresibilidad, depende del tipo de transformación termodinámica que experimenta el gas. En el caso del aire que contiene el bulbo, la transformación puede considerarse adiabática, puesto que es despreciable la cantidad de energía (calor) que intercambia con su entorno en el breve tiempo de cada expansión o compresión.

En las transformaciones adiabáticas, las variaciones de la presión y del volumen están relacionadas en la forma

0=∆+∆ pVVpγ ,

donde γ es el llamado índice adiabático.

Determine en este caso la dependencia de κ con la presión y el índice adiabático.

c) Considere en adelante que la densidad del tapón es igual a la del aire, lo que equivale a decir que es un tapón de aire, y que la experiencia se realiza a una temperatura T = 300 K.

V, p0 p0

L

S

Fig. 1

Tapón

V - ∆V p0 + ∆p

S

Fig. 2

Page 147: XXIII olimpiadas españolas de física, enunciados y soluciones de las pruebas propuestas - pag 214.pdf

XVI OLIMPIADA ESPAÑOLA DE FÍSICA

8 Universidad de Almería, 1 a 4 de abril de 2005

Suponiendo que el aire es un gas perfecto, obtenga la expresión de la frecuencia angular de las oscilaciones, ω´, en función de γ, S, T, V, L, R (constante de los gases) y de M (masa molar del aire).

d) Con los siguientes datos numéricos, calcule el valor de la frecuencia, f, de las oscilaciones.

Volumen del bulbo: V = 1,0·10-3 m3 Sección y longitud del cuello del matraz: S = 1,0·10–4 m2, L = 5,0·10-2 m Índice adiabático del aire: γ = 1,4 Constante de los gases: R = 8,314 J mol–1K–1 Masa molar del aire: M = 2,9·10–2 kg mol–1

Solución

a) Cuando el tapón se introduce una distancia x, la disminución del volumen de aire dentro del bulbo es

SxV =∆ (1)

La fuerza que actuará sobre el tapón (figura 3) es pSF ∆=

por lo que su aceleración será

pSm

a ∆=1 (2)

Despejando ∆p de la expresión del coeficiente de compresibilidad, sustituyendo en (2) y teniendo en cuenta (1),

xmVSa

κ

2−= ,

que es la aceleración de un movimiento oscilatorio armónico, con frecuencia angular

κ

ωmVS 2

= (3)

b) De acuerdo con la expresión que proporciona el enunciado, ∆V / ∆p = – V / γp, el coeficiente de compresibilidad es

pV

V ∆∆

−=1κ ⇒

pγκ 1

= (4)

c) Si el tapón es de aire, su masa es SLm ρ= , donde ρ es la densidad del aire.

( )Spp ∆+0 Sp0

Fig. 3

Page 148: XXIII olimpiadas españolas de física, enunciados y soluciones de las pruebas propuestas - pag 214.pdf

XVI OLIMPIADA ESPAÑOLA DE FÍSICA

Universidad de Almería, 1 a 4 de abril de 2005 9

Teniendo en cuenta la expresión (4), la frecuencia angular (3) de oscilación toma la forma

pLVS γ

ρω =′2 (5)

Por otra parte, si llamamos mB a la masa de aire del bulbo, la densidad del aire es

V

mB=ρ (6)

y a su vez, si en el bulbo hay n moles de aire nMmB = (7) en la que M es la masa molar del aire. Por la ley de los gases

nRTV

p 1= (8)

Llevando (6), (7) y (8) a la expresión (5), queda finalmente TLVM

RSγω =′ .

d) Con los datos numéricos del enunciado se obtiene,

rad/s1094 2⋅=′ ,ω

Por tanto, la frecuencia de las oscilaciones es π

ω2

′=f Hz78≈f

Esta frecuencia es del orden de la del sonido que produce el “descorche” de una botella de características similares a la que se ha descrito.

Page 149: XXIII olimpiadas españolas de física, enunciados y soluciones de las pruebas propuestas - pag 214.pdf

XVI OLIMPIADA ESPAÑOLA DE FÍSICA

10 Universidad de Almería, 1 a 4 de abril de 2005

Prueba experimental El vuelo del capacillo1 (15 Puntos)

Cuando un cuerpo se mueve en el seno de un fluido con velocidad v, su movimiento se ve frenado por una fuerza, llamada de resistencia. Supongamos que esta fuerza depende de v en la forma F c vγ= , donde c y γ son constantes que dependen de la forma y tamaño del cuerpo y de las características del fluido (densidad y viscosidad).

En este problema se va a estudiar experimentalmente la caída en el aire de uno o varios capacillos superpuestos, de manera que cambia la masa del objeto que cae, pero no su forma (aerodinámica). Como podrá comprobar, la velocidad de caída es prácticamente uniforme desde el momento en que se sueltan. a) Las únicas fuerzas que actúan sobre los capacillos cuando caen son su peso y la resistencia

del aire. Teniendo esto en cuenta, y llamando n al número de capacillos y m0 a la masa de cada uno, obtenga una expresión analítica para la velocidad uniforme (límite) de caída, vn. Transforme esta expresión y demuestre que la dependencia entre el logaritmo de tn y el logaritmo de n, donde tn es el tiempo de caída de n capacillos desde una altura h, es de la forma

( ) ( ) ( )hc

gmntn lnln1ln1ln 0 +⎟

⎞⎜⎝

⎛−−=

γγ.

b) Basándose en la expresión anterior y en sus medidas experimentales de tiempos de caída, tn, para n = 1, 2, 3 y 4, determine el valor de γ en este experimento.

c) Haga una estimación de la incertidumbre (margen de error) del valor de γ obtenido. d) A partir de las medidas experimentales que considere oportunas, determine la masa M de

la arandela metálica que se suministra. Para responder a esta pregunta, tenga en cuenta que la masa de un capacillo es m0 = 0,25 g.

Material del que dispone:

• Cuatro capacillos de igual masa, m0. • Un cronómetro. • Papel milimetrado. • Una arandela de masa M desconocida. • Cinta adhesiva.

Comentarios y sugerencias:

• Dispone inicialmente de cuatro capacillos superpuestos y bien empaquetados. Comience midiendo el tiempo de caída de este conjunto y, después, vaya retirándolos con cuidado.

1 Una prueba experimental muy similar a ésta fue propuesta en la IX Olimpiada Iberoamericana de Física, que se celebró en La Habana (Cuba) en septiembre de 2003. Por deferencia a sus autores, hemos mantenido el nombre de capacillos para los moldes de magdalena.

Page 150: XXIII olimpiadas españolas de física, enunciados y soluciones de las pruebas propuestas - pag 214.pdf

XVI OLIMPIADA ESPAÑOLA DE FÍSICA

Universidad de Almería, 1 a 4 de abril de 2005 11

• La forma de los capacillos influye notablemente en el valor de la fuerza de resistencia, a través de la constante c. Por tanto, debe tener especial cuidado de manipularlos sin deformarlos.

• Deje caer los capacillos con la parte abierta hacia arriba (al revés que un paracaídas). • Déjelos caer desde la máxima altura h que le permita su brazo. No necesita conocer el

valor de h; basta con que sea siempre el mismo (aproximadamente). • Emplee la cinta adhesiva para fijar la arandela al interior de la base de un capacillo. • En la hoja de respuestas tiene una tabla para ordenar sus medidas y los datos obtenidos. • Emplee el papel milimetrado para realizar las gráficas y ajustes que considere oportunos.

Solución a) Una vez alcanzada la velocidad uniforme (límite) de caída

Aceleración nula ⇒ γncvgmn =0 ⇒

γ/10

⎟⎟⎠

⎞⎜⎜⎝

⎛=

cgmn

vn

Tomando logaritmos en la expresión anterior

( ) ( ) ⎟⎟⎠

⎞⎜⎜⎝

⎛+=

cgm

nvn0ln1ln1ln

γγ (1)

El tiempo de caída desde una altura h es n

n vht = (2)

Sustituyendo (2) en (1) se obtiene inmediatamente

( ) ( ) ( )hc

gmntn lnln1ln1ln 0 +⎟⎟

⎞⎜⎜⎝

⎛−−=

γγ

Por tanto, se obtiene una dependencia lineal entre ln(tn) y ln(n), con pendiente –1/γ . Tabla de datos

N Tiempos de caída (s) (s)nt (s)nt∆ ( )nln ( )ntln n∆

1 1,67 1,68 1,65 1,73 1,68 1,69 1,73 1,75 1,69 1,73 1,700 0,0207 0,0000 0,5306 0,0122

2 1,25 1,31 1,30 1,29 1,23 1,33 1,30 1,25 1,32 1,25 1,283 0,0221 0,6931 0,2492 0,0172

3 1,07 1,10 1,08 1,11 1,07 1,15 1,10 1,12 1,12 1,13 1,105 0,0167 1,0986 0,0998 0,0151

4 0,99 0,96 0,98 0,99 1,04 1,00 1,05 1,00 0,99 0,98 0,998 0,0173 1,3863 -0,0020 0,0173

1+arand. 1,13 1,17 1,19 1,17 1,18 1,12 1,17 1,18 1,17 1,10 1,158 0,0194 0,1467

Page 151: XXIII olimpiadas españolas de física, enunciados y soluciones de las pruebas propuestas - pag 214.pdf

XVI OLIMPIADA ESPAÑOLA DE FÍSICA

12 Universidad de Almería, 1 a 4 de abril de 2005

b) En la tabla de la hoja de respuestas se tabulan diez medidas de cada tiempo de caída (desde h ≈ 2,1 m), junto con su valor medio y su incertidumbre ∆tn, calculada como el doble del error típico (nivel de confianza 95 %). En las últimas columnas aparecen los datos para la gráfica: ln(n), ln(tn) y la incertidumbre ∆n propagada a este último dato, calculada en la forma

( )[ ]n

nnn t

tt

∆=∆=∆ ln .

En la siguiente gráfica se presenta el ajuste de estos puntos a una recta (realizado con

Excel). Se obtiene ( ) ( ) 5255,0ln3852,0ln +−= ntn (3)

Por tanto

3852,01=γ ⇒ 60,2=γ

c) La incertidumbre de γ se puede estimar a partir de las rectas que se ajustan razonablemente a los puntos experimentales con pendientes máxima y mínima, teniendo en cuenta las incertidumbres de dichos puntos.

Midiendo sobre la gráfica anterior se obtiene2

2 Se llega a un resultado similar calculando analíticamente el error típico de la pendiente.

y = -0,3852x + 0,5255

-0,1

0,0

0,1

0,2

0,3

0,4

0,5

0,6

0,0 0,2 0,4 0,6 0,8 1,0 1,2 1,4 1,6

ln(n )

ln( t n

)

Page 152: XXIII olimpiadas españolas de física, enunciados y soluciones de las pruebas propuestas - pag 214.pdf

XVI OLIMPIADA ESPAÑOLA DE FÍSICA

Universidad de Almería, 1 a 4 de abril de 2005 13

⎪⎭

⎪⎬⎫

≈⇒−≈

≈⇒−≈

5,240,0

7,237,0

minmin

maxmax

γ

γ

p

p ⇒

−=∆

2minmax γγ

γ 1,0≈∆γ

En total, el resultado completo para γ sería 2,6 0,1γ = ± .

d) En la última fila de la tabla de la hoja de respuestas se presentan diez tiempos de caída de

un capacillo con la arandela en su interior. Se obtiene un tiempo medio de caída t = 1,158 s.

Sustituyendo en la ecuación (3) de la recta ajustada y operando, se obtiene que este tiempo de caída corresponde a un número equivalente de capacillos n = 2,67 Por tanto, la masa de la arandela es3 M = 1,67 m0 ⇒ g42,0=M

Teniendo en cuenta únicamente la incertidumbre del tiempo de caída y propagando errores, se obtiene M = (0,42 ± 0,03) g.

Nota:

En las páginas que siguen se dan las tablas de respuestas pedidas en las pruebas.

3 La masa real de la arandela empleada en las medidas es M = 0,40 g.

-0,1

0,0

0,1

0,2

0,3

0,4

0,5

0,6

0,0 0,2 0,4 0,6 0,8 1,0 1,2 1,4 1,6

ln(n )

ln(t

n)

Page 153: XXIII olimpiadas españolas de física, enunciados y soluciones de las pruebas propuestas - pag 214.pdf

XVI OLIMPIADA ESPAÑOLA DE FÍSICA

14 Universidad de Almería, 1 a 4 de abril de 2005

Tablas de respuestas Problema Teórico 1. La noria

Problema teórico 3. El tapón oscilante

Pregunta / puntos

Fórmulas básicas usadas

Resultado analítico Resultado numérico

a)

3 puntos

212, ,

,c cW E E mv

W Pt v Rω= ∆ =

= =

2 2 2

2 2W Ptmv Rω

= = 41,0·10 kgm =

b)

2 puntos

212 , h hh gt d v t= = 2 ; h

ht d Rtg

ω= = 2,0 s;4,0 m

td==

c)

2 puntos 2

Peso ,

centrípeta

mgF m Rω

=

= g

Rω =

0,99 rad/s En el punto más altoω =

d)

3 puntos

2,v R a Rω ω= =

( ) ( )

2

2

2 ; 2

' 2 ; ' 2

v R a R

v R a R

ω ω

ω ω

= =

= =− −

2

Circular uniforme, , 2

4 m/s; 0,8 m/sIdéntico, salvo signos:

4 m/s; 0,8 m/s

Ana Ana

Ana Ana

Pedro Pedro

R R

v a

v a

ω ω= =

= =

= − =

e)

2 puntos

.;

CteL mvR v Rω

== =

L 1i

MM m

ω ω ⎛ ⎞∆ = −⎜ ⎟+⎝ ⎠

45,0·10 rad/sω −∆ = −

Pregunta / puntos

Fórmulas básicas usadas Resultado analítico Resultado numérico

a)

3 puntos

pSF ∆=

xmamF 2ω−== κ

ωmVS 2

=

b) 1,5 puntos

pV

pV

γ−=

∆∆

pγκ 1

=

c)

2,5 puntos

LSm ρ= ; V

mB=ρ

nRTpV =

TLVM

RSγω =′

d)

1 punto π

ω2

′=f Hz78≈f

Page 154: XXIII olimpiadas españolas de física, enunciados y soluciones de las pruebas propuestas - pag 214.pdf

XVI OLIMPIADA ESPAÑOLA DE FÍSICA

Universidad de Almería, 1 a 4 de abril de 2005 15

Problema Teórico 2. El motor iónico

Pregunta / puntos

Fórmulas básicas Resultado analítico Resultado numérico

a)

2 puntos VemvE ec ∆== 2

21

212 /

e mVe

v ⎟⎠

⎞⎜⎝

⎛ ∆= m/s1044 4⋅= ,ve

b)

2,5 puntos dtdpF = , p Mv= , dMC

dt=

212 /

mVe

CF ⎟⎠

⎞⎜⎝

⎛ ∆= N1015 2−⋅= ,F

c)

1,5 puntos tQI

∆∆

= mCN = ,

meC

I = 18 15,3 10 sN −= ⋅

A850,I =

d)

2 puntos

+= ENPi m

ECPi

+=

W10=iP

e)

2 puntos

ia PPP += (a: aceleración)

IVPa ∆= ( )++∆= EVe

mCP

W1011 3⋅= ,P

Prueba experimental

Pregunta / Puntos Resultados analíticos Resultados numéricos

a)

1 Punto

γ/10

⎟⎟⎠

⎞⎜⎜⎝

⎛=

cgmn

vn

( ) ( ) ( )hc

gmntn lnln1ln1ln 0 +⎟⎟

⎞⎜⎜⎝

⎛−−=

γγ

b)

8 Puntos

6,2=γ

c)

2 Puntos

1,0≈∆γ

d)

4 Puntos

g42,0=M

Page 155: XXIII olimpiadas españolas de física, enunciados y soluciones de las pruebas propuestas - pag 214.pdf

XVI OLIMPIADA ESPAÑOLA DE FÍSICA

16 Universidad de Almería, 1 a 4 de abril de 2005

“La alegría de ver y entender es el más perfecto don de la

naturaleza”

Page 156: XXIII olimpiadas españolas de física, enunciados y soluciones de las pruebas propuestas - pag 214.pdf

T-1) EL MUÑECO SALTARÍN (10 puntos) Un muñeco saltarín se puede aproximar mediante un muelle de longitud l0 = 5,0 cm de masa despreciable, unido a un disco de masa m = 40 g, y el conjunto está ensartado en un largo eje vertical fijo al suelo. El juguete se deja caer desde una altura h0 = 25 cm (ver figura, arriba). Después de tocar el suelo, el muelle se comprime hasta una longitud l = 2,0 cm (abajo). Se desprecian todos los rozamientos.

1. Describir el movimiento del disco,

mediante:

a) Una explicación cualitativa analizando el balance energético. (1,5 puntos)

b) Una representación gráfica de su altura h en función del tiempo t. (1,5 puntos)

c) Las ecuaciones del movimiento, indicando el significado de los términos incluidos en ellas. (2 puntos)

2. Calcular:

a) La constante elástica del muelle. (2 puntos)

b) La máxima energía potencial almacenada en el muelle. (1 punto)

c) El periodo del movimiento. (2 puntos)

XIV OLIMPIADA ESPAÑOLA DE FÍSICA

Cuenca, abril de 2003

h0 l0

l

Page 157: XXIII olimpiadas españolas de física, enunciados y soluciones de las pruebas propuestas - pag 214.pdf

T-1) El muñeco saltarín (solución) 1.a) El movimiento es periódico no amortiguado, ya que todas las fuerzas son conservativas y la energía total permanece constante. Inicialmente cae convirtiendo energía potencial gravitatoria en cinética. Tras hacer contacto con el muelle, sigue descendiendo, transformando ahora energía cinética y potencial gravitatoria en potencial elástica. Después de alcanzar la máxima compresión del muelle, con energía cinética nula, recupera invirtiéndose el movimiento. El ciclo de este movimiento consta de dos partes: mientras el muelle está apoyado en la mesa es armónico simple (sistema muelle-masa); cuando no lo está, es uniformemente acelerado (descenso y ascenso en el campo gravitatorio terrestre).

1.b) Sea h la coordenada posición. Suponiendo que en t = 0 se suelta el muñeco en la posición h = h0 (figura), la gráfica es un arco de parábola hasta que el muelle hace contacto con el suelo. En ese momento (posición h = l0) se inicia un movimiento de tipo armónico simple, primero comprimiéndose el muelle hasta la posición h = l, y después elevándose hasta despegar del suelo en la misma posición en que hizo contacto, h = l0. Se describe así medio ciclo (aproximadamente) del M.A.S1. A continuación se realiza el movimiento de subida hasta la altura inicial h = h0, repitiéndose el ciclo.

1.c)2 Movimiento armónico simple: tAth ωsin)( = , amplitud A = l0 − l = ∆l, y frecuencia angular ω.

Caida libre: 221

0)( gthth −= , h0 altura inicial y g aceleración de la gravedad.

2.a) De la conservación de la energía en un descenso completo,

=∆

−=⇒∆+= 2

02120

)(2 ,l

lhmgklkmglmgh 2,0·102 N m−1

2.b) =∆= 212max lkE 90·10−3 J.

2.c) ( )

=−

+

=+=

glh

kmTTT caidaarm

0012

222π 4,5·10−1 s.

1 El análisis riguroso es algo más complicado, difícilmente realizable por los alumnos de la

olimpiada: el centro de oscilación está debajo del punto h = l0, a una distancia del mismo δl = mg/k = 1,93 mm (deformación estática del muelle). Por tanto, la parte armónica del ciclo total dura más de medio periodo del M.A.S.

2 No se espera que los alumnos establezcan las constantes iniciales y los intervalos de validez de cada expresión analítica.

t

h0

0

h

∆l l0

Tarm

221

0)( gthth −=

Tcaíd

arm

tlth ωsin)( ∆=

l

Page 158: XXIII olimpiadas españolas de física, enunciados y soluciones de las pruebas propuestas - pag 214.pdf

T-2) LA FUERZA DE LORENTZ (10 puntos) Un móvil se desliza por un plano inclinado sobre el que pende el conductor cilíndrico AC a una distancia h de la línea de máxima pendiente, tal como indica la figura. Aunque se trata de un objeto macroscópico, se considera al móvil simplemente como una partícula de masa m, cargada positivamente con una carga q. El plano inclinado se considera de material aislante, de manera que el móvil no se descargue, y el conductor cilíndrico de longitud infinita y de radio despreciable para simplificar el efecto de la corriente que lo recorre. En este problema se analiza el movimiento bajo la acción combinada de la gravedad, el campo electromagnético y el rozamiento, partiendo del reposo desde la parte superior del plano inclinado. 1. Suponga despreciable el rozamiento entre el móvil y el plano. Por el conductor

circula una corriente eléctrica de intensidad I en el sentido de A hacia C. a. Haga un esquema indicando claramente el campo creado por la corriente y su

valor en la posición del móvil. (1 punto) b. Haga un nuevo esquema indicando todas las fuerzas que actúan sobre el móvil

mientras desciende por el plano e indique el valor de cada una de ellas.(1 punto) c. Obtenga las ecuaciones del movimiento del móvil. (1 punto) d. Demuestre que cuando la velocidad del móvil alcanza un cierto valor, éste se

“despega” del plano. Obtenga el valor vd de esta velocidad. (2 puntos) e. Determine la distancia d recorrida por el móvil antes del despegue. (0.5 puntos)

2. Considere en adelante que existe un coeficiente de rozamiento µ entre móvil y plano, Determine la nueva velocidad de despegue v’d. (1.5 puntos) 3. Suponga ahora que, también con rozamiento, se repite la experiencia partiendo del reposo, pero con la corriente circulando en sentido contrario, de C hacia A.

a. Demuestre que el móvil tiende a alcanzar una velocidad uniforme (velocidad límite). Obtenga el valor vl de esta velocidad. (2 puntos)

b. Haga una gráfica en la que se muestre de forma cualitativa la velocidad del móvil en función del tiempo. (1 punto)

XIV OLIMPIADA ESPAÑOLA DE FÍSICA

Cuenca, abril de 2003

I

θ

A

h

q C

Page 159: XXIII olimpiadas españolas de física, enunciados y soluciones de las pruebas propuestas - pag 214.pdf

T-2) La fuerza de Lorentz (solución) 1.a. El problema tiene simetría cilíndrica. Por la ley de Ampere: B 2π h = µ0 I

B= µ0 I/(2π h).

El sentido de B, dado por el de la corriente I y la regla de signos adecuada, es

crucial en lo que sigue. 1.b. En el esquema P es el peso, FL la fuerza de Lorentz y N la reacción normal del plano.

Nótese que FL es normal al plano y hacia arriba, pues se supone que el móvil desciende por el plano con cierta velocidad v y FL es perpendicular a v y a B

P = mg, FL = q v B, N=P cosθ - FL

Page 160: XXIII olimpiadas españolas de física, enunciados y soluciones de las pruebas propuestas - pag 214.pdf

1.c. En el esquema:

∑ fx = Ft, por lo tanto m g senθ = m a ⇒ a = g senθ

∑ fy = 0, es decir, que FL -P cosθ +N =0 ⇒ N = mg cosθ -q v B,

1.d. Obsérvese que, mientras que la aceleración del móvil es independiente de la acción del campo magnético, la reacción del plano N disminuye a medida que la velocidad aumenta. Llegará un momento en que la velocidad alcanzada es tal que N = 0. A partir de ese momento, el móvil deja de actuar sobre el suelo y... despega.

N = 0 ⇒ mg cosθ -q v B = 0 ⇒ vd = m g cosθ /(q B)

1.e. Por ejemplo: td = vd/a = m cotθ /(q B) ⇒ d=1/2 a td2 = m2 g cotθ cosθ/(2 q B)

2. Hay una nueva fuerza: el rozamiento R, que es proporcional a la reacción del plano y actúa en sentido contrario a Ft.

Se tiene: R = µ N = µ (mg cosθ -q v B). Las demás fuerzas no se alteran.

Ft – R = m a ⇒ a = g (senθ - µ cosθ) + µ (q B/m) v

N + FL - P cosθ= 0 ⇒ N= m g cosθ - q v B,

El despegue (cuando N = 0) se produce para v = vd = m g cosθ/(q B), El mismo valor que sin rozamiento (valor que se tardará mas tiempo en alcanzar).

Page 161: XXIII olimpiadas españolas de física, enunciados y soluciones de las pruebas propuestas - pag 214.pdf

Sustituyendo vd en la ecuación de la aceleración a, se obtiene la aceleración en el momento de despegue: ad = g senθ, la misma que si no hubiera rozamiento.

3. Al cambiar el sentido de la corriente, cambia el de B y, por tanto, el de FL. El nuevo valor de la fuerza de rozamiento es R= µ (mg cosθ +q v B)

3.a. Ft – R = m a ⇒ a = g (senθ - µ cosθ) - µ (q B/m) v

N - FL - P cosθ = 0 ⇒ N= m g cosθ + q v B,

Las ecuaciones del apartado anterior indican que cuando v crece desde 0, la aceleración a va disminuyendo hasta alcanzar a=0. A partir de ese momento cesa la aceleración y el movimiento continúa a velocidad constante vL.

Cuando se alcanza la velocidad límite vL

0 = g (senθ - µ cosθ) - µ (q B/m) vL ⇒ vL = m g (senθ - µ cosθ)/(µ q B).

3.b. Gráfica de la velocidad en función del tiempo.

Nótese que al ser la curva v(t), su pendiente da la aceleración en cada instante, a(t) = dv(t)/dt. Su valor inicial, a(0) = tg α, corresponde a la situación de velocidad nula en que se anula la fuerza de Lorentz.

Page 162: XXIII olimpiadas españolas de física, enunciados y soluciones de las pruebas propuestas - pag 214.pdf

T-3) PROBLEMA MÚLTIPLE (10 puntos)

• MECÁNICA (3 puntos) Los cuerpos celestes, salvo por algunas extensas masas rocosas en planetas de tipo terrestre, están constituidos por materiales sueltos sólidos, líquidos o gaseosos. Se dice por ejemplo que los asteroides, buena parte de la Luna, etc., no son otra cosa que un “apilamiento de escombros” mantenidos juntos por la fuerza de la gravedad. Sin embargo ésta puede ser insuficiente si los cuerpos tienen movimiento de rotación. Para simplificar, supongamos que un asteroide está constituido por dos fragmentos esféricos (de densidad ρs y radio rs) en rotación alrededor de su punto de contacto. Siendo G la constante de gravitación universal, se pide calcular en función de ρs y rs :

1. La fuerza de atracción entre los dos fragmentos del asteroide. (1,5 puntos) 2. La velocidad de rotación máxima ωm que puede tener el asteroide sin que se

disgregue. (1,5 puntos)

• ÓPTICA (3 puntos) En una cubeta se vierte un líquido de índice de refracción n = 1,5 . Se hace incidir un rayo láser sobre la superficie libre del líquido bajo un ángulo de incidencia α . Si suponemos que el índice de refracción del aire es la unidad, se pide:

1. Para qué ángulo de incidencia α el ángulo de refracción β resultaría ser

β = α/2 . (1,5 puntos) 2. La situación de la pregunta anterior no se cumple para todos los líquidos.

Para qué valores del índice de refracción n del líquido, dicha relación es imposible. (1,5 puntos)

______________________________________________________________________

XIV OLIMPIADA ESPAÑOLA DE FÍSICA

Cuenca, abril de 2003

Page 163: XXIII olimpiadas españolas de física, enunciados y soluciones de las pruebas propuestas - pag 214.pdf

• EFECTO FOTOELÉCTRICO (4 puntos) Al iluminar una superficie metálica A con luz de diferentes longitudes de onda λ, se arrancan electrones que salen despedidos con de diferentes energías. En el experimento se miden los potenciales de frenado1 V de los electrones producidos, cuyos valores se muestran en la tabla (en micras y voltios) λ (µm) 0.579 0.546 0.492 0.436 0.405 0.366 V (V) 0.24 0.32 0.62 0.93 1.15 1.48

1. Represente el potencial de frenado V en función de la frecuencia υ de la luz incidente. (1,5 puntos)

2. A partir de la representación gráfica realizada determine: a. La frecuencia umbral υ0. (1 punto) b. El cociente (h/e) entre la constante de Planck y la carga eléctrica. (1 pto) c. El trabajo Φ de extracción de los electrones del metal. (0,5 puntos)

Tómese c = 3x108 m s-1 y h = 6,62x10-34 J s

Fig. Dispositivo experimental: El dispositivo de la izquierda del dibujo es una fuente de alimentación variable responsable del campo E. El de la derecha un amperímetro que detecta el cese de la corriente de electrones entre A y C.

1 Se denomina de frenado a aquel potencial a partir del cual cesa la llegada de electrones al cátodo C.

Page 164: XXIII olimpiadas españolas de física, enunciados y soluciones de las pruebas propuestas - pag 214.pdf

T-3) Problema múltiple (soluciones) MECÁNICA Masa de cada fragmento: m = (4/3) π rs

3 ρs 1. Atracción gravitatoria: FG = G m2 /(2 rs)2

2. Fuerza disgregante: Fc = m ω2 rs

Equilibrio Fc = FG ⇒ ω2 = G m / (4 rs

3) ⇒ ω = (G π ρs / 3)1/2 ÓPTICA 1. (sin 2β) / sin β = n ⇒ 2 cos β = n ⇒ β = arc cos (n/2) = 41 o ⇒ α = 82 o

2. Valores posibles: α ≤ 90o ⇒ β ≤ 45 o ⇒ n = 2 cos β ≥ 21/2 ⇒ Valores imposibles: n < 21/2 = 1,41 EFECTO FOTOELÉCTRICO 1. La gráfica pedida es del tipo

Page 165: XXIII olimpiadas españolas de física, enunciados y soluciones de las pruebas propuestas - pag 214.pdf

2. El efecto fotoeléctrico se rige por la ecuación de Einstein:

Φ−= νheV Por lo tanto tg α = (h/e) y υ0 = (Φ/h). Los resultados obtenidos de la gráfica, tomando c = 0.3x109 m s-1 y h = 6.62x10-34 J s, son:

2.a. υ0 = 4.62x1014 Hz.

2.b. El cociente (h/e) = 3.99x10-15 J s / C

2.c. Trabajo de extracción Φ = 3.06x10-19 J s = 1.91 eV

Page 166: XXIII olimpiadas españolas de física, enunciados y soluciones de las pruebas propuestas - pag 214.pdf

PRUEBA EXPERIMENTAL: RESISTENCIA Y RESISTIVIDAD (10 puntos) OBJETIVO Medida de la resistencia eléctrica y estimación de la resistividad de un conductor. El conductor empleado está formado principalmente por grafito (mina de lápiz). MATERIAL A EMPLEAR Pila de 9 V, potenciómetro, voltímetro, resistencia de 27 Ω, cables de conexión con pinzas de cocodrilo y clavijas, cinta adhesiva (para fijación de elementos sobre la mesa) y mina de lápiz (resistencia a medir). DESCRIPCIÓN DE MATERIAL: EL POTENCIÓMETRO El potenciómetro consta de una resistencia fija entre los extremos A y B, en nuestro caso el valor nominal es 100 Ω (este valor puede variar ligeramente en cada unidad). Los extremos A y B en el dispositivo suministrado son sus patillas laterales, las cuales se han representado en el dibujo como pequeños círculos. Además hay un cursor móvil, representado en la figura mediante el círculo señalado por C, consistente en un contacto que puede moverse a lo largo de la resistencia. En el dispositivo suministrado el cursor C es la patilla central. A medida que el cursor móvil se desplaza desde A hasta B cuando giramos el eje del potenciómetro, la resistencia

Potenciómetro

entre A y C aumenta (y la resistencia entre B y C disminuye). De este modo, el potenciómetro puede emplearse como resistencia variable, conectándolo entre A y C. En consecuencia, puede utilizarse para variar la corriente en un circuito en serie. ______________________________________________________________________

XIV OLIMPIADA ESPAÑOLA DE FÍSICA Cuenca, abril de 2003

100 Ω

A

B

C

Page 167: XXIII olimpiadas españolas de física, enunciados y soluciones de las pruebas propuestas - pag 214.pdf

MONTAJE DEL CIRCUITO Utilizando el material suministrado, móntese el circuito esquematizado en el dibujo, donde R simboliza la resistencia eléctrica de la mina de lápiz que se pretende medir. Las resistencias del circuito se fijarán sobre la mesa utilizando cinta adhesiva. Precaución: la conexión de la mina de lápiz usando pinzas de cocodrilo ha de hacerse cuidadosamente, ya que este material es muy frágil. Además, debe procurarse que la pinza de cocodrilo sujete la mina cerca de sus extremos. MEDIDAS Una vez fijado el montaje sobre la mesa usando cinta adhesiva para sujetar algunos elementos, mida con una tira de papel milimetrado la longitud de mina de lápiz que queda libre entre las dos pinzas de cocodrilo que la sujetan . Para las medidas eléctricas se irá variando la resistencia entre A y C girando en pequeños pasos sucesivos el eje del potenciómetro, de modo que se haga variar la corriente circulante. En cada uno de estos pasos (utilice un número suficiente de ellos) use el voltímetro para medir: 1º) La diferencia de potencial entre los extremos de la resistencia de 27 Ω (la cual se considerará como resistencia patrón), y permitirá conocer la intensidad. 2º) La diferencia de potencial entre los extremos de la resistencia R a determinar. Para tomar estas lecturas conecte al voltímetro los cables que llevan clavija por un extremo y pinza de cocodrilo por el otro, y en cada una de ellas cambie las pinzas de cocodrilo de la resistencia de 27 Ω a las pinzas que sujetan la mina de lápiz (no se recomienda morder directamente con las pinzas la propia mina).

100 Ω

A

B C

27 Ω

9 V

R

Page 168: XXIII olimpiadas españolas de física, enunciados y soluciones de las pruebas propuestas - pag 214.pdf

PREGUNTAS 1º) Presente una tabla de tres columnas: en las dos primeras los valores de las diferencias de potencial medidas en la resistencia de 27 Ω y en la resistencia R, respectivamente, y en la tercera el valor de la intensidad de corriente circulante, en amperios: Efectúe las medidas. (hasta 5 : 0,5 puntos; de 6 a 10: 1 pto.; de 11 a 15: 1,5 ptos.) Controle el número de cifras significativas de los datos. (número correcto: 1 punto) 2º) Represente gráficamente la diferencia de potencial entre los extremos de la mina de lápiz frente a la intensidad de corriente. Utilice para esto una hoja de papel milimetrado, escogiendo las escalas más adecuadas. Obténgase de esta gráfica el valor de la resistencia R y su cota de error (Esto debe hacerse utilizando la representación gráfica, no mediante ajuste de calculadora programable): Dibujo de la gráfica. (1,5 puntos) Calcule R a partir de la gráfica indicando claramente el procedimiento seguido.(1,5 pto) Cálculo de la cota del error de R. (1,5 puntos) 3º) Para un objeto en forma cilíndrica, de longitud L y sección recta S, la relación entre su resistencia eléctrica R y su resistividad ρ es:

SLR ρ=

Utilizando el valor de la resistencia eléctrica R obtenido anteriormente, el dato de longitud L medida con el papel milimetrado, y tomando como valor del diámetro de la mina (2.00±0.05) mm, estímese el valor de la resistividad del material de la mina de lápiz (grafito mezclado con otros materiales) con su cota de error correspondiente: Cálculo de la resistividad. (1 punto) Cota del error de la resistividad. (1 punto) Presentación de resultados y claridad de exposición del método. (1 punto)

Page 169: XXIII olimpiadas españolas de física, enunciados y soluciones de las pruebas propuestas - pag 214.pdf

Prueba experimental: Resistencia y resistividad (Modelo de solución) EJEMPLO DE MEDIDAS V1 V2 I 1.43 0.31 0.053 1.67 0.36 0.062 1.96 0.42 0.073 2.43 0.52 0.090 2.78 0.59 0.103 3.13 0.66 0.116 3.29 0.69 0.122 3.49 0.73 0.129 3.80 0.80 0.141 4.09 0.86 0.151 4.39 0.92 0.163 4.67 0.98 0.173 V1 = voltaje medido en la resistencia de 27 Ω (V) V2 = voltaje medido en la mina de lápiz (V) I = intensidad (A)

0.04 0.06 0.08 0.10 0.12 0.14 0.16 0.18

0.3

0.4

0.5

0.6

0.7

0.8

0.9

1.0

Correlación r = 0.99994∆b = 0.02 Ωb = 5.56 Ω

Pendiente experimental:

V (v

oltio

s)

I (amperios)

Page 170: XXIII olimpiadas españolas de física, enunciados y soluciones de las pruebas propuestas - pag 214.pdf

Ejemplos de gráficas y cálculo de la resistencia R y su cota de error, según criterios complementarios.

Page 171: XXIII olimpiadas españolas de física, enunciados y soluciones de las pruebas propuestas - pag 214.pdf

Modelo de cálculo de la resistividad y cota de error: R = (6,3±0,3) Ω L = (11,8±0,2) cm D = (2,00±0,05) mm R = ρ L / S ⇒ ρ = R S / L = R π D2 / 4L = 1,68x10-4 Ω m Incertidumbre de ρ con Propagación cuadrática:

∆ρ = 1,2x10-5 Ω m ⇒ ρ = (1,7±0,1)x10-4 Ω m Con Propagación lineal:

∆ρ = 1,9x10-5 ⇒ ρ = (1,7±0,2)x10-4 Ω m

22222

+

+

=

∆LL

DD

RR

ρρ

+

+

=

∆LL

DD

RR 2

ρρ

Page 172: XXIII olimpiadas españolas de física, enunciados y soluciones de las pruebas propuestas - pag 214.pdf

XV OLIMPIADA ESPAÑOLA DE FÍSICA

Problema C1. Curva de calentamiento (4 Puntos) El diagrama adjunto muestra la temperatura de un cuerpo de masa en

función del calor que se le ha transferido.

kgm 2,0=

a) Calcule los calores específicos del sólido y del líquido, así como el calor de fusión. (1.5 Puntos)

b) Calcule la variación de energía interna de la sustancia entre los estados correspondientes al origen y al punto A, suponiendo que el sólido y el líquido tienen la misma densidad. (1 Punto)

c) Trace una curva de calentamiento entre las mismas temperaturas inicial y final (sobre los mismos ejes) que corresponda a valores mitad de los calculados en el apartado a). (1.5 Puntos)

T (ºC)

Vigo, España. 19 a 22 de Marzo de 2004

SOLUCIÓN

a)

kgkJkgKcalmQqmqQ

KkgJKkgKcalc

KkgJKkgKcal c

TmQcTcmQ

ff

l

s

/10·18.41002.0

20

·/10·18.4·/1050·2.0

10 :líquido

·/698·/167.0150·2.05 :sólido

5

3

====⇒=

⎪⎪⎩

⎪⎪⎨

==≅

=≅≅=

Δ=⇒Δ=

b) kJKcalQWQU i 3,14635 ==Σ=−=Δ

c) En rojo.

Q (Kcal) 10 20 30

sólido

líquido

50

Q (Kcal) 10 20 30

A

200

150

100

Page 173: XXIII olimpiadas españolas de física, enunciados y soluciones de las pruebas propuestas - pag 214.pdf

XV OLIMPIADA ESPAÑOLA DE FÍSICA

Problema C2. Central hidroeléctrica (4 Puntos) Un salto de agua de altura y cuyo caudal es , alimenta un

generador de corriente continua, que produce energía eléctrica a una tensión . El rendimiento energético,

m20=h smG /0,5 3=

V300=V η , de la operación es del 60 %.

a) ¿Cuál es la intensidad, I , de la corriente obtenida? (1.5 Puntos)

b) Si el generador funciona ininterrumpidamente en las condiciones descritas, ¿cuál es su producción energética anual, E , en ? (1 Punto) hkW ·

c) Calcule la variación de energía interna por unidad de tiempo, tU ΔΔ , y la elevación de temperatura, TΔ , del agua tras su caída. (1.5 Puntos)

SOLUCIÓN

a) tΔ

Δ=

mecánica)Energía ( eléctrica Potencia η

( ) AV

GghItVgh

tmghVI 310·96.16.06.0)(

=⇒Δ

Δ=

ΔΔ

=Δρρη

b) ( ) hkWhorastVIE ·10·15.5)(24·365· (kW)10·588 63 ==ΔΔ=

c) ( ) ( )

⎪⎪⎩

⎪⎪⎨

=ΔΔ

=ΔΔΔΔ

Δ=

ΔΔ

−=ΔΔ

− KGc

tUtmctUT

WtVgh

ttU

3

5

10·7.18

10·92.34.0mecánica) Energía(1

ρ

ρη

Vigo, España. 19 a 22 de Marzo de 2004

Page 174: XXIII olimpiadas españolas de física, enunciados y soluciones de las pruebas propuestas - pag 214.pdf

XV OLIMPIADA ESPAÑOLA DE FÍSICA

Problema C3. Dos partículas cargadas en un campo magnético (4 Puntos)

Dos partículas con cargas iguales y de signo opuesto se mueven en una región libre de campos con velocidades paralelas entre si, en el mismo sentido y de módulos diferentes. Las partículas penetran en otra región en la que existe un campo magnético uniforme, B , cuya dirección es perpendicular al plano de sus trayectorias.

Las partículas se encuentran después de haber girado ángulos º150,º90 21 == ϕϕ .

Despreciando la interacción entre las partículas en toda su trayectoria, calcule la relación entre:

a) Sus masas, 12 mm . (1.5 Puntos)

b) Los radios de sus órbitas, 12 RR . (1.5 Puntos)

c) Los módulos de sus velocidades, 12 vv . (1 Punto)

SOLUCIÓN

3102·

35 c)

2º30sin b)

53

)(

35

90150

// a)

11

22

1

2

1

221

2

1

1

2

21

2

1

2

===

=⇒=

==⇒

=⇒=

==ΔΔΔΔ

=

RR

vv

RRRR

mm

mqBRmBRq

tt

ωω

ωω

ωωω

ϕϕ

ωω

Vigo, España. 19 a 22 de Marzo de 2004

90º

150º B

1m

2m

1v

Región 1

Región 2

R1

R2

2v

Page 175: XXIII olimpiadas españolas de física, enunciados y soluciones de las pruebas propuestas - pag 214.pdf

XV OLIMPIADA ESPAÑOLA DE FÍSICA

Problema C4. Interferencias de electrones1 (8 Puntos) En la figura se esquematiza un dispositivo experimental para poner de manifiesto que los

electrones, además de ser partículas, se comportan también como ondas. El aparato consta básicamente de tres placas planas cargadas de longitud L que crean campos eléctricos uniformes de módulo E en los espacios intermedios, tal como se indica en la figura. Por la izquierda de las placas (región 1) incide un haz colimado de electrones con velocidad vx paralela a las placas.

_ _ _ _ _ _ _ _ _ _

vx

Región 2

Y

X O

E

E

vx

vy

vy

Pantalla fluorescente

vx

vx

Región 1

L

+ + + + + + + + + +

_ _ _ _ _ _ _ _ _ _

a) Calcule la componente transversal vy de la velocidad de los electrones a la salida de las placas (región 2). Nota: No es necesario el cálculo relativista. (2 Puntos) Datos: L =5 mm; E = 570 V/m ; vx = 1,24·108 m/s; e = 1,60·10-19 C; me = 9,11·10-31 kg.

b) El haz de electrones en la región 1 puede ser considerado como una onda plana. Llamando A a su amplitud y kx a su número de ondas (kx = 2π/λ1, siendo λ1 la longitud de onda asociada), esta onda puede expresarse en un instante dado, t = 0, en la forma:

)(cos)( x1 xkAx =ξ . En la región 2 se superponen las dos ondas planas asociadas a los electrones desviados entre las placas. Por tanto, la onda resultante, en t = 0, puede expresarse en la forma:

)( cos)( cos)( yxyx2 ykxkykxkAy,x −++=ξ .

Determine las dos componentes del vector número de ondas, kx y ky, en la región 2. (3 Puntos)

Dato: h = 6,63·10-34 J s.

c) Se sitúa como indica la figura una pantalla fluorescente, que brilla con una intensidad proporcional a la intensidad de la onda de electrones que llega a cada uno de sus puntos. En la pantalla se detectan franjas de interferencia perpendiculares al plano de la figura. Obtenga una expresión para la distribución de intensidad en la pantalla, I(y), y determine la distancia interfranja. (3 Puntos)

Nota: 2

cos2

cos2coscos βαβαβα −+=+

1 Este problema fue propuesto en la V Olimpiada Iberoamericana de Física que se celebró en Jaca en Septiembre de 2000.

Vigo, España. 19 a 22 de Marzo de 2004

Page 176: XXIII olimpiadas españolas de física, enunciados y soluciones de las pruebas propuestas - pag 214.pdf

XV OLIMPIADA ESPAÑOLA DE FÍSICA

El experimento más bello de la Física

El experimento de interferencia de electrones que pasan a través de una doble rendija

fue realizado por vez primera por Claus Jönsson de la universidad de Tübingen en 1961. En 1976 Giulio Pozzi consiguió repetir el experimento en el Istituto de Física della Materia de Bolonia enviando los electrones uno a uno a través de la rendija, de modo que sólo hubiese un electrón a la vez dentro del aparato y que éstos llegasen e impactasen en la pantalla individualmente (como se muestra en las fotos). El experimento se repitió en Japón en 1989 por investigadores de Hitachi (Akira Tonomura y colaboradores). Hoy en día se ha conseguido reproducir con neutrones, átomos y moléculas tan grandes como los Füllerenos (60C y 70C). En Mayo del 2003 el experimento fue declarado “El experimento más bello de la Física” por los lectores de Physics World.

La intensidad de la fuente es tan débil que los electrones llegan muy distanciados

temporalmente, de modo que nunca hay más de uno en vuelo en el interior del aparato. De este modo ningún electrón puede interferir ni con el anterior ni con el siguiente, sino sólo con él mismo. Conforme se van acumulando los impactos en la pantalla, se van haciendo visibles las características franjas de interferencia que ilustran de manera impresionante la existencia de "ondas" cuánticas cuyo significado es el de amplitudes de probabilidad. Fotos cortesía de Giulio Pozzi.

Puede verse una película del experimento en:

www.lamel.bo.cnr.it/educational/educational.html

Vigo, España. 19 a 22 de Marzo de 2004

Page 177: XXIII olimpiadas españolas de física, enunciados y soluciones de las pruebas propuestas - pag 214.pdf

XV OLIMPIADA ESPAÑOLA DE FÍSICA

SOLUCIÓN

Vigo, España. 19 a 22 de Marzo de 2004

xeyy v

LmeEtav =Δ= m/s10044 3

y ·,v =

a) 2y

2xe

2vvm

hph

+==λ h

vvmk

2y

2xe

22

22 +==

π

λπ

h2 xe

2vm

coskkxπ

α == 112x m10071 −= ·,k

h

2sen ye

2vm

kkyπ

α == 17y m10·49,3 −=k

b) ( ) ( ) ( ) ( )ykcosxkcosAy,xy,xI y2

x222 4=Φ∝

Sobre la pantalla: ( ) ( )ykcosyI y2∝

I máxima para ⇒( ) 1y2 =ykcos πnyk =ny ⇒

yn k

ny π=

Interfranja: eELhv

vmh

kyyi

22x

yeyn1n ===−= +

π; nm090,i =

Page 178: XXIII olimpiadas españolas de física, enunciados y soluciones de las pruebas propuestas - pag 214.pdf

XV OLIMPIADA ESPAÑOLA DE FÍSICA

Problema 5. Pequeñas variaciones orbitales (15 Puntos)

En las maniobras de las naves espaciales en órbita, los cambios que se realizan en las velocidades y alturas son habitualmente muy pequeños comparados con las velocidades y radios orbitales respectivamente. Lo mismo ocurre para las energías, momentos angulares, etc. en juego. En consecuencia, los cálculos involucran valores numéricos muy dispares y para efectuarlos con sencillez, controlando al mismo tiempo la precisión, se deben adoptar ciertas estrategias.

Vigo, España. 19 a 22 de Marzo de 2004

0T

El objetivo de este problema es expresar los cambios que experimentan algunos parámetros de una órbita cuando variamos otros. Veremos que los efectos resultan aproximadamente proporcionales a las variaciones causantes de los mismos cuando unos y otros son muy pequeños.

Como modelo, supondremos que la Tierra es una esfera de radio y que carece de atmósfera. Se sugiere emplear como dato la aceleración de la gravedad en la superficie terrestre: .

kmR 6370=

281,9 −= msg

a) Calcule la velocidad de un satélite que orbitase en torno a la Tierra a ras del suelo, como se muestra en la figura 1. (2 Puntos)

0v

b) Calcule el período de esta órbita. (1 Punto)

Órbita circular baja

Tierra

Fig. 2

h

vvvP δ+=

Tierra

h

Fig. 3

Órbita elíptica

Pr

Ar

Av

Órbita rasante

Tierra

Fig. 1

0v

v

La presencia de la atmósfera hace que las órbitas razonablemente seguras deban situarse por lo menos a una altura de unos 300 km sobre la superficie terrestre (órbitas llamadas “bajas”). Como 300 km son muy pocos frente a los 6370 km del radio terrestre, son posibles ciertos razonamientos aproximados.

c) Para una órbita baja, a una altura h = 300 km, como se muestra en la figura 2 (no está dibujada a escala), calcule de forma aproximada (a primer orden en h) la velocidad orbital y determine la aproxv

Page 179: XXIII olimpiadas españolas de física, enunciados y soluciones de las pruebas propuestas - pag 214.pdf

XV OLIMPIADA ESPAÑOLA DE FÍSICA

diferencia, , entre dicha velocidad orbital y la velocidad de la órbita rasante en

función de la altura h. (2 Puntos) aproxvΔ 0v

d) Efectúe el cálculo numérico exacto de v, determine la diferencia y compárela con la obtenida en el apartado anterior,

0vvv −=Δ

aproxvΔ . (1 Punto)

e) Calcule también la diferencia ΔT entre el periodo exacto de la órbita baja (h = 300 km) con su valor aproximado a primer orden en h. (2 Puntos)

Consideremos ahora la órbita circular baja del apartado anterior. Con un breve encendido de los motores de maniobra de la nave, se incrementa ligeramente el módulo de su velocidad en

vδ , sin modificar su dirección (véase la figura 3). Con esta operación, la nave pasa de la órbita circular a una órbita elíptica y se incorpora a la nueva órbita en su perigeo, a una distancia

del centro de la Tierra, con una velocidad siendo hRrP += vvvP δ+= vv <<δ . El apogeo de la órbita se encontrará a una distancia y en él la velocidad de la nave será , ambas magnitudes estarán relacionadas con sus correspondientes en el perigeo a través de la conservación de la energía mecánica de la nave y de su momento angular,

Ar Av

mvrL = , respecto al centro de la Tierra.

f) Suponiendo que , calcule, en términos de s/mv 10=δ vδ a primer orden de aproximación, la diferencia entre las distancias del centro de la Tierra al apogeo y perigeo de la nueva órbita, es decir . (3 Puntos) PA rrr −=δ

g) En el mismo orden de aproximación que el apartado anterior, calcule la diferencia, Tδ , entre el período de la nueva órbita elíptica T´ y el de la órbita circular inicial, T. (4 Puntos)

Ayuda matemática

Binomio de Newton: ( ) ( ) ...xnnnxx n +−++=+ 212111

Si 1<<ε , podemos despreciar y las potencias más elevadas y escribir 2ε ( ) εε nn +≈+ 11

Esta expresión, que vale también para n no entero, nos da el valor de ( “a primer orden en )nε+1ε ”.

Por ejemplo, a primer orden en ε tenemos: ( ) 211 21 // εε −≈+ −

Aplicación: Si , tendremos , “ a primer orden en b” ab << ( ) ( a/nbaba nn +≈+ 1 )

Vigo, España. 19 a 22 de Marzo de 2004

Page 180: XXIII olimpiadas españolas de física, enunciados y soluciones de las pruebas propuestas - pag 214.pdf

XV OLIMPIADA ESPAÑOLA DE FÍSICA

SOLUCIÓN

a) Orbita circular: 2

20

RMmG

Rm

v= ⇒ v0 = gR = 7,90 ⋅103 ms-1 ⎟

⎠⎞

⎜⎝⎛

=2R

GMg

Rg

Rv

== 00ω ⇒ min84,4 s100652 3

0 =⋅== ,gRT π

b) Órbita baja: hRr += 2

2

2

2

rRg

rGM

rv

== ⇒ r

Rgv2

= .

Utilizando la aproximación a primer orden obtenemos:

⎟⎠⎞

⎜⎝⎛ −=Δ+

RhgRvv approx 2

10

y por tanto, 12 sm1086,1

2−⋅−=−=Δ

Rghvapprox

Por otro lado, el cálculo exacto de Δ = v – vv 0 :

12 sm1080,11 −⋅−=⎟⎟⎠

⎞⎜⎜⎝

⎛−=Δ

rRgRv

La aproximación comete un error del 6% del valor exacto.

2

232

2

gR

rv

rT

π== ⇒

gR

Rh

RhTT

232

23

0 π==Δ ⇒ s105733 2⋅==Δ ,hgR

T π

c) Orbita elíptica

Conservación de la energía mecánica:

A

AP

P rMmGmv

rMmGmv −=− 22

21

21

⇒ 12

vP2 1−

vA2

vP2

⎝ ⎜

⎠ ⎟ = GM

1rP

−1rA

⎝ ⎜

⎠ ⎟

Conservación del momento angular:

AAPP rmvrmv = ⇒ A

P

P

A

rr

vv

= ⇒

Vigo, España. 19 a 22 de Marzo de 2004

Page 181: XXIII olimpiadas españolas de física, enunciados y soluciones de las pruebas propuestas - pag 214.pdf

XV OLIMPIADA ESPAÑOLA DE FÍSICA

( ) ( PAAP

PAA

P rrrr

GMrrrv

−=− 222

2

21 ) ⇒

12

vP2 rA + rP( ) = GM

rA

rP ⇒

( )

PP

PPPA

rGMv

rvGMrr

−=−

2

2

21

Cálculo a primer orden en vδ :

⇒ vvvP δ+= ⎟⎠⎞

⎜⎝⎛ +=

vv

vvPδ

2122 ⇒ ⎟⎠⎞

⎜⎝⎛ +=

vv

GMrv PPδ

212 ⇒

vv

r

vrv

r

vv

rrr P

PP

PAδ

δ

δ

δ 4

21

2=

−=−= ⇒ m1045,34 4

2/3

⋅== vgR

rr P δδ

Semieje mayor de la órbita circular: ; Semieje mayor de la órbita elíptica: Prra ==

( )PA rr'a +=21

Tercera ley de Kepler: 3

3

2

2

a'a

T'T

= ⇒

3/ 23/ 23/ 2

3/ 2 3/ 2

' 1' 22 2

1 1 2 12

a P

P P

P

r ra rT T T Ta r r

r vT T Tr v

δ

δ δ

⎡ ⎤⎛ ⎞ ⎛ ⎞+⎛ ⎞= = = +⎢ ⎥⎜ ⎟ ⎜ ⎟⎜ ⎟⎝ ⎠ ⎝ ⎠ ⎝ ⎠⎣ ⎦

⎛ ⎞ ⎛ ⎞ ⎛= + = + = +⎜ ⎟ ⎜ ⎟ ⎜⎝ ⎠ ⎝⎝ ⎠

3 vv

δ

=

⎞⎟⎠

⇒ svgr

RvvT P 310.88,333 −=== δδδ

En estos cálculos la precisión de la aproximación es elevada porque vv <<δ .

Vigo, España. 19 a 22 de Marzo de 2004

Page 182: XXIII olimpiadas españolas de física, enunciados y soluciones de las pruebas propuestas - pag 214.pdf

Prueba experimental (15 puntos)

Determinación de la distancia focal f de una lente y del índice de refracción n del vidrio del que está hecha

Desde que se inventaron las lentes hace ya cinco siglos, han sido utilizadas tanto para observar objetos muy lejanos (Astronomía) como para observar objetos muy pequeños (Microscopía). En esta prueba experimental se propone comprobar la denominada ley de las lentes y determinar las características fundamentales de una lupa (lente convergente). Material:

− 1 lente convergente de vidrio (lupa con mango desmontable) − 1 vela − 2 soportes de madera para ambas − cinta métrica − pantalla de cartulina − calibre con nonio − 2 hojas de papel milimetrado − pequeño material auxiliar (clips, palillos, papel cello, encendedor,...)

Primer experimento: Comprobación experimental de la ley de las lentes Desde muy pronto, físicos como Galileo y Newton descubrieron una ley que se verifica en las lentes, consecuencia de la ley de la refracción de la luz: “Cuando se forma la imagen de un objeto en una pantalla a través de una lente, la suma de las inversas de la distancia objeto, s, y de la distancia imagen, s’, es una cantidad constante, característica de dicha lente”. En la Figura 1 se indica el esquema del montaje experimental que proponemos, así como las distancias s y s’.

Figura 1: Esquema del montaje experimental.

Operaciones a realizar:

1. (2 puntos) Una vez encendida la vela, colocar la lente a una distancia s y desplazar la pantalla hasta conseguir una imagen nítida de la llama. En estas condiciones medir la

Page 183: XXIII olimpiadas españolas de física, enunciados y soluciones de las pruebas propuestas - pag 214.pdf

distancia s’ . Repetir la operación otras 4 veces más con valores distintos de s y consignar los datos en las dos primeras columnas de la Tabla 1.

2. (2 puntos) Hacer una representación gráfica en papel milimetrado de 1/ s’ frente a 1/s , y determinar la pendiente de la recta y la ordenada en el origen.

Tabla 1

s (cm) s’ (cm) 1/s (cm-1) 1/s’ (cm-1)

3. (1 punto) A partir de la ordenada en el origen, determinar el valor de la distancia focal f’ de

la lupa. 4. (1,5 puntos) Estimar las incertidumbres tanto en la pendiente de la recta como en la

ordenada en el origen y en la distancia focal, indicando el método seguido para ello. Segundo experimento: Determinación de la distancia focal por el método de Bessel Bessel ideó un procedimiento para determinar la distancia focal de una lente de manera más precisa, que consiste en lo siguiente: conocido un valor aproximado de la distancia focal (determinado, por ejemplo, por el método anterior), se fijan la vela y la pantalla a una distancia L mayor que cuatro veces la distancia focal. En estas condiciones, existen dos posiciones de la lente para las cuales se obtiene una imagen nítida de la vela, una de ellas de mayor tamaño que el objeto y otra de menor tamaño. Si llamamos D a la distancia que media entre estas dos posiciones, la distancia focal f’ se determina mediante la siguiente fórmula:

2 2

'4

L DfL−

=

Operaciones a realizar:

5. (1,5 puntos) Elegir la distancia L entre la vela y la pantalla. Determinar la distancia D entre las dos posiciones de la lente antes mencionadas con el mayor cuidado posible. Determinar con ayuda de la fórmula anterior el valor de f’.

Tabla 2

L (cm) D (cm) f’ (cm)

6. (1 punto) Calcular la incertidumbre en f’.

Tercer experimento: Determinación del índice de refracción n del vidrio Los constructores de lentes podían diseñarlas con la distancia focal deseada para la aplicación concreta. Se valían para ello de una fórmula, deducida también a partir de la ley de la refracción de la luz, que tiene en cuenta los radios de curvatura de sus superficies, así como del índice de refracción del material transparente del que están hechas. En el caso que nos ocupa, la fórmula del constructor de lentes se reduce a:

Page 184: XXIII olimpiadas españolas de física, enunciados y soluciones de las pruebas propuestas - pag 214.pdf

1 2( 1)

'n

f R= −

donde R es el radio de curvatura de las superficies esféricas que constituyen las caras de la lente. Operaciones a realizar:

7. (2 puntos) Con ayuda de la Figura 2 y del teorema de Pitágoras, determinar la expresión de R en función de las variables indicadas en la Figura 2.

8. (2,5 puntos) Para la

determinación experimental de h, e y eo, desmontar la lente desenroscando con cuidado el mango de la lupa y utilizar el calibre. Consignar los resultados en las tres primeras columnas de la Tabla 3. A partir de estos valores, determinar el valor de R, indicando su incertidumbre.

Figura 2: Esquema geométrico para el cálculo de R.

Tabla 3

h (cm) e (cm) eo (cm) R (cm)

9. (1,5 puntos) Aplicando la fórmula del constructor de lentes, determinar n, indicando

también su incertidumbre.

Solución de la prueba experimental Primer experimento: 1. (2 puntos) Colocar la lente entre la vela y la pantalla procurando que su centro se encuentre a la misma altura que la parte central de la llama. Para conseguirlo, se ajusta el mango de la lupa en el agujero del soporte, calzándolo con palillos. Se desplaza la pantalla sobre la cinta métrica hasta que la imagen de la llama aparece nítidamente. En estas condiciones se miden s y s’ y se consignan los datos en las dos primeras columnas de la Tabla 1. En la prueba se han distribuido dos tipos de lente: la lente A, de 7,5 cm de diámetro, y la lente B, de 9 cm. Cada alumno ha trabajado con una sola lente. En las Tablas 1A y 1B se proporcionan ejemplos de medida en cada caso.

Page 185: XXIII olimpiadas españolas de física, enunciados y soluciones de las pruebas propuestas - pag 214.pdf

0 1 2 3 4 5

1/s (m)

0

1

2

3

4

5

6

1/s' (m)

LENTE A

Y = mX+cm = -1.001990352

c = 5.127591797 m-1

0 1 2 3 4

1/s (m)

0

1

2

3

4

1/s' (m)

LENTE B

Y = mX+cm = -1.011932168

c = 3.730599356 m-1

Tabla 1A

s (m) s’ (m) 1/s (m-1) 1/s’ (m-1)0,25 0,894 4,00 1,12 0,30 0,561 3,33 1,78 0,35 0,440 2,86 2,27 0,40 0,380 2,50 2,63 0,45 0,346 2,22 2,89

Tabla 1B

s (m) s’ (m) 1/s (m-1) 1/s’ (m-1)0,35 1,180 2,86 0,85 0,40 0,840 2,50 1,19 0,45 0,680 2,22 1,47 0,50 0,580 2,00 1,72 0,55 0,530 1,82 1,89

2. (2 puntos) Se hace la correspondiente representación gráfica en papel milimetrado de 1/ s’ frente a 1/s (ver figuras adjuntas), y de ella se obtienen los siguientes datos:

Lente A: pendiente = 1,01 ; ordenada en el origen = 5,18 m-1

Lente B: pendiente = 0,98 ; ordenada en el origen = 3,70 m-1

3. (1 punto) 1ordenada en el origen

f =

fA = 0,193 m ; fB = 0,270 m B

4. (1,5 puntos) Pendiente teórica – pendiente experimental de la recta

Lente A : 1 – 1,01 = - 0,01 (1% de desviación) Lente B : 1 – 0,98 = + 0,02 (2% de desviación)

Segundo experimento: Se fijan la vela y la pantalla a una distancia L entre ellas mayor que cuatro veces la distancia focal determinada anteriormente. 5. (1,5 puntos)

Tabla 2A

L (m) D (m) f (m) 1,00 0,478 0,193

Tabla 2B

L (m) D (m) f (m)1,30 0,52 0,273

Page 186: XXIII olimpiadas españolas de física, enunciados y soluciones de las pruebas propuestas - pag 214.pdf

6. (1 punto)

21 24

D f Lf L DL f L D

⎛ ⎞ Δ Δ Δ⎛ ⎞= − ⇒ = +⎜ ⎟ ⎜ ⎟⎝ ⎠⎝ ⎠

Las incertidumbres en la medida de L y D debidas al aparato (cinta métrica) podemos tomarlas igual a ± 0,001 m. Por lo tanto:

Lente A: L = 1 m ; D = 0,478 m ; Δf / f = ± 0,006 ⇒ fA = 0,193 ± 0,001 m Lente B: L = 1,30 m ; D = 0,520 m ; Δf / f = ± 0,005 ⇒ fA = 0,273 ± 0,001 m

Tercer experimento: Se determina n a partir de la fórmula del constructor:

1 2( 1) 12Rn n

f R f= − ⇒ = +

7. (2 puntos) Por el teorema de Pitágoras, de la figura se deduce:

2 22 2 2 2 2 2( ) 2 ; con

2 2oh d e eR h R d h R d Rd R d

d+ −

= + − = + + − ⇒ = =

8. (2,5 puntos) Se desenrosca el mango de la lupa y se miden h, e y eo con el calibre. Se consignan los resultados en las tres primeras columnas de la Tabla 3.

Tabla 3A h (cm) e (cm) eo (cm) R (cm) R (m) 3,735 0,906 0,226 20,69 0,2069

Tabla 3B

h (cm) e (cm) eo (cm) R (cm) R (m) 4,465 0,972 0,282 29,07 0,2907

2 2 o

o

R h d h e eR h d h e e

⎛ ⎞Δ Δ Δ Δ Δ Δ⎛ ⎞= + = + +⎜ ⎟⎜ ⎟⎝ ⎠ ⎝ ⎠

Las medidas de h, e y eo con el calibre llevan una incertidumbre de aparato de ± 0,005 cm. Por lo tanto:

Lente A: ΔR / R = 0,06 (6%) ; Lente B: ΔR / R = 0,05 (5%) 9. (1,5 puntos) Con la fórmula indicada al principio de este experimento se determina n:

Lente A: n = 1,54 ; Δn / n = 0,07 Lente B: n = 1,53 ; Δn / n = 0,05

Page 187: XXIII olimpiadas españolas de física, enunciados y soluciones de las pruebas propuestas - pag 214.pdf

UNI VERSIDAD DEBURGOS

REAL SOCIEDAD ESPAñOLA DE FíSICA

XIIIOlimpiadaESP AÑOLADE F ISICA

XXXIII INTE RNACIONAL

XIII Olimpiada Nacional de Física Burgos, 21-24 de marzo de 2002

Dispositivo de Gay-Lussac para sus experiencias sobre la dilatación de los gases

Tratado de Física Elemental, E. Fernet, (París, 1885)

Prueba experimental Los gases ideales: Determinación del cero absoluto de temperatura

Page 188: XXIII olimpiadas españolas de física, enunciados y soluciones de las pruebas propuestas - pag 214.pdf

PROBLEMA EXPERIMENTAL

En el dispositivo experimental mostrado en la Figura 1a se encuentra encerrado un gas en un bote de refresco. Un esquema simplificado del experimento puede verse en la Figura 1b.

(a) (b)

Figura 1: Dispositivo experimental para el estudio de la dilatación de los gases

A la temperatura T0 de la escala absoluta (0º de la escala centígrada) el líquido del tubo en forma de ∪ tiene el mismo nivel en las dos ramas, lo que quiere decir que la presión P0 interior coincide con la presión atmosférica del exterior. En estas condiciones, el volumen ocupado por el gas es V0 = 0,333 litros (l ).

Calentamos el gas a la temperatura T de la escala absoluta (temperatura t de la escala centígrada) y observamos que el nivel del líquido en la rama de la izquierda desciende y en la rama de la derecha asciende (ver Figuras 2a y 2b).

(a) (b)

Figura 2: Dilatación del gas por calentamiento

Page 189: XXIII olimpiadas españolas de física, enunciados y soluciones de las pruebas propuestas - pag 214.pdf

Esto es debido a que la temperatura del gas ha aumentado (T = T0+ t), lo que ha provocado un aumento en la presión y en el volumen (P = P0+ ΔP; V = V0+ ΔV). Este fenómeno cumple la ecuación general de los gases ideales:

PV nRT=

donde n es el número de moles del gas encerrado y R la constante universal de los gases.

En la Tabla I se indica la diferencia de alturas h entre los niveles del líquido en función de la temperatura t del gas (para el análisis del experimento supondremos que t es la misma para todo el gas encerrado).

t (ºC) h (cm)

0 0 2 5,8 4 11,6 6 17,3 8 23,1 10 28,8

Tabla I: Diferencia de alturas h frente a la temperatura t del gas

A partir de los valores de h podemos determinar los valores de ΔV y ΔP para cada temperatura. Del análisis de estos datos se puede deducir el valor T0 correspondiente a 0ºC en la escala absoluta de temperaturas; es decir, se puede determinar el valor de la temperatura correspondiente al cero absoluto en la escala centígrada (que en la bibliografía es de – 273,15ºC). Conocido dicho valor experimental, se puede determinar el número de moléculas del gas encerrado en el bote de refresco.

Tabla de datos del experimento

Densidad del líquido ρ = 103 kg m-3

Aceleración de la gravedad g = 9,8 m s-2

Radio interior del tubo en ∪ r = 2,5 mm

Presión atmosférica P0 = 1 atm = 101325 Pa

Volumen del gas a 0ºC V0 = 0,333 l

Constante universal de los gases R = 0,082 atm l K-1 mol-1

Número de Avogadro NA = 6,0221367 ⋅ 1023 mol-1

Page 190: XXIII olimpiadas españolas de física, enunciados y soluciones de las pruebas propuestas - pag 214.pdf

Tareas a realizar

1. (3 puntos) Establecer una relación del tipo: A h + B h2= C t , y determinar el cociente B h2 / A h para alguna pareja de valores (t,h) de la Tabla I. Utilizar para ello la ecuación general de los gases ideales aplicada al estado inicial (P0,V0,T0) y a un estado genérico (P,V,T).

2. (3 puntos) Atendiendo al resultado obtenido en el apartado anterior y despreciando el término B h2, representar gráficamente h frente a t y obtener el valor de T0.

3. (3 puntos) Con objeto de hacer una determinación un poco más precisa (sin despreciar el término en h2), encontrar la relación que liga la variable y = (PV/P0V0) – 1 con la temperatura t de la Tabla I, y hacer una representación gráfica de y frente a t. Cumplimentar para ello la Tabla que se adjunta más abajo1. Determinar a partir de ella el nuevo valor de T0.

t (ºC) h (cm) ΔP (atm) ΔV (cm3) y = (PV/PoVo) – 1

0 0

2 5,8

4 11,6

6 17,3

8 23,1

10 28,8

4. (1 punto) A partir de esta última determinación de T0, calcular el número de moléculas que contiene el gas encerrado en el bote de refresco.

1 Para una mejor precisión en los cálculos, consignar al menos cuatro cifras decimales.

Page 191: XXIII olimpiadas españolas de física, enunciados y soluciones de las pruebas propuestas - pag 214.pdf

Hojas de respuestas

1. (3 puntos) Establecer una relación del tipo: A h + B h2= C t , y determinar el cociente B h2 / A h para alguna pareja de valores (t,h) de la Tabla I. Utilizar para ello la ecuación general de los gases ideales aplicada al estado inicial (P0,V0,T0) y a un estado genérico (P,V,T).

Page 192: XXIII olimpiadas españolas de física, enunciados y soluciones de las pruebas propuestas - pag 214.pdf

2. (3 puntos) Atendiendo al resultado obtenido en el apartado anterior y despreciando el término B h2, representar gráficamente h frente a t y obtener el valor de T0.

T0 =

(añadir la gráfica en el papel milimetrado que se adjunta)

3. (3 puntos) Con objeto de hacer una determinación un poco más precisa (sin despreciar el término en h2), encontrar la relación que liga la variable y = (PV/P0V0) – 1 con la temperatura t de la Tabla I, y hacer una representación gráfica de y frente a t. Cumplimentar para ello la Tabla que se adjunta más abajo. Determinar a partir de ella el nuevo valor de T0.

t (ºC) h (cm) ΔP (atm) ΔV (cm3) y = (PV/PoVo) – 1 0 0 2 5,8 4 11,6 6 17,3 8 23,1 10 28,8

T0 =

(añadir la gráfica en el papel milimetrado que se adjunta)

4. (1 punto) A partir de esta última determinación de T0, calcular el número de moléculas que contiene el gas encerrado en el bote de refresco.

N =

Page 193: XXIII olimpiadas españolas de física, enunciados y soluciones de las pruebas propuestas - pag 214.pdf

SOLUCIÓN

1.

0

0

0

T T tP P PV V V

= + ⎫⎪= + Δ ⎬⎪= + Δ ⎭

( )( ) ( )0 0 0

0 0 0 0 0

0 0

PV P P V V nR T tPV P V V P P V nRT nRt

P V V P P V nRt

= + Δ + Δ = +

+ Δ + Δ + Δ Δ = +Δ + Δ + Δ Δ =

0 0 0PV nRTPV nRT=

= ⎫⎬⎭

Pero: 0 00 0 0 0

0 0

PV tnR P V V P P V PVT T

= ⇒ Δ + Δ + Δ Δ =

Por otra parte:

212

P gh

V r h

ρ

π

Δ =

Δ =

De donde se deduce:

2 20 0 0

0

1 12 2

tP r h V gh gh r h PVT

π ρ ρ π+ + = 0

Reagrupando términos:

2 2 0 00 0

0

1 12 2

A B C

PVP r gV h g r h tT

π ρ ρ π⎛ ⎞⎜ ⎟+ + =⎜ ⎟⎜ ⎟⎝ ⎠

2

De la Tabla I: t = 10ºC ; h = 28,8 cm

2

1, 2263 J0,0080 J

AhBh

= ⎫⎬

= ⎭

2

0,0065 (0,7%)BhAh

=

6

Page 194: XXIII olimpiadas españolas de física, enunciados y soluciones de las pruebas propuestas - pag 214.pdf

2. 0 0

0 0

7,9239 PVAh t hT T

= ⇒ = t

0 2 4 6 8t (ºC)

10

0

0.1

0.2

0.3

h (m

)

h = A' tA' = 0,0288 m K-1

10 1

0

7,9239 7,9239 m0,0288 m K 275,1 K0,0288 m K

A TT

−−

′ = = ⇒ = =

3.

0 0 0

0 0 0 0 0

1PV T tPV PV T tT T PV T T T0

+= ⇒ = = = +

0 0 0

= 1PV tyPV T

− =

t (ºC) h (cm) ΔP (atm) ΔV (cm3) y = (PV/PoVo) - 1 0 0 0 0 0 2 5,8 0,0056 0,5694 0,0073 4 11,6 0,0112 1,1388 0,0147 6 17,3 0,0167 1,6984 0,0219 8 23,1 0,0223 2,2678 0,0293 10 28,8 0,0279 2,8274 0,0366

7

Page 195: XXIII olimpiadas españolas de física, enunciados y soluciones de las pruebas propuestas - pag 214.pdf

0 2 4 6 8 10t (ºC)

0

0.01

0.02

0.03

0.04

y

y = B' tB' = 1/To = 0,00366 K-1

00

1Pendiente 273,2 KTT

= ⇒ =

La diferencia con el valor medido en otros laboratorios:

273,2 – 273,15 = 0,05 K

4.

0 00 0 0

0

PVPV nRT nRT

= ⇒ =

1 1

1 atm 0,333 0,0149 mol0,082 atm K mol 273,2 K

lnl − −

⋅= =

218,97 10AN nN= = ⋅ moléculas

8

Page 196: XXIII olimpiadas españolas de física, enunciados y soluciones de las pruebas propuestas - pag 214.pdf

UNI VERS IDAD DE

B URGOSREAL SOCIEDAD ESPA ñOLA DE FíSICA

XIII OlimpiadaE SP AÑOL ADE F ISI CA

X XX II I IN TE RN A CI O NA L

Burgos, 21-24 Marzo 2002

__________________________________________________________________________________________ T-1) PROBLEMA MÚLTIPLE (10 puntos) MECÁNICA (2,5 puntos).- Un cuerpo de 400 g se deja caer desde lo alto de un plano inclinado 45º y con un coeficiente de rozamiento por deslizamiento 0,15; el cuerpo se encuentra a una distancia de 200 cm, medidos sobre el plano, de un resorte ideal de constante elástica 250 N/m. Después de rebotar en el resorte el cuerpo vuelve a subir por el plano. Calcule: a) La máxima compresión del muelle b) La posición, medida sobre el plano, del punto más alto que alcanza el cuerpo respecto de la posición inicial. g= 9,8 m/s2. CALORIMETRÍA (2,5 puntos).- Se dispone de una mezcla de hielo y agua, con 200 y 600 g respectivamente, a O oC. Se coloca la mezcla sobre una placa de vitrocerámica de 1 kW de potencia y se observa que a los 6,7 min comienza a hervir y que 15 min después solamente queda la mitad del agua en el recipiente. Si el calor específico del agua es 4186 J/kg oC, calcule: a) calor latente de fusión del hielo b) calor latente de ebullición del agua N.B. Desprecie el calor absorbido por el recipiente. ELECTRICIDAD (2,5 puntos).- En el dibujo se representa el esquema de un circuito de corriente continua, con los valores de las fuerzas electromotrices de las baterías, la fuerza contraelectromotriz del motor, así como los valores de las resistencias presentes, incluidas las resistencias internas de los aparatos. Calcule: a) La diferencia de potencial entre los puntos A y B b) La energía disipada en la resistencia de 5 Ω en 5

minutos.

ÓPTICA (2,5 puntos).- Un foco luminoso, que puede considerarse puntual, se encuentra en el fondo de un estanque, junto a su pared vertical; el estanque tiene agua, con 1 m de altura sobre el fondo, y una capa superficial de benceno de 20 cm de espesor sobre el agua. Si se mira desde cerca del borde del estanque, prácticamente en la vertical del foco, calcule: a) ¿ A qué profundidad veremos al foco? b) ¿ Qué radio tendrá la mancha luminosa en la superficie del estanque?. Índices de refracción: nagua= 1,33; n benceno= 1,50.

Page 197: XXIII olimpiadas españolas de física, enunciados y soluciones de las pruebas propuestas - pag 214.pdf

Soluciones T-1) Problema múltiple MECÁNICA (2,5 puntos) Aplicando el Principio de Conservación de la Energía, la energía inicial (solo potencial gravitatoria) se transforma en elástica más lo que se disipa debido al trabajo de la fuerza de rozamiento; la ecuación sería, si x es la compresión máxima del muelle: a) mg(2+x) sen 45 = 1/2 k x2 + μ mg cos 45 (2+x)

Sustituyendo:

0,4·9,8·√2/2·(2+x)= 0,5·250·X2 + 0,15·0,4·9,8·√2/2·(2+x) Resolviendo esta ecuación resulta: x= 0,204 m b) La ecuación correspondiente a la subida es:

1/2 k x2 = μ mg cos 45 (x+d) + mg(x+d) sen 45, donde d es la distancia que sube por el plano a partir de la posición inicial del muelle. Sustituyendo:

0,5·250·0,7152 = 0,15·0,4·9,8·√2/2·(0,715+d) + 0,4·9,8·√2/2·(0,715+d) de donde resulta que d = 1,43 m. Se queda pues (2,0-1,43) = 0,57 m más abajo (a lo largo del plano) que el punto de salida. CALORIMETRíA (2,5 puntos) En los primeros 6,7 min (= 402 s) la mezcla inicial pasa a ser agua a 100 oC, y por tanto la ecuación de este proceso será:

0,200·Lh . + (0,200 + 0,600)·4186·(100-0) = 1000·402 , de donde resulta que:

Lh . = 335 600 J/kg= 80,5 cal/g En los 15 min (= 900 s) siguientes pasan a vapor 400 g de agua que estaban a 100 oC, y por tanto:

0,400·Lv= 1000·900 de donde: Lv= 2 250 000 J/kg= 540 cal/g ELECTRICIDAD (2,5 puntos) Si se asignan las intensidades I1 por el conductor de la izquierda, I2 por el de la derecha e I3 por el central, y aplicando la ley de las mallas a las mallas derecha e izquierda del circuito resultan las ecuaciones:

I1 + I3= I2Malla izda recorrida en sentido anti-horario: -5= 7 I3 - 10 I1Malla derecha recorrida en sentido horario: 0= 11 I2 + 7 I3

Resolviendo el sistema resulta que:

I1= 0,350 A = 350 mA ( de B a A) (izdo) I2= 0,136 A= 136 mA ( de A a B) (dcho)

Page 198: XXIII olimpiadas españolas de física, enunciados y soluciones de las pruebas propuestas - pag 214.pdf

I3= 0,214 A= 214 mA (de A a B) (central) Con el valor de I3 se calcula el calor disipado en la resistencia de 5 Ω en 5 min(=300 s):

Q= I32 R t= 0,212 . 5. 300 = 66.15 J Para calcular VAB se puede hacerlo en cada uno de los tres conductores del circuito:

Conductor izdo: (VB - VB A) + 10= 0,35. (9 +1) Conductor central: (VA - VB) -15+ 10= 0,214. (5 +1+1) B

Conductor derecho: (VA - VB) -5 = 0,136. (10+1) B

De las que resulta en cualquiera de ellas: : (VA - VB) = 6,5 V B

ÓPTICA (2,5 puntos) Si se considera una observación prácticamente perpendicular, los ángulos de incidencia y refracción en cada superficie (agua y benceno) son muy pequeños, y por tanto la función seno puede igualarse a la tangente, y por tanto si escribimos las ecuaciones de Snell para cada caso:

Refracción en superficie agua-benceno: 1,33 sen α = 1,50 sen β =>

1,33 tg α = 1,50 tg β => 1,50 /1,33 = tgβ/tgα (1)

y como el ángulo de incidencia en la superficie benceno-aire también sería β, para la refracción en esta superficie se escribiría:

1,50 sen β = 1,00 sen ϕ => 1,50 tg β = 1,00 tg ϕ => 1,00/1,50= tgβ/tgϕ (2)

De (1) resulta que 1,50/1,33= h/1,00 => h= 1,13 m, profundidad a partir de la superficie superior del agua q la que se forma la imagen en la primera refracción. De (2) resulta la profundidad de la imagen respecto de la superficie del estanque:

H=1/1,5 * (1,13 + 0,2)= 0,885 m Para calcular la mancha del foco hay que calcular en primer lugar el ángulo límite de la refracción en la superficie benceno-aire: sen iL= 1/1,50= 0,667 => iL= 41,8º , y por tanto este debe ser el ángulo de incidencia en la superficie benceno-aire que determina el radio de la mancha. En la refracción agua-benceno el ángulo de refracción debe ser 41,8º y por tanto el ángulo de incidencia ξ sería:

1,33 sen ξ, = 1,50 sen 41,8º => ξ = 48,74º

Por tanto la distancia del foco a la que se produce la refracción agua-benceno será:

D= 1,00. tg 48,74º = 1,14 m

, y la distancia desde el punto de salida en el benceno hasta el de la refracción benceno-agua será: d= 0,20 tg 41,8º= 0,18 m, y por tanto el radio de la mancha del foco en la superficie será R= 1,32 m

Page 199: XXIII olimpiadas españolas de física, enunciados y soluciones de las pruebas propuestas - pag 214.pdf

UNI VERS IDAD DEB URGOS

R EAL SOCIEDAD ESPA ñOLA DE FíSICA

XIII OlimpiadaE SP AÑOL ADE F ISI CA

X XX II I IN TE RN A CI O NA L

Burgos, 21-24 Marzo 2002

________________________________________________________________________________________ T-2) De la Tierra a la Luna (10 puntos) En su famosa novela de 1865 Julio Verne imaginaba un viaje a la Luna en el que la nave espacial era una gran bala de cañón disparada desde un profundo pozo reforzado apuntado a nuestro satélite. Dejando a parte el freno del incandescente paso por la atmósfera terrestre, añadamos una simplificación más a la fantasía admitiendo que la Luna está quieta respecto a la Tierra. En este problema vamos a hacer algunos números para verificar la rudeza del método para la integridad de los tres ocupantes y la insensatez del proyecto.

1 Calcule a qué distancia l de la tierra se halla el punto de equilibrio entre las gravedades lunar y terrestre. (1 punto)

2 Llamando r a la distancia desde el centro de la tierra, calcule las contribuciones de la energía potencial ET(r) de la nave debida al campo gravitatorio terrestre, y de la lunar EL(r). (2 puntos)

3 Dibuje una gráfica a mano alzada de la energía potencial total E(r) describiendo sus características. (2 puntos)

4 Calcule la velocidad mínima VT con que debe partir la nave de la Tierra para alcanzar la Luna. (2 puntos)

5 Suponiendo que la puntería haya sido mejor que en la novela y realmente dieran en nuestro satélite, calcule la velocidad mínima de caída VL sobre la superficie lunar. (1,5 puntos)

6 Calcule la aceleración media durante el disparo en unidades de g y el tiempo del recorrido dentro del cañón. (1,5 puntos)

DATOS Radio terrestre: RT = 6,37·103 km Radio lunar: RL = 1,74·103 km Distancia entre sus centros: d = 3,84·105 km Masa de la Luna: ML = 0,0123 MT Gravedad en la superficie terrestre: g = 9,81 m s-2

Profundidad del pozo-cañón: h = 300 m

Page 200: XXIII olimpiadas españolas de física, enunciados y soluciones de las pruebas propuestas - pag 214.pdf

UNI VERS IDAD DE

B URGOSR EAL SOCIEDAD ESPA ñOLA DE FíSICA

XIII OlimpiadaE SP AÑOL ADE F ISI CA

X XX II I IN TE RN A CI O NA L

Burgos, 21-24 Marzo 2002

_____________________________________________________________________________________ T-3) INDUCCIÓN (10 puntos) El circuito de la figura consiste en una espira rectangular de 0,30 m de ancho y 1,50 m de largo situada en un plano vertical perpendicularmente al campo magnético B = 0,40 T dirigido hacia dentro. La parte del rectángulo que queda fuera del campo magnético tiene 0,10 m de longitud. La resistencia de la espira es de 0,20 Ω y su masa 0,50 kg. Se suelta la espira desde el reposo en el instante inicial t = 0. a) ¿Cuál es la magnitud y dirección de la

corriente inducida cuando la espira alcanza una velocidad vertical hacia abajo v?(1 pto)

b) ¿Cuál es la fuerza que actúa en la espira como resultado de esta corriente? (1 pto)

c) ¿Cuál es la fuerza resultante que actúa en la espira? (1 pto)

d) Escriba la ecuación de movimiento de la espira. (1 pto)

e) ¿Cuál sería la velocidad de la espira al dejar la región con campo magnético si hubiera descendido los 1,40 m en caída libre (B = 0)? ¿Cuánto tiempo habría tardado? (1 pto)

f) Sabiendo que la expresión de la velocidad de la espira en función del tiempo (con B ≠ 0) viene dada por

v(t) = vM 1−e−bt ,

calcule las constantes y . (1 pto) Mv bg) Sabiendo que la expresión del

desplazamiento vertical de la espira en función del tiempo (con B ≠ 0) viene dada por

x(t)= c1 t− c2 1−e−bt( )( ), calcule las constantes y . (1 pto) 1c 2ch) En realidad, ¿Cuánto desciende la espira en

el tiempo calculado en la pregunta e)? (1 p) i) ¿Cuanta carga habrá circulado por la espira

durante ese tiempo? (1 pto) j) Las expresiones dadas en las preguntas f) y

g) ¿Son válidas en cualquier instante de tiempo?¿Se pueden aplicar en las preguntas h) e i)? (1 pto)

N.B. Recuerde que ataeate

dtd

= si a es constante.

Page 201: XXIII olimpiadas españolas de física, enunciados y soluciones de las pruebas propuestas - pag 214.pdf

CORRIENTES INDUCIDAS A TRAVÉS DEL ESPACIO

Esquema del montaje experimental La bobina grande de la Figura, constituida por N = 400 espiras de cobre, de radio medio R = 13,21 cm (± 0,03 cm), está conectada en serie con una bombilla y un amperímetro A a la red de corriente alterna. Cuando el circuito está cerrado, por la bobina circula una intensidad de corriente I = I0 sen ω t, cuyo valor eficaz, medido con el amperímetro, es Ief = 0,832 A (± 0,001 Α). Esta corriente sinusoidal crea un campo magnético en el punto P del eje de la bobina, paralelo a éste y que varía también de forma sinusoidal con el tiempo t. Si en torno al punto P se coloca otra bobina de n = 500 espiras, de radio medio r = 1,53 cm (± 0,01 cm), sobre ella se genera una fuerza electromotriz inducida E = E0 cos ω t, cuyo valor eficaz Eef puede ser determinado con el voltímetro V. Dicho valor depende de la intensidad eficaz Ief , de la frecuencia angular ω y de la distancia d del punto P al centro de la bobina grande (ver Figura). La expresión que proporciona Eef es la siguiente:

Eef = A ω X(d), donde:

A = (μ0/2) π N n r2 Ief ; μ0/4π = 10-7 N A-2 ; X(d) = R2 (R2 + d2)-3/2

Page 202: XXIII olimpiadas españolas de física, enunciados y soluciones de las pruebas propuestas - pag 214.pdf

En la Tabla I se indica un ejemplo de medida de Eef para distintas posiciones del punto P sobre el eje de la bobina grande; d* es la coordenada del punto P respecto a un origen O arbitrariamente elegido (ver Figura). Haciendo uso de los datos de la Tabla I se pueden determinar los valores de la frecuencia angular ω, del número de ciclos por segundo de la corriente alterna (frecuencia ν de la red) y del valor máximo de la fuerza electromotriz inducida eficaz, Eef , que se obtiene en la bobina pequeña.

TABLA I

d* (m) Eef (10-2 V)

0,31 4,09 0,33 5,08 0,35 6,35 0,37 7,96 0,39 9,95 0,41 12,17 0,43 14,56 0,45 16,69 0,51 17,69 0,53 15,77 0,55 13,34 0,57 10,97 0,59 8,82 0,61 7,09 0,63 5,61 0,65 4,48

Para llevar esto a cabo se pide realizar las siguientes tareas:

1. Representar gráficamente Eef frente a d* en la hoja de papel milimetrado que se adjunta y, a partir de la representación, determinar el valor d0

* correspondiente al máximo de Eef . Consignar el resultado en la casilla correspondiente de la hoja de respuestas (hasta 3 puntos).

2. Calcular la coordenada d de cada punto P respecto a d0* y llevar los valores a la

Tabla II de la hoja de respuestas. Escoger 8 parejas de valores (d, Eef ) y determinar en cada caso la variable X(d), consignando el resultado en dicha Tabla (hasta 3 puntos).

3. Representar Eef frente a X(d) en papel milimetrado y determinar la pendiente de la línea recta que resulta (hasta 2 puntos).

4. Calcular el valor máximo de Eef , ω y ν, indicando sus correspondientes unidades (hasta 2 puntos).

Page 203: XXIII olimpiadas españolas de física, enunciados y soluciones de las pruebas propuestas - pag 214.pdf

HOJA DE OPERACIONES (no hace falta entregarla)

Se añaden columnas en blanco por si fueran útiles en los cálculos parciales.

d* (m) Eef (10-2 V) d (m) X(d) (m-1)

0,31 4,09

0,33 5,08

0,35 6,35

0,37 7,96

0,39 9,95

0,41 12,17

0,43 14,56

0,45 16,69

0,51 17,69

0,53 15,77

0,55 13,34

0,57 10,97

0,59 8,82

0,61 7,09

0,63 5,61

0,65 4,48

Page 204: XXIII olimpiadas españolas de física, enunciados y soluciones de las pruebas propuestas - pag 214.pdf

HOJA DE RESULTADOS Apellidos: Nombre: DNI: Resultado 1: Presentar la gráfica en papel milimetrado y proporcionar el valor de d0

*: d0

* = Resultado 2:

TABLA II

d* (m) Eef (10-2 V) d (m) X(d) (m-1) 0,31 4,09 0,33 5,08 0,35 6,35 0,37 7,96 0,39 9,95 0,41 12,17 0,43 14,56 0,45 16,69 0,51 17,69 0,53 15,77 0,55 13,34 0,57 10,97 0,59 8,82 0,61 7,09 0,63 5,61 0,65 4,48

Resultado 3: Presentar la gráfica en papel milimetrado y proporcionar el valor de la pendiente: Pendiente = Resultado 4: Eef

= ω = ν =

Page 205: XXIII olimpiadas españolas de física, enunciados y soluciones de las pruebas propuestas - pag 214.pdf

HOJA DE RESULTADOS

Resultado 1: Se hace la representación gráfica de Eef frente a d* eligiendo adecuadamente las unidades en los ejes, de manera que la figura ocupe el máximo espacio posible en la hoja de papel milimetrado. (Hasta 1,5 puntos.) Aplicando la simetría de la función teórica, seleccionando pares de puntos sobre líneas horizontales, se determina la posición del centro d0

*. Para ello se puntuará que haya dibujado una función continua que pase por los puntos de la gráfica. (Hasta 1,5 puntos.)

d0* = 0,484 m (± 0,001 m)

Resultado 2: Para calcular d basta que se dé cuenta en el propio esquema que: d = d* - d0

*. De esta manera calcula los valores de la tercera columna de la Tabla I. Este cálculo es muy rápido y no tiene ninguna dificultad. (Hasta 1 punto.) Para determinar los 8 valores de X(d), aunque no hay ninguna dificultad conceptual, su cálculo es laborioso. Se necesita una calculadora científica. Se valorará que razonen qué puntos han elegido (si son todos seguidos, si están en diferentes zonas repartidos, etc.) . Este apartado es el que requiere más tiempo y ha de hacerse con más cuidado. (Hasta 2 puntos.)

d* (m) Eef (10-2 V) d (m) X(d) (m-1) 0,31 4,09 - 0,174 1,674 0,33 5,08 - 0,154 2,089 0,35 6,35 - 0,134 2,619 0,37 7,96 - 0,114 3,285 0,39 9,95 - 0,094 4,095 0,41 12,17 - 0,074 5,027 0,43 14,56 - 0,054 6,004 0,45 16,69 - 0,034 6,876 0,51 17,69 + 0,026 7,151 0,53 15,77 + 0,046 6,376 0,55 13,34 + 0,066 5,419 0,57 10,97 + 0,086 4,456 0,59 8,82 + 0,106 3,592 0,61 7,09 + 0,126 2,868 0,63 5,61 + 0,146 2,286 0,65 4,48 + 0,166 1,828

Page 206: XXIII olimpiadas españolas de física, enunciados y soluciones de las pruebas propuestas - pag 214.pdf

Resultado 3: Se hace la representación de gráfica de Eef frente a X(d) en papel milimetrado y se proporciona el valor de la pendiente de la línea recta que resulta.

Pendiente = 2,45 × 10-2 V m Se valorará que tomen el origen como punto de la línea recta que se obtiene y que proporcionen adecuadamente las unidades de la pendiente. (Hasta 2 puntos.) Resultado 4: En primer lugar, determinamos el valor de X(d) para d0

*, es decir, pra d = 0:

X(d=0) = R2 / (R2 + 0)3/2 = 1/R = 7,57 m-1

A partir de la gráfica anterior, se puede determinar el valor de Eef correspondiente a este

valor, que será el valor máximo de Eef .

Eef (máximo) = 18,55 × 10-2 V

También puede determinarse a partir del valor de la pendiente:

Eef (máximo) = 2,45 × 10-2 V m × 7,57 m-1 = 18,55 × 10-2 V (hasta 0,75 puntos)

De la expresión Eef = A ω X(d) se deduce que la pendiente es A ω. Por lo tanto, si calculamos A (utilizando la expresión del enunciado) determinaremos el valor de ω:

A = (μ0/2) π N n r2 Ief = 2 π × 10-7 × π × 400 × 500 × (0,0153)2 × 0,832 =

= 7,689 × 10-5 m2 N A-1 (hasta 0,75 puntos)

Pendiente = A ω ⇒ ω = 318,6 s-1

ν = ω / 2 π = 50,7 s-1 (hasta 0,5 puntos)

Page 207: XXIII olimpiadas españolas de física, enunciados y soluciones de las pruebas propuestas - pag 214.pdf

Prueba teórica 1

Trineo sobre hielo Se ha construido un trineo de juguete de masa m, que se desliza por una superficie horizontal de hielo con un coeficiente de rozamiento cinético μ. 1.a Si su velocidad inicial es v0, calcule el tiempo que tarda en pararse, la distancia recorrida, y la energía disipada en función de m, v0, μ, y g. [1 punto] 1.b Dé los resultados numéricos para el caso m = 2,0 kg, v0 = 4,0 m/s, μ = 0,070. [1 punto] 1.c Explique por qué μ es adimensional (basta un renglón para ello). [1 punto] El trineo transporta un gran recipiente lleno de agua, con dos grifos cerrados, uno B en su base, y otro lateral L en su parte posterior, como indica la figura. Por sencillez, consideraremos que el trineo se desliza sin rozamiento (μ = 0), de modo que su velocidad de deslizamiento con los grifos cerrados permanece constante v(t) = v0. 2.a Se abre ahora el grifo de la base B, por donde sale el agua por gravedad a un ritmo ρ constante, de modo que la masa total del trineo con el agua que contiene disminuye* como m(t )= m -ρ t. Demuestre que† la velocidad del trineo sigue siendo constante. [3 puntos ] 2.b A continuación se cierra B y se abre L, por donde sale agua con el mismo ritmo ρ de antes a una velocidad horizontal va constante con respecto al recipiente. Aplique como antes el principio de conservación del momento para ver si el trineo se acelera y determine, en su caso, la aceleración en un instante t en función de ρ, va , y m(t). [4 puntos ] Incluyamos de nuevo en nuestras consideraciones el coeficiente de rozamiento que habíamos despreciado antes, de modo que en todo este apartado 3 se tiene μ ≠ 0. 3 Teniendo en cuenta que una fuerza F actuando durante un pequeño intervalo de tiempo Δt cambia el momento en una cantidad Δp = F Δt, calcule la aceleración del trineo con los dos grifos cerrados, con sólo el grifo B abierto, y por último, con sólo el grifo L abierto. [5 puntos]

------------------------------------------------------------------------------------------------------------------------------------------------------------------------------------

* Evidentemente, ρ es la masa de agua que sale por unidad de tiempo. Además, se consideran solo intervalos de tiempo tales que t < (m/ρ). †Se sugiere aplicar el principio de conservación del momento durante un pequeño intervalo de tiempo al conjunto trineo→trineo + agua expulsada.

Page 208: XXIII olimpiadas españolas de física, enunciados y soluciones de las pruebas propuestas - pag 214.pdf

T-1 Solución 1.a v(t)= v0– μ g t ⇒ t = v0/(μ g), x(t)= v0 t – (½) μ g t2 ⇒ x = v0

2 /(2 μ g), E = (½) m v0

2

1.b t = 5,8 s, x = 12 m, E = 16 J 1.c Porque es un cociente entre dos fuerzas, que tienen la misma dimensión. 2.a ( m(t) + ρ Δt ) v = m(t) (v + Δv ) + ρ Δt v ⇒ Δv = 0. 2.b ( m(t) + ρ Δt ) v = m(t) (v + Δv ) + ρ Δt (v – va) ⇒ (Δv/Δt ) = ρ va / m(t) = v/ ((m/ρ) - t). 3. A las consideraciones del apartado anterior hay que añadir la disminución de momento debida al rozamiento. Así, con el grifo B abierto se tiene: ( m(t) + ρ Δt ) v - μ m(t) g Δt = m(t) (v + Δv ) + ρ Δt v ⇒ (Δv/Δt) = - μ g, es decir, la aceleración es la misma esté B abierto o cerrado. Sin embargo, con el grifo L abierto:

( m(t) + ρ Δt ) v - μ m(t) g Δt = m(t) (v + Δv ) + ρ Δt (v – va) ⇒

(Δv/Δt) = ( ρ va / m(t) ) - μ g.

Page 209: XXIII olimpiadas españolas de física, enunciados y soluciones de las pruebas propuestas - pag 214.pdf

PRUEBA TEÓRICA 2

Tarragona, 30 de marzo de 2001 (12,00 h.)

EFECTO HALL

Page 210: XXIII olimpiadas españolas de física, enunciados y soluciones de las pruebas propuestas - pag 214.pdf

Prueba Teórica 2 Conducción, Inducción y Efecto Hall

I) Corriente eléctrica en el interior de un conductor.

La corriente eléctrica es un desplazamiento colectivo de cargas. Se define la intensidad, I, como la carga, Q, que atraviesa la sección transversal del conductor por unidad de tiempo: I = Q/t.

Microscópicamente, la corriente se debe a que una gran cantidad de portadores de carga, de carga q cada uno (electrones o iones), se mueven con una cierta velocidad media, v. Es fácil demostrar que la corriente viene dada por I = n q v A, donde n es el número de portadores por unidad de volumen y A el área de la sección transversal del conductor.

1.a) Calcule la velocidad media de los portadores de carga (electrones) en un conductor de Cu de sección A = 1 mm2 por el que circula una intensidad I = 2 A. [ 2 Puntos]

Datos: ρCu = 8,9·103 kg/m3; Masa atómica del Cu = 63 g/mol; NA = 6,022·1023 mol-1.

Cada átomo de Cu aporta un electrón. e =1,60·10 -19 C.

II) Inducción electromagnética.

Por otra parte, la ley de Faraday establece una relación entre la fuerza electromotriz, ε, (fem) inducida en un circuito cerrado y la variación temporal de flujo magnético, Φ, a través de la superficie limitada por dicho circuito.

El circuito de la Fig. 1 consiste en un alambre ABCD de resistencia despreciable doblado en forma de U (ver figura), por el que se mueve con velocidad constante, V, una varilla de longitud L y resistencia R. En la región existe un campo magnético uniforme, de intensidad B, perpendicular al plano del circuito.

2.a) Deduzca las expresiones de la fem inducida en el circuito, de la intensidad de la corriente que circula por él y de la potencia eléctrica disipada por efecto Joule. Así mismo indique justificadamente el sentido de la corriente inducida. [3 Puntos]

2.b) Para mantener uniforme la velocidad de la varilla, ¿con qué fuerza hay que actuar externamente sobre ella?. [2 Puntos]

III) Efecto Hall.

En la Fig. 2 se muestra una cinta conductora de anchura L y grosor d, por la que circula una corriente, I, en presencia de un campo magnético uniforme, B, perpendicular a la cinta.

AB

C D

B

R

Fig. 1

V L

3.a) Demuestre que entre los bordes de la cinta aparece una diferencia de potencial inducida B

I d

Fig. 2

L VH= BI/nqd (Efecto Hall). [2 Puntos] 3.b) ¿Cómo puede deducirse el signo de la carga

de los portadores a partir de la polaridad de VH ? [1 Punto]

Page 211: XXIII olimpiadas españolas de física, enunciados y soluciones de las pruebas propuestas - pag 214.pdf

T-2 Solución

1.a) nqA

Iv =

3283

233

Cu

ACu /mportadores10·51,8

10·6310·023,610·9,8 ===−M

Nn ρ m/s10·47,1 4−=v

2.a) dt

dΦε −= ; BLx=Φ ; Vdtdx

= ⇒ BLVdt

d=

Φ

BLV=ε ;

R

BLVI = Sentido horario

Potencia disipada: 2dis RIP =

2.b) R

vLBILBF22

m == ; opuesta a v

ectv = ⇔ 0mext =+ FF ⇒R

vLBF22

ext = Sentido de v

3.a) Un elemento de grosor xΔ equivale a la varilla del apartado anterior, con velocidad

⇒ vV =

nqA

IBLBLVVH == ; dLA = ⇒nqdBIVH =

3.b)

vq>0 B Fm - - - - - - - - - -

+ + + + + + + ΔV

I vq<0 B Fm

- - - - - - - - - -

+ + + + + + +

ΔV I

Page 212: XXIII olimpiadas españolas de física, enunciados y soluciones de las pruebas propuestas - pag 214.pdf

PRUEBA TEÓRICA 3

Tarragona, 30 de marzo de 2001 (13,00 h.)

DUALIDAD ONDA-CORPÚSCULO

Page 213: XXIII olimpiadas españolas de física, enunciados y soluciones de las pruebas propuestas - pag 214.pdf

Prueba teórica 3

Dualidad onda-corpúsculo

En el año 1924 el físico francés Louis de Broglie introdujo una hipótesis revolucionaria, según la cual a toda partícula atómica podía asociársele una onda, cuya longitud de onda dependía de su masa y de su velocidad. Esto significaba que los electrones podían difractarse como lo hacen la luz o los rayos X.

En un experimento realizado por H. Mark y R. Wierl en 1931, se aceleraron electrones con una diferencia de potencial de ΔV = 3,6·104 V. Estos electrones, después de pasar a través de la ventana colimadora (ver Figura), atravesaban una lámina de plata y se difractaban dando lugar a círculos

concéntricos claros y obscuros (anillos de difracción) sobre una placa fotográfica.

Para una distancia entre la lámina y la placa fotográfica D = 30 cm, el radio del primer círculo obscuro observado era r = 7,0 mm.

La teoría de la difracción establece que el seno del ángulo de difracción θmin, correspondiente al primer anillo oscuro, es proporcional a la longitud de onda λ de la onda difractada:

sen θmin = k λ,

donde el valor de la constante k para la lámina de plata empleada es 3,68·109 m-1.

1) Determine la longitud de onda de los electrones según este experimento. [5 Puntos]

2) Calcule la longitud de onda λB prevista para los electrones según la teoría de Broglie en este experimento y compárela con la obtenida en el apartado anterior. . [5 Puntos]

Datos: h = 6,63·10-34 J s; me = 9,11·10-31 kg; e = 1,60 x 10-19 C; c = 3,00·108

Page 214: XXIII olimpiadas españolas de física, enunciados y soluciones de las pruebas propuestas - pag 214.pdf

T-3 Solución

1) Dkr

kk=≈= minmin tgsen θθ

λ m1034,6 12−⋅=λ

2) ph

=Bλ

Cálculo clásico de p:

Vemm

Vemvmp ΔΔe

eee 22

=== m10·47,6 12B

−=λ

Discrepancia teoría-experimento: 2%

La velocidad de estos electrones es m/s10·12,12 8

e==

mVev Δ , no despreciable frente a c.

Cálculo relativista de p:

22e

22e cmpcVecm +=+ Δ ⇒

2

e2 ⎟⎠⎞

⎜⎝⎛+=

cVeVemp ΔΔ

m10·36,6 12B

−=λ

Discrepancia teoría-experimento: 0,3%